Jump to content

Wikipedia:Reference desk/Miscellaneous

From Wikipedia, the free encyclopedia

This is an old revision of this page, as edited by 71.74.181.173 (talk) at 22:10, 1 January 2009 (→‎pharmaceutical oath?). The present address (URL) is a permanent link to this revision, which may differ significantly from the current revision.

Welcome to the miscellaneous section
of the Wikipedia reference desk.
Select a section:
Want a faster answer?

Main page: Help searching Wikipedia

   

How can I get my question answered?

  • Select the section of the desk that best fits the general topic of your question (see the navigation column to the right).
  • Post your question to only one section, providing a short header that gives the topic of your question.
  • Type '~~~~' (that is, four tilde characters) at the end – this signs and dates your contribution so we know who wrote what and when.
  • Don't post personal contact information – it will be removed. Any answers will be provided here.
  • Please be as specific as possible, and include all relevant context – the usefulness of answers may depend on the context.
  • Note:
    • We don't answer (and may remove) questions that require medical diagnosis or legal advice.
    • We don't answer requests for opinions, predictions or debate.
    • We don't do your homework for you, though we'll help you past the stuck point.
    • We don't conduct original research or provide a free source of ideas, but we'll help you find information you need.



How do I answer a question?

Main page: Wikipedia:Reference desk/Guidelines

  • The best answers address the question directly, and back up facts with wikilinks and links to sources. Do not edit others' comments and do not give any medical or legal advice.
See also:


December 20

regional demographics

Greetings. I am searching for data on the number of ethnic groups in multiple regions, for example: How many ethnicities are represented / can be found in: Los Angeles, Miami, NY, London, etc. I have looked on the US census but have not found thorough / complete lists. Thank-you, Mike —Preceding unsigned comment added by 69.181.158.143 (talk) 03:58, 20 December 2008 (UTC)[reply]

I'm afraid the answer to your question is basically going to be "all of them." No, there probably aren't any Hi-Merima people wandering around New York, but it's almost certain that every European group is represented, along with most Native American tribes and at least one (usually more) ethnic group from every country in the world. That's quite a list, and it's impossible to compile in full because so many people are of mixed ancestry and don't know their family history (and that's assuming that you could, somehow, survey every person in New York). Cities like New York, London, and Los Angeles are not only huge, they're also immigration hubs and population magnets, so they'll inevitably have a diverse populace. You might find it easier—and more interesting—to look up information on randomly selected small cities, since they sometimes have unexpected concentrations of specific ethnic groups, and it's quite interesting to look at immigration patterns on a small scale. --Fullobeans (talk) 06:01, 20 December 2008 (UTC)[reply]

So my mother is terminally ill

My mother is possibly terminally ill with cancer. She might die within a few years. If so, that'll make my father a widower. This is quite uncommon, isn't it? I mean, it's more common for the man to die first, but out of my parents, it's almost certain my mother'll die first, yet they're still married and they're about the same age as each other. Is this the case? Please, tell me.-Nubile Servant (talk) 04:33, 20 December 2008 (UTC)[reply]

See List of countries by life expectancy and find where you live. In the more affluent parts of the world, the women outlive the men on average. In poorer countries, the gap is smaller, and in some cases, the men actually outlive the women. This is usally due to the inherent difficulties in child birth; in countries with poor medical care MANY more women die in child birth than in countries with good medical care. Incidentally, there was a time in the not-too-distant past when worldwide men, on average, were expected to outlive women. For example, in research I dug up while working on the article Plymouth Colony, historian John Demos quotes statistics showing that men usually outlived women in 17th century New England. Men there lived, on average 7 years longer than women (in modern USA, that number is about flipped, with the women outliving the men by 7 years) and women only had a 70% chance of seeing their 50th birthday. Men had an 85% chance of reaching the same age. --Jayron32.talk.contribs 05:06, 20 December 2008 (UTC)[reply]
(edit conflict, because I always get distracted before clicking "Save page") Apparently the life expectancy in Scotland is 75 for men, 80 for women. That's not too much of a difference, especially compared to extreme cases like Russia, which has a life expectancy of 59 for men and 73 for women. Worldwide, the five-year disparity seems to be fairly average (see: List of countries by life expectancy), and suggests that women have a slight tendency to outlive men, but not by much. Some families have a history of gender-specific disease, though, which will cause the life expectancy of someone in that lineage to differ from the national average. There are also local cultural factors which can come into play ("All the women in my neighborhood hang out smoking menthols at the bingo hall", "All the men in my town work in coal mines, shoot meth, and play Russian roulette on Tuesdays after work"). But here's some original research: the US has the same male/female life expectancy as Scotland, and I know quite a few men who've outlived their wives (though not in my family), and quite a few women who've outlived their husbands. I also know quite a few women who've beaten cancer, so here's hoping we can add your mom to the list. I'm sorry she's sick—that's never easy—and I wish you both luck. --Fullobeans (talk) 05:30, 20 December 2008 (UTC)[reply]
One thing to remember when talking life expectancies is that there also tends to be an age gap between couples. On average, the male tends to be older then the female. For examples the Age at first marriage shows a nearly 2 year gap in the US and slightly over 2 years in the UK. Obviously we're talking about averages here. If a 50 year old woman marries a 30 year old man, it's hardly surprising if the woman dies first. Nil Einne (talk) 12:18, 22 December 2008 (UTC)[reply]

As a quick way of getting some statistics, I just downloaded the raw data file for the biographies section of the Internet Movie Database. It lists marriage data for many of the people in the database. Of these marriages, 12,177 are listed as being ended by "his death" and 4,630 by "her death".

However, this may be unrepresentative of the statistics for people in general for several reasons. First, marriages in show business may really be different. Second, I expect the database contains more men than women, and the reason for termination of a marriage is more likely to be known if it is the death of the person who the database entry belongs to. Another source of error is that if both spouses happen to be in the database, their marriage will be counted twice unless one of the entries happens to be incomplete.

Incidentally, the number of marriages shown as terminated by divorce is 22,274. --Anonymous, 07:09 UTC, December 20/08.

St.Paul

After the baptism of St.Paul given by anania ,which city did st. paul go first?THANK YOU —Preceding unsigned comment added by 59.92.252.237 (talk) 12:28, 20 December 2008 (UTC)[reply]

Paul says only that he went to "Arabia" for three years. Presumably he means Arabia Petraea (as opposed to Arabia Deserta or Arabia Felix, so I'd assume he went to Petra. Adam Bishop (talk) 12:41, 20 December 2008 (UTC)[reply]
Actually, in Acts 9:19-30 it describes Paul's first days after baptism. It states that he spent some time in Damascus (which is where Ananais lived and baptized him) preaching in the synagogues. The Jews in Damsacus ran him out of town, whereupon he went to Jerusalem; however the Christians there feared him(since like, a week before, he had been having them tortured and executed!), and threatened to run him out of town. Barnabas stood up for him, and sponsored him. From there he went to Caesarea and then on to Tarsus. There have been a LOT of questions lately about Paul, most of this stuff can be looked up in the Bible. I tend to use this one online: Biblegateway. Its fully searchable, and you could just, you know, read Acts (which is about 2/3 about Paul anyways) or any other book you wish... --Jayron32.talk.contribs 01:44, 21 December 2008 (UTC)[reply]
Ah, I was reading the Wiki article which points to Galatians. Adam Bishop (talk) 07:41, 21 December 2008 (UTC)[reply]
It has long been noted that Acts of the Apostles' statements about Paul often contradict his own in the epistles. Luke may be the better - and more dramatic - writer, but Paul is the more reliable. For instance, Luke tells the Road to Damascus story three times in Acts, but with contradictions between versions. Paul never mentions such an event. B00P (talk) 09:32, 21 December 2008 (UTC)[reply]
Ah yes, Galatians. Well in there, it does say he went to Arabia first. Well, regardless, the answers are quite findable in the Bible. You could just look it up... --Jayron32.talk.contribs 03:16, 22 December 2008 (UTC)[reply]

Minefield → Shiretoko

<moved to RD/C> flaminglawyerc 22:44, 20 December 2008 (UTC)[reply]

It's nice to put a link when we do that. --Milkbreath (talk) 16:07, 21 December 2008 (UTC)[reply]


December 21

Is there such a thing as a bus/train hybrid?

I couldn't have been the first one to think of this. Imagine the time saved- Take road to crossing, jump on tracks, beeline to next town, take backroad shortcut, hop on tracks again. Town billboard touts Time saved. Trus to New York city! Faster than Amtrak. At Trus,- we earn your trust!.--THE WORLD'S MOST CURIOUS MAN (talk) 00:50, 21 December 2008 (UTC)[reply]

See Dual-mode bus, Trolleybus, and Tram. The first is probably exactly what you are talking about, but Trams (which run on city streets, often between traffic lanes) could be thought of as a hybrid bus/train, as could a Trolleybus, which is like a Tram, but not tied to tracks. --Jayron32.talk.contribs 01:37, 21 December 2008 (UTC)[reply]
No, Jayron, a dual-mode bus does not mean it can run on tracks. This question is asking about a road-rail bus. As you see at that link, they have been tried, but not with any success. The engineering requirements for a good bus and a good train are just too different. (Similarly with road vehicles that turn into boats or into airplanes. Okay, DUKWs have found a niche role as tour vehicles, but it's a very limited niche.) --Anonymous, 03:34 UTC, December 21, 2008.

In Adelaide they have a bus which leaves the road and goes onto its own guided trackway, its called the O-Bahn Busway. —Preceding unsigned comment added by 58.165.239.250 (talk) 04:57, 21 December 2008 (UTC)[reply]

  • Again, this does not run on railway tracks. --Anon, 21:51 UTC, December 21, 2008.
The one I know is developed by Hokkaido Railway Company in Japan. They are working toward the practical use. Watch this. Oda Mari (talk) 07:59, 21 December 2008 (UTC)[reply]
It might work in fairly isolated communities where trains are infrequent, but on more tightly controlled lines it would never work. However, making a bus and a train can work - for instance the British Rail Class 143 is built on a bus chassis. -mattbuck (Talk) 01:49, 22 December 2008 (UTC)[reply]

goats milk

waht would cause yeast in goats milk and how can i fix the problem —Preceding unsigned comment added by 124.179.180.71 (talk) 01:22, 21 December 2008 (UTC)[reply]

Yeast is floating around in the air all around you; it will alight on foods, and if it finds a friendly medium it will grow and multiply all on its own. You could perhaps pasteurize it... --Jayron32.talk.contribs 01:32, 21 December 2008 (UTC)[reply]
And any particular goat teat may have a yeast infection. If so, I suggest not drinking such milk until the goat is treated and healthy. I don't know that the yeast itself would be a prob, though, it's opportunistic infections that worry me. StuRat (talk) 14:55, 21 December 2008 (UTC)[reply]
Infections of goat mammaries are noticeably smelly and painful. Udderly disgusting. Polypipe Wrangler (talk) 11:03, 24 December 2008 (UTC)[reply]

Where did the word "gozangas" and another crude term originate?

What is the origin of the phrases "wham bam, thank you maam" and "gozangas"? As in "Hey, checkowtdat brawd wit da big gozangas!" and "with him, it's wham, bam, thank you maam!" These sound kind of silly and raunchy at the same time so I suspect one of those old WWII type burlesque shows.Sunburned Baby (talk) 03:50, 21 December 2008 (UTC)[reply]

And since we're on the subject, does anyone know where the word "broad" came from?Sunburned Baby (talk) 17:59, 23 December 2008 (UTC)[reply]

Sounds like a bastardized version of either congas or maracas. The second phrase sounds a bit "Island pidgin". WWII seems a bit too far back IMHO. 76.97.245.5 (talk) 08:38, 21 December 2008 (UTC)[reply]
To me, WWII doesn't seem far enough back for "Wham, bam, thank you ma'am"; I'd be surprised if it weren't older than that. (Why, didn't the twelfth-century women refer to Maimonides as "Rambam, thank you ma'am"? ;-)) Deor (talk) 00:33, 22 December 2008 (UTC)[reply]
Well, I suppose I'll have to take that back (the skepticism, not the parenthetical joke); the OP's guess of a WWII origin may well be correct, at least according to the dictionary cited in this posting. Deor (talk) 05:48, 22 December 2008 (UTC)[reply]
Oh, and this book dates gazongas—note the spelling, which seems to be much more common than yours—to 1978, although this one dates it to the 1960s. As this one points out, it seems to be one in an extensive series of terms that ultimately derive from bosom. Deor (talk) 06:13, 22 December 2008 (UTC)[reply]

St.Paul

How many times St.Paul tell about his conversion in the Book of Acts ? THANKYOU —Preceding unsigned comment added by 59.92.252.237 (talk) 06:05, 21 December 2008 (UTC)[reply]

This is the FIFTH "St.Paul" question in the last few days! Maybe it's time to stop and read the book? SteveBaker (talk) 06:31, 21 December 2008 (UTC)[reply]

Excuse sir , i have read the book —Preceding unsigned comment added by 59.92.252.237 (talk) 06:53, 21 December 2008 (UTC)[reply]

Why don't you just count how many times then? Adam Bishop (talk) 07:40, 21 December 2008 (UTC)[reply]
As I said in answer to your previous question, the Road to Damascus story appears three times in Acts, once when it happens, and twice more when Paul tells about it. The details are different each time. Paul, himself, never wrote about any such experience in the epistles. B00P (talk) 09:41, 21 December 2008 (UTC)[reply]
That would be because the Bible is a rambling incoherent mess...as is obvious to anyone who attempts to actually read it from cover to cover as I once did. It does not appear to contain historical truth - so you're really just asking what was in the minds of the original authors and of the hundreds of subsequent amenders and tinkerers with the book. SteveBaker (talk) 22:09, 21 December 2008 (UTC)[reply]
It rambles because its written by several dozen authors over the course of a thousand years. Go to your library, take 70 random books off of a few shelves, and read them all. You don't expect a consistant narrative there, do ya? The Bible was compiled to be in roughly chronological order, but there is not the expectation for it to be a cohesive story. Outside of the Pentateuch (which is likely written by dozens of authors and compiled over hundreds of years itself) each individual book is rather coherant and self-consistant. Even in the Pentatauch, there are coherant stories that have a beginning, middle, and end, but they don't quite match up to the beginnings and ends of the books. Remember, the Bible is not a story, its an Anthology. --Jayron32.talk.contribs 02:57, 22 December 2008 (UTC)[reply]
If you pick 70 random books on different topics - sure you get a mess. But if you take 70 random books on Quantum Theory - or 70 books about the Watergate Affair - then I expect a fairly consistent story with a reasonable degree of correlation on the basic facts. The bible can't even get straight such rather fundamental things as the 10 Commandments! You'd really think that the one time God told the world what the actual RULES were - carved them on actual stone tablets - that the people who wrote about it afterwards would have taken just the teensiest bit of effort to get the wording straight. But no - there are at least three different versions (wildly different actually) of such a fundamental mission statement - not just different wording - totally different commandments! If the book (as it and it's proponents claim) is 'The One True Book' - then it is indeed a horrible mess - even on very basic factual matters.
At any rate - for our OP, when you've read all three versions of the story about the Road to Damazcus - you've read all there is to be said on the matter. The inconsistencies are...inconsistancies...and that's because this is mostly a work of fiction written by a bunch of different people with different agenda's to push. If you've read it from cover to cover without preconceptions (as I have) then such 'enlightenment' as you're likely to get (zero, IMHO) has already been gotten - and asking a bunch of people on the Internet for more information is a road to nowhere because they have no sources of information that you don't already have in that book. It's like asking what happened to Harry Potter on his 50th birthday - if J.K.Rowling didn't write about it then that information does not and cannot ever exist. Worse still, the authors of the Bible have all been dead for a very long time...and what they wrote has been messed about with considerably since then (evidence the number of different versions of the Bible over the years). SteveBaker (talk) 15:21, 22 December 2008 (UTC)[reply]
"The inconsistencies are...inconsistancies..." - very good, Steve. I don't know if that was intentional or not, but I might turn it into a handy quote ("At the end of the day, inconsistencies are just that - inconsistancies"). Thanks. -- JackofOz (talk) 19:35, 22 December 2008 (UTC)[reply]
Surprisingly - that one was intentional! Too many people see an inconsistancy and imagine something deeper going on - building huge wobbly conspiracy theories (or even entire religions) on something that was just an author making a mistake in a work of fiction. Just head out to any Star Trek convention and you'll be amazed at the amount of time spent trying to see deeper meaning in outright inconsistancy on the part of an author. SteveBaker (talk) 22:14, 22 December 2008 (UTC)[reply]
Someone slap me if I ever use the form X is just that: X.Tamfang (talk) 17:29, 28 December 2008 (UTC)[reply]

Help

The battleship in the forground of the picture File:USS Iowa (BB-61) Preps.jpg is in fact Indiana and not Iowa. How do I fix this without getting reverted? —Preceding unsigned comment added by 75.31.26.179 (talk) 07:28, 21 December 2008 (UTC)[reply]

Start by providing a source for your claim. Algebraist 08:17, 21 December 2008 (UTC)[reply]
Its an observational thing, you need to look carefully at the bridge designs for both classes for the difference to show. Additionally, the text of the image caption suggests that the battleship Iowa is in fact in the background and not the foreground. —Preceding unsigned comment added by 75.31.26.179 (talk) 08:22, 21 December 2008 (UTC)[reply]
I take it the Indiana is meant to be in the background (captioned left to right), and here navy archives[1] state the same as the description in the file, "The Indiana (BB-58) & Iowa (BB-61) underway" then goes on to describe the Iowa. Do you have information to contradict this? It would help to have references or information that supports your view. Just saying "observation" is vague. Julia Rossi (talk) 08:26, 21 December 2008 (UTC)[reply]
No, Indiana is meant to be in the foreground, Iowa is supposed to be in the background. The description of the Iowa would be impossible to make from the location of the photographer: the paint scheme which is discussed can only be seen on the background ship, not the foreground, thus the caption in the Iowa article is incorrect. —Preceding unsigned comment added by 68.72.221.167 (talk) 03:21, 22 December 2008 (UTC)[reply]
I take it then, that you feel the navy has the wrong ship on their Iowa page. Will you take your concerns to the talk page of the USS Iowa (BB-61) article? Julia Rossi (talk) 06:09, 22 December 2008 (UTC)[reply]

St.Paul

In the first missionary journey of st.paul who were the two fellow workers with him? I got one fellow worker that is barnabas but i didn't get the other. I read the bible but i couldn't find the other . Please help me . THANKYOU —Preceding unsigned comment added by 59.92.253.102 (talk) 10:14, 21 December 2008 (UTC)[reply]

Titus ((Galatians 2:1–10). Available in the article, Saint Paul. Julia Rossi (talk) 10:41, 21 December 2008 (UTC)[reply]

Are you right? —Preceding unsigned comment added by 59.92.250.113 (talk) 11:10, 21 December 2008 (UTC)[reply]

About the second visit, not the first. This is not my thing, so read the article, especially that table with loads of links for you to follow. I stop short after command+F "Barnabas and" – Julia Rossi (talk) 11:25, 21 December 2008 (UTC)[reply]

i think it is not titus , it may be john mark —Preceding unsigned comment added by 59.92.241.214 (talk) 15:04, 21 December 2008 (UTC)[reply]

Why would the person post if she didn't think she was right? And, if you think it was John Mark, why are you asking us who it was?
Now, I'll admit to having a degree so I am probably further along with my memory, but John Mark accompanied them on one journey, and he got cold feet and went AWOL. This caused a split between Paul and Barnabas, the latter thinking that they could still use him. (And, later, in his last days, Paul includes Mark in those he wants to see, as he is "profitable for the ministry," so apparently the lad did grow up quite a bit later.)
Reasoning logically, Paul made 2 with Barnabas, and 2 with Silas, IIRC. Okay, so John Mark could only have fled on one of those, because the split was right after that. Therefore, it was Titus on the first, John Mark on the second. And, Silas on the 3rd and 4th.Somebody or his brother (talk) 17:24, 21 December 2008 (UTC)[reply]

Look - this is the SIXTH "St Paul" question this week. All of the information that exists about this person (who in all likelyhood is fictional anyway) exists in the pages of one book. Our OP claims to have read that book. That being the case - he/she is now in possession of all of the information there is to be had on the subject. If there are any remaining questions - then they cannot be answered because they are entirely in the minds of some long-dead authors. We are edging into the realms of 'trolling' here. SteveBaker (talk) 22:14, 21 December 2008 (UTC)[reply]

Thank you Somebodyorhisbrother, for righting my wrong. Julia Rossi (talk) 23:40, 21 December 2008 (UTC)[reply]

Download Music of Blue - All Rise

I can't Download the Music of Blue - All Rise. everybody asks money to download the song. can anybody say me the site from where i can dowload the song Blue - All Rise. THANKYOU —Preceding unsigned comment added by 59.92.241.214 (talk) 14:21, 21 December 2008 (UTC)[reply]

Sorry, buddy, their music is copyrighted. That means that if you want it, you'll have to pony up. (but your best "illegal" chance is torrents or similar) flaminglawyerc 14:55, 21 December 2008 (UTC)[reply]

Daily Wages

equality among men and women in daily wages —Preceding unsigned comment added by 59.92.241.214 (talk) 15:38, 21 December 2008 (UTC)[reply]

Sounds like you're trying to use a search engine. The refdesk is staffed by real people... But as to your answer, please see the article occupational sexism. flaminglawyerc 16:25, 21 December 2008 (UTC)[reply]

Odd recurrent music in online features

<moved to WP:RD/E>

It's nice to put a link to the section. --Milkbreath (talk) 20:37, 21 December 2008 (UTC)[reply]

Mind relaxation techniques

well I am a 22 year old working in a reputed software concern. I am working nearly 15-18 hours daily. I would kindly request you to give me some suggestions on how can manage my time and some mind relaxation techniques. I would be very much pleased if you provide me with simple actions which I can add up in the routine life which will make me feel relaxed. —Preceding unsigned comment added by 220.227.68.5 (talk) 18:12, 21 December 2008 (UTC)[reply]

Did you read the article about meditation? The first line says: "Meditation is a mental discipline by which one attempts to get beyond the conditioned, "thinking" mind into a deeper state of relaxation or awareness." However, personally I would say to you: work less. Not being able to relax properly is a sign that you're working too much. Lova Falk (talk) 20:32, 21 December 2008 (UTC)[reply]
To be honest, 15-18 hour working days are dangerous to your mental/physical health. How many days are you working a week? Are there no regulations for your working hours? Exxolon (talk) 20:36, 21 December 2008 (UTC)[reply]
Have a look at our articles burnout (psychology) and stress management. There is also a fascinating bit on stress balls, which, to quote the article "are presented to employees ... as gifts". Mind you, you may prefer to squeeze your own in an emergency, unless, of course, it were to be a software concern. --Cookatoo.ergo.ZooM (talk) 21:42, 21 December 2008 (UTC)[reply]
Let me tell you something - I've been a software engineer and a team leader since before you were born and I take my job very seriously. I've worked in - (and run) many projects with horrible deadlines and a need to work 'crunch time'. Once you start working more than about 70 hours per week, within just a couple of days you'll be so unproductive (making mistakes - failing to notice things that are important) that you'll actually get your work done in a shorter time if you work fewer hours. I've seen this happen too many times. You can productively work 60 hours a week for several months - and you can productively push it up to 70 hours a week for a few weeks. But over 70 hours a week - or continuous 'crunch' over too many weeks is not doing either you OR your job function any good at all. If you utterly have to do this - a short nap (20 minutes maybe) works wonders. A caring employer may be able to set aside a quiet room with sofa's and such to let employees do this whenever they need to - it really helps when you are pushing it out over 60 hours a week. SteveBaker (talk) 22:04, 21 December 2008 (UTC)[reply]
Kevin Rudd needs to read this. He seems to have gotten by on 3 hours a sleep a night over the past year, which works out to a 147-hour week (mind you, he has a lot of problems to contend with, many of them legacies of the previous government) but also expects his staff to be there at any hour of the day, which is why a stack of them have become burnt out and quit [2]. -- JackofOz (talk) 22:40, 21 December 2008 (UTC)[reply]
As a programmer who has worked many 80+ hour weeks, I've found one technique that radically improved my productivity: staying out of the office during normal working hours. If I was at work 9-5, I'd be called into meetings, asked to help other workers, then waste more time by being bitched out by the boss for not meeting my deadlines. I would spend weekends and evenings at work, but was at home during the 9-5 period (or maybe 7-7 period, to allow for early birds and people who stay late). When you present this to the bosses as the only way to finish the project on time, they become remarkably agreeable, all of a sudden.
As for stress relief, I liked to walk around the building to work off stress. I tried to time it when my program was running a test or something like that, and also would get something from the vending machines during the walk, so I had an "excuse" if asked why I was walking around.
I'm also prone to getting finger cramps, so I would occasionally shake my hands with my fingers flailing about wildly (is there a name for this exercise ?). This really helped. StuRat (talk) 04:00, 22 December 2008 (UTC)[reply]
When your hands start to crap out on you - it's definitely time to stop. I've suffered all sorts of repetitive strain problems after 35 years of keyboard pounding. Learning to manage that is vitally important if you still want to be using a keyboard 20 years from now. SteveBaker (talk) 15:03, 22 December 2008 (UTC)[reply]
I'm hoping speech recognition will actually work in 20 years, unless MicroSoft buys up all the patents so they can put out their own defective versions. StuRat (talk) 02:17, 28 December 2008 (UTC)[reply]
Wow! That must be what modern slavery is like. Most of my working life I've rarely exceeded 50 hours in a week, usually managing to keep it to 40-45 hours - and I've usually managed to make my deadlines, though on a few occasions it has gone wrong (eg. very late arrival of requirements specs after weeks of asking) and I have had to do some extra hours or work a couple of weekends. What the OP (and StuRat) needs to ask himself is: "what the worst that would happen if I went home now and picked this up tomorrow?". In most cases they will find the answer is nothing; the work will still get sone and the deadline will still be achieved. A couple of hints: Don't provide (or be forced into accepting) unrealistic estimates based on you working a 80-hour week. If the deadline is too tight or you are going seriously off target, tell the project manager or your boss as soon as you can. Astronaut (talk) 14:00, 22 December 2008 (UTC)[reply]
I have blown off some crazy overtime assignments. For example, I was asked to work over the holiday season 1999 checking for Y2K bugs, but found out this company planned to lay me off soon after. I called in sick, instead. Alas, they didn't have any Y2K bugs, what a shame. StuRat (talk) 01:38, 24 December 2008 (UTC)[reply]
Crazy overtime (aka 'Death march' and 'Crunch time') is endemic to some parts of the computer software business. Computer games companies are particularly bad at this. There is an utter-utter-drop-dead date to get computer games done in...if you don't get it done in time for Xmas - your sales will either be a tiny fraction of what they would otherwise be - or you have to delay release for an entire year - which will cause horrible liquidity problems for your company AND risk your game being 'trumped' by some other product in the meantime. What makes this worse is the open-ended nature of a computer game. They are NEVER "done" - we always could have made the game a little better if only we'd had more 'polish' time...another level...more easter eggs...more variety in the AI behavior of the bad guys...nicer graphics...more objects that react under game-physics...another sound effect. There is always SOMETHING that you could do. So sadly - for a few months at the end of pretty much every game project, you go into crazy crunch-time - 60 to 70 hour weeks (hopefully no more unless you have very dumb management). It doesn't matter how well you planned coming up to the deadline because the game is never, ever finished. So the pressure to put in just a few more hours to get that fancy last-minute effect in there is unrelenting and hard to resist. What's amazing is that the games business doesn't pay overtime...people do this mostly because of the love of the project and the peer pressure of not letting down your buddies. People will quite voluntarily put in more hours that would ever be demanded of them - and it often takes forceful pressure from team leaders to limit what people will do. When they get tired they make mistakes that cost more hours to fix - before you know it, each hour worked late at night takes more than an hour to fix up the following morning.
But over 70 hours is just plain stupid - and doing it for longer than a few weeks is also just plain stupid. A few games companies are starting to realise this and while we all know that long crunch hours just prior to a release is well worth it in terms of review scores and sales success - the resulting 'burn-out' of your best people isn't worth it in the longer term.
SteveBaker (talk) 15:03, 22 December 2008 (UTC)[reply]
As I suggested above, part of the problem is poor estimating. Many times I've been asked "how long will it take" and I used to say "I'll have it done this evening" (or "Friday", or some other guess) without giving too much thought to it. It then turns out to be trickier than I thought and end up staying late to meet that unrealistic dealine. The trouble with this approach is that I'm giving the project manager what I think he wants to hear, not a realistic estimate. For future projects, that project manager comes to see us software engineers as miracle workers and estimates according to past history... and we get a project doomed to be late from the start. Now, I try to as vague as possible, have a good look first to see what is involved, and if I'm really pressed for an estimate I give myself lots of contingency to get the job done comfortably. If I finish up early, I look like the genius I am and I get to go home early; and if it turns out to be a difficult task, I don't get management breathing down my neck a week later. Astronaut (talk) 18:46, 22 December 2008 (UTC)[reply]
Yes - that's a classic problem. I know I can't do time-estimation to save my life. Nowadays I take a good hard look at what I honestly think it'll take - and double it. Only recently (in my previous job) did I discover that my boss was privately doubling what I told HIM - and still it sometimes took longer than it should. My best advice is to have short 'sprints' with well defined goals and to split the task into sprint-sized chunks. Treat the end of each sprint as a deadline - and work hard to meet each one. If the work doesn't fit the time available (and it never does) then the 'crunch' happens in little bits at the end of each sprint rather than piling up at the end. We use 3 week sprints - and you might find yourself working late a few nights at the end of every sprint - but when the last sprint is done - we're DONE...except that we won't be - for the reasons I outlined above. No computer game (and precious few computer programs of ANY kind) are ever "Finished". SteveBaker (talk) 22:09, 22 December 2008 (UTC)[reply]

Income of felons/nonfelons controlled for other demographics?

I can understand that released felons as a group have substantially lower income than non-felons, but are there any comparative income data controlling for other demographic factors? (e.g. education, age, race, sex, etc) —Preceding unsigned comment added by Boomerpdx (talkcontribs) 22:23, 21 December 2008 (UTC)[reply]


December 22

Chicago vs Salt Lake City

Is Chicago's numbering system for its streets anything similar to the one Salt Lake City has? 75.169.197.68 (talk) 01:47, 22 December 2008 (UTC)[reply]

Many cities use a numbering system of some sort for its streets, but Salt Lake City (and indeed most of Utah) takes the cake as far as maddening adherance to the grid. Chicago's is pretty regular as well, but not exactly so. The numbers on Chicago streets ONLY apply to the east-west streets on the South Side (and some on the southside are named, and their "numbers" are skipped, like Roosevelt, Cermak, and Garfield) while all of the north-south streets and all of the streets north of Madison Street are named, and not numbered, meaning you have to "memorize" the name-number correlation. See Streets and highways of Chicago which has an explanation of the system in Chicago. I lived there for 2 years, and I have to say it was a GREAT system, by far the easiest major city in the U.S. to navigate. I personally find the Utah system confusing (for example, confusing the addresses between 1500 North 2100 West Street and 2100 North 1500 West Street and 1500 West 2100 North Street and... you get the idea) and would prefer a system which was a little easier to keep track of... --Jayron32.talk.contribs 02:46, 22 December 2008 (UTC)[reply]
Ah, I see. I've lived in Salt Lake for eleven years and personally I find its numbering system to be quite easy. Using your example, the first address (1500 N 2100 West) is fifteen blocks north of the north-south line of the city, and then on a street that is twenty-one blocks west of the east-west line, while the second address is located on a totally different street (1500 West) and that location is twenty-one blocks north of the line on that street (if that makes any sense). If someone tells me to go to a house on 11400 S 2000 East I know exactly where that is. However, saying you live on a house that's on 2150 East 3000 South certainly does not have its appeal :) and sometimes addresses like 100 West North Temple are just plain confusing. But one gets used to it - 75.169.197.68 (talk) 03:03, 22 December 2008 (UTC)[reply]
My brain tends to confuse the numbers easier. Its easier to know that, for example, 2400 W Roosevelt means the corner of Roosvelt and Western, because its only got the one number in it. Even like 2400 W 23rd Street (corner of Western and 23rd Street, or one block south of Cermak on Western) is easier to keep track of because it uses a Cardinal number for the address and an Ordinal number for the street name. Having both the address and street name to be part of the same class of words makes it hard for me to parse. I suppose I would have gotten used to it had I lived there, but in general I find the system somewhat confusing. --Jayron32.talk.contribs 03:11, 22 December 2008 (UTC)[reply]
So a Salt Lake address always has a redundant '00'? — Street signs in San Francisco have a little number showing what block you're on; this can be important north of Market Street, where numbers don't run in parallel (because Market Street is oblique). If I were in charge they wouldn't all be n00: the four signs around a crossing would be 498, 499, 500, 501 so that you can see immediately which side is odd and which end of the block is which. —Tamfang (talk) 19:24, 28 December 2008 (UTC)[reply]

Credit music to The Incredibles

What time signature is the music played during the ending credits of The Incredibles in? (you can listen to it on the Amazon page) 72.200.101.17 (talk) 02:06, 22 December 2008 (UTC)[reply]

It's all over the place. I heard 5/4 and 6/8, I believe, and maybe 4/4, too. Dynamic piece, and a great movie, dahlink. --Milkbreath (talk) 11:40, 22 December 2008 (UTC)[reply]
Can't remember it, and the link is not working, but it's possible it is in a mixed meter (okay, try Time_signature#Mixed_meters). That could be why people can't agree on the time signature. -- 128.104.112.113 (talk) 19:43, 22 December 2008 (UTC)[reply]
The link worked for me. Under the "Incredibles" logo on the left is a link "Listen to Samples". "The Incredits" is the one. --Milkbreath (talk) 21:52, 22 December 2008 (UTC)[reply]

Movies in the Great Depression

This is brief: Who could afford movie tickets during the Great Depression?

Thank as always.


Always Cardinal Raven (talk) 03:03, 22 December 2008 (UTC)Cardinal Raven[reply]

Considering they likely cost a dime or so, probably most people. This article: [3] reports that a couple could get two movie tickets for $0.30, or $0.20 for a matinee. even that was affordable to most people. The first $1.00 movie ticket wasn't recorded until the 1960's: see [4]). Also see [5] where it notes that by the end of the 1930s, 80 million tickets were sold annually. The answer is obviously "a lot of people". Its pretty useless to speculate that "there is no way they could have afforded it" when the evidence is already clear that they did afford it. Also, remember that, in the 1930s, there were less things to spend money on. Consider that today you spend money monthly on cable TV, internet, sattelite radio, i-tunes subscriptions, NetFlix, land-line phones, cell phones, as well as buying the devices to use those, such as video game systems, computers, Televison sets, DVD players, CD players, MP3 players, etc. etc. In 1935, you had a) A phonograph b) a radio and c) the movies. And that was literally IT. So, the entertainment budget went a lot farther, since you didn't have a multitude of different media to spend it on. A dollar a month could get you all the movies you needed to see and then some... --Jayron32.talk.contribs 03:27, 22 December 2008 (UTC)[reply]

See that is what I expected, just had to make sure. Another question certain movies that come from 1930s and what not. If you ever watched them it appears the actors are talking really fast for a normal human being. Was this because they had a time limit of when to get their lines in? Why did they speak so fast? Thank you again. Its always appreciated.

Always Cardinal Raven (talk) 03:34, 22 December 2008 (UTC)Cardinal Raven[reply]

There was a certain "rapid banter" that was quite common for a few decades (Catherine Hepburn was one offender). Still, some people, like comedians Robin Williams and Dennis Miller, do the same thing today. I would like to sic them on each other and see which one explodes first. StuRat (talk) 03:50, 22 December 2008 (UTC)[reply]
Howard Hawks, in particular, is known for the rapid, overlapping dialogue in his films, with His Girl Friday being the locus classicus. Deor (talk) 04:18, 22 December 2008 (UTC)[reply]
Its also not confined to Hawks, or indeed to films of the 1940s. More modern directors who used similar techniques include Robert Altman, noted for such use of dialogue in MASH (1970), and Whit Stillman, who was famous for his "talky" films of the 1990's, including Metropolitan (1990) and Barcelona (1994). --Jayron32.talk.contribs 05:07, 22 December 2008 (UTC)[reply]
The Gilmore Girls also did that. Adam Bishop (talk) 08:14, 22 December 2008 (UTC)[reply]
The frame rate might have been different. Movie cameras from that era might have run at between 15 and 20 frames per second, but playing them back on modern equipment at 24 or 25 frames per second makes everying seem faster. In a dialogue scene, it would seem like rapid banter. Astronaut (talk) 13:26, 22 December 2008 (UTC)[reply]
This may be the case with movies from the silent era, but by the time of the talkies, the frame rate had been standardised at 24fps. (The WP article I've linked to references silent-film historian Kevin Brownlow as the source for this statement). Valiantis (talk) 14:38, 22 December 2008 (UTC)[reply]
Remember that the $1 movie ticket in the 1960's translates to a $7 movie ticket today - because inflation has pushed down the value of money by a factor of 7 over that same period. It's hard to track inflation through the depression - but it's highly likely that the dime it cost for a movie ticket back then would be more like a dollar today. You'd have to be REALLY badly off not to be able to afford $1 in present-value money. There is evidence that distractions like TV, movies, computer games and such actually do better in times of financial crisis than they do in boom times because they are amongst the cheapest forms of entertainment - and people need (more than ever, actually) a way to take their minds off the daily grind and spend a couple of hours zoned out in a comfy chair with popcorn.
Another reason why it would have been popular in the 1930's would be that this was the only way for most people to see news footage - newsreel theatres did boom business because it was the only way to SEE the larger events of the world - and for people who probably didn't even own a radio, it was the only way to hear it either.
SteveBaker (talk) 14:38, 22 December 2008 (UTC)[reply]
SteveBaker (talk) 14:38, 22 December 2008 (UTC)[reply]

People had to have some relief from the staggering harshness of the depression. Some would take their main squeeze to a honky tonk, others to a movie, spending the price of a filling meal to have some joy in a stark world. Some sacrifice might have been involved. They skip a meal, or walk 3 miles to save bus fare, or put pieces of cardboard in the worn out shoes instead of having them half-soled. Edison (talk) 02:39, 23 December 2008 (UTC)[reply]

Japanese Criminal Tattoos

Ive searched far and wide and can't seem to find any specifics on the tattooing of criminals in Edo period Japan. I only know that different markings or particularly bands were given for certain crimes committed. Bokukei or bokkei is the term thats used for the punishment by tattoos, but I can't seem to find any information regarding exactly which tattoos were given for which crimes. Ive only ever come across a book that showed an old illustration with various markings, but I couldnt translate any of the wording. Mugen, a protagonist in the Samurai Champloo anime is a good example. He has blue bands around his wrists and ankles, but other than him, I know no examples of reference. I've tried contacting traditional japanese tattoo websites for information, but have never received replies. plz halp. —Preceding unsigned comment added by 76.177.106.51 (talk) 03:03, 22 December 2008 (UTC)[reply]

Maybe following threads from our Yakuza article will help. Irezumi has a bit of info, but no examples. [Criminal punishment in Edo-period Japan] only mentions tattoos for prisoners. This site [6] had following info: The original punitive markings on criminals, usually in the form of rings around the wrists, or lines down the arm, were called Geishin. another source [7] had this, aparently from a different period: First offenses were marked with a line across the forehead. A second crime was marked by adding an arch. A third offense was marked by another line. Together these marks formed the Japanese character for "dog". It appears this was the original "Three strikes, you're out" law. For googling it would help if you got in touch with s.o. who can read Japanese. 76.97.245.5 (talk) 05:36, 22 December 2008 (UTC)[reply]
Link #13 is a past version of our article tattoo. As for the letter 'dog' in Japanese, the stroke order is 「一」「ナ」「大」「犬」. It has four strokes. Tattoos as criminal punishment started in the middle of Edo period. A man named Hanbei was the first criminal who received the punishment. It was May 11, 1720. Where the tattoos were marked was different from place to place. The dog letter was used in Hiroshima and when the criminal committed the 5th crime, it meant death. Horizontal line/s on forehead was used in Tokushima. X mark/s on forehead in Hizen. And band/s on arm, mostly on the left arm, was used in many places for theft. See this. Oda Mari (talk)10:22, 22 December 2008 (UTC)[reply]
The irezumiwaku link is precisely what I'm talking about! Would a translation of that even be possible? The reference is perfect except that I cant determinine what any of the tattoos mean. thank you for the help.
@oda mari Our Irezumi article says that tatooing criminals began in the Kofun period but doesn't cite any references. Since you have such detailed info maybe you should add that to the article. 76.97.245.5 (talk) 12:29, 22 December 2008 (UTC)[reply]
As for the linked image, almost all words are place names. The top line from l to r: 京都/Kyoto, 人足寄場/Ninsokuyoseba/vagrant camp,?代/unreadable, 甲府/Kofu, 江戸(割増?)/Edo (additional tattoo?), 江戸/Edo. The second line: 大阪/Osaka, 伏見/Fushimi, 長崎/Nagasaki, 奈良/Nara, 駿河/Suruga, 堺/Sakai. The bottom line: 長州/Choshu, 筑後/Chikugo, 紀州/Kishu, 非人/hinin, 日光/Nikko, 佐渡/Sado. I wrote the fifth time means death, but some sites say it was the fourth and this one was sentenced death after his second arrest. Oda Mari (talk) 14:55, 22 December 2008 (UTC)[reply]

School teachers

Why is that either 1 or 2 of 100 techer of physics know how to derive why like charges or poles(magnetics) repel and unlike attract but say like repel and unlike attract —Preceding unsigned comment added by 210.212.126.39 (talk) 06:34, 22 December 2008 (UTC)[reply]

Do you have sources for that statistic? Dismas|(talk) 08:17, 22 December 2008 (UTC)[reply]
To respond to the OP. The answer to "Why do opposite charges attract and like charges repel?" is not that complicated, and that your physics teacher has not explained it satisfactorily is probably because the real answer itself is so simple that it seems like an unsatisfactory answer. The real answer is "Because that's the way it works". Let me expand a bit on that, but its not any more complicated. The reality is, there are two fundemental Electric charges, and these charges either attract or repel one another. If we look at the atom, and look at say a proton and an electron, we can observe that protons will repel other protons, and attract electrons. Likewise electrons repel other electrons, and attract protons. This is an observation, and it is upon observation that all scientific thought is based. Now, the property of that attraction is called "electric charge" and, given that there appears to be two fundemental and complementary ways that charge works (i.e. the charge that attracts electrons and repels protons VERSUS the charge that attracts protons and repels electrons), we could call them anything we want, black and white, Peter and Stewie, A and B, whatever, but + and - has the advantage of also being mathematically useful in the sense that we can quantify this property called "electric charge" and then do real, useful math. By convention (and somewhat randomly) a proton's charge is called "positive" and an electron's charge is called "negative" and then all other item's charge is compared to these two; i.e. if it behaves like a proton does it is +, if it behaves like an electron it is -. Other articles that you may find interesting if you want to explore the nature of electric charge are probably electromagnetism, elementary charge, and quantum mechanics. --Jayron32.talk.contribs 19:09, 22 December 2008 (UTC)[reply]
I read the OP's question/missive/whatever differently. The way I read it was "Why is it that so few physics teachers know even the most basic of material?" I got a sense of frustration from the OP, not curiosity. Dismas|(talk) 01:26, 23 December 2008 (UTC)[reply]
It's a basic problem in the educational system. The ideal physics teacher would have a degree in physics and a degree in teaching. But someone with two decent degrees can get a job doing some sort of technological thing for about twice what a teacher earns. Hence quite a few high school physics teachers don't have a degree in the subject and learned what they needed to learn in order to teach the curriculum but not much more. There are exceptions - people who do it for the love of it despite the poor pay - but from what I've seen as a parent, they are not in the majority. Personally - I'd love to be a teacher - but I can't afford it and (rightly or wrongly) I believe that the constraints of having to teach to the curriculum and deal with standardized testing would drive me nuts. My kid is now in college - he went to a 'magnet' school (The School of Science and Engineering in Dallas, Texas) which has been in the top ten rated high schools in the whole of the USA (public or private) and which is renowned for it's strength in teaching science and technology - and STILL the science teaching was (mostly) terrible. It's unlikely that was due to them being able to get good people because what dedicated science teacher wouldn't want to work for one of the best high schools in the country? So the problem must be with the curriculum or the 'teach the test' approach that's being forced down the throats of competent teachers. But that excuse doesn't cover the OP's complaint - which (I'm sure) is due to underqualified teachers - which in turn has got to be due to pay and working practices causing well-qualified people to be turned away. SteveBaker (talk) 13:17, 23 December 2008 (UTC)[reply]
I would disagree with that assessment. I have a Bachelor's of Science degree in Chemistry from a top-10 ACS ranked chemistry school (University of Delaware) and a Master's of Education degree, and I am a teacher, and could not imagine doing anything else, even for some more money. Nearly every science teacher I have worked with has had a degree, or equivalent, in the subject area they taught in. In the 6 years I was an active classroom teacher (I am on somewhat of a sabatical right now), every physics teacher I worked alongside had a degree in either Physics or Physics education. The greater problem is that there is a somewhat unrealistic expectation that everyone needs to know everything. Many students are placed in classes which they have no use or aptitude for, and yet we have to teach all of them. If, as a high school chemistry teacher, I didn't get to teach my students how to work with the Schroedinger equation or we never got to work with the Wittig reaction it wasn't because I was a bad teacher; there is a limited number of days to teach the curriculum, and by necessity there is some material we just can't cover in 135 contact hours. So when a Physics teacher doesn't take an hour out of his class to explain all of quantum mechanics to answer one student's question about electromagnetism, its not because he couldn't answer the question. Its because he's got all 135 hours filled with OTHER material that he has to teach first. You want science to be a more effective for these students? Introduce more levels into classes, and get students into the correct level, so that students who would benefit from it can move at a faster pace, and students who don't have the aptitude can move at a slower pace and work on the most important stuff only, so we can be sure they really get it. --Jayron32.talk.contribs 14:01, 23 December 2008 (UTC)[reply]

What does she want?

OK, I've got a question. I've been out with this girl 3 times. The first date was dinner. We hit it off pretty good, kept talking long after dinner was finished. At the end of the date, she gave me a hug. She e-mailed me the next day to say thanks. The next date was on my hour long lunch break. At the end of the date, she gave me a hug. Usually, I'm used to a kiss on the first or second date. Thought it was strange, but whatever. Yesterday was our third date. I took her to see the symphony. At the end of the date, I was expecting at least a kiss. Anyway, it was just a hug. I think she's into me but I can't tell if it is as a friend or more. She's going through a divorce which isn't finalized, so maybe she just wants to take things slowly. She has said things about wanting to see me in the future. For example, on the second date, she said she wanted to watch me play softball. On the third date - well, this takes a bit of explaining. I'm a musician who's recorded 3 demo albums, plus a Christmas album which I won't let her listen to because it's pretty bad (it was recording in a week and I used my family as singers) and she said that she wanted to hear it. I said no. And then she said she would hear it eventually, even if it's not until spring. Anyway, the thing that really got me was as she was leaving, she said that the holidays were coming up (which I understand), so "keep in touch". What the fuck does that mean? "Keep in touch"? I have no freaking clue what she wants. 67.184.14.87 (talk) 12:45, 22 December 2008 (UTC)[reply]

EDIT: She has three children. I'm 37, she's 33. (I'm the OP on a different computer.) 216.239.234.196 (talk) 13:41, 22 December 2008 (UTC)[reply]

She wants security and children. --Milkbreath (talk) 12:57, 22 December 2008 (UTC)[reply]
EDIT: She wants to inspire love. --Milkbreath (talk) 16:53, 22 December 2008 (UTC)[reply]
Personally, I'd interpret that as 'phone me over Christmas and have a chat about whatever people talk about on phones once they've got past the "how you doin'?" and "what you been up to?" stage of the conversation'. --Kurt Shaped Box (talk) 13:01, 22 December 2008 (UTC)[reply]
I guess it's also possible that the divorce situation means she has to be extra careful about not showing herself up as being the cause of the divorce - or risk losing her kids if her ex- makes nasty accusations about her going out partying all the time. Perhaps she's wondering why you don't kiss her? A peck on the cheek ought to be OK during one of those goodbye hugs. She's got to be under all kinds of stress right now. I think she's taking it slow for all kinds of very obvious reasons. I would try to help her out with that - keep in contact - phone often enough to keep things rolling but not so often as to appear "needy" (the last thing she needs is "needy"!) - invite her on low-pressure dates (lunch is good) - make it a regular thing (so she has it to look forward too when times are tough). And for chrissakes - she's 100% right about the Xmas album. On your very next date - put a copy of that crappy Xmas album inside a $20 portable CD player (don't forget the batteries!), gift-wrap the whole thing with a big bow on the top - give it to her and make her listen to it there and then! Tell her that she was right and that now she owes you the low-down on her worst screwup. If you can't laugh over your screwups with her - it's already over. SteveBaker (talk) 14:29, 22 December 2008 (UTC)[reply]
Have you asked her what she wants? --Moni3 (talk) 14:38, 22 December 2008 (UTC)[reply]
That's always a good idea. The phrase "So how does that make YOU feel?" is one of the most useful in the English language when making conversation with women. While she tells you, you can go back to wondering how much better she'd look if that top button were to inadvertently come undone. SteveBaker (talk) 15:24, 22 December 2008 (UTC)[reply]
I know that was a joke but I should point out actually listening is sometimes more important then asking Nil Einne (talk) 17:22, 22 December 2008 (UTC)[reply]
Really?! Oh - crap. SteveBaker (talk) 22:02, 22 December 2008 (UTC)[reply]
Not wanting to be a dick about it, but external events do not "make" people feel things. We all retain control over our feelings, no matter how compelling and automatic our knee-jerk reactions may seem. Take 9/11: the emotional responses ranged from fear and panic, through to joy and celebration. Some were amazed such a thing could ever have happened. Others were amazed it hadn't happened years earlier. Others were completely indifferent to it. I'm not saying don't ask her about her feelings, but rather than "How does that make you feel?", a better question might be "How do you feel about that?". -- JackofOz (talk) 22:12, 29 December 2008 (UTC)[reply]

Knowing ladies well,and have had the same done to me,they are complicated,change their minds in a blink of an eye,keep in touch means just that,then you also might never hear from her again,it also sounds as if she wants time,and would probably like to keep you just as a friend for now,to see you when she wants.If you want more,you might want to seek another lady or just contiunually waste your time and money on dates with a woman that won"t even give you a kiss good night. Fluter. —Preceding unsigned comment added by 170.86.15.15 (talk) 15:45, 22 December 2008 (UTC)[reply]

After your next date, go to kiss her first. But be prepared for any eventuality from a longer kiss and more, to a slap and a "I never want to see you again". Crude, but at least you'll know where you stand. —Preceding unsigned comment added by 87.114.155.166 (talk) 18:59, 22 December 2008 (UTC)[reply]

Thanks for everyone's response. I think at this point I'm inclined to believe that she wants to take things slowly because she's still going through the divorce. 216.239.234.196 (talk) 19:25, 22 December 2008 (UTC)[reply]

My experience: if you're looking for anything beyond a casual physical relationship, a 2 or 3 year post-divorce No Fly Zone will serve you well. --Sean 20:43, 22 December 2008 (UTC)[reply]

Non-Profit and Not For Profit

I was under the impression that any organization/person could establish a Non-Profit Company, but only the government could establish a Not-For-Profit company.

However, I have been told that non-profit and not-for-profit are synonymous and have no difference.

What is the difference between Non-Profit and Not-For Profit?

130.221.224.7 (talk) 19:27, 22 December 2008 (UTC)December22, 2008[reply]

It may differ from country to country. I used to work for a not-for-profit organisation, but their survival (and my bonus) actually depended on them making handsome profits. Not-for-profit in Australia means a company whose primary goal (officially, at least) is something other than profit, but they're not precluded from making profits. Non-profit means any profits they do make, after taking care of admin expenses, planned expansion etc etc, have to be disposed of and can't be retained for investment etc - not sure how this is done. -- JackofOz (talk) 19:42, 22 December 2008 (UTC)[reply]
According to non-profit organization, the terms are synonymous. --—— Gadget850 (Ed) talk - 02:45, 23 December 2008 (UTC)[reply]
In the UK the two terms mean the same thing. The main distinction is between "not for profit" and "profit" in the description of the way business is carried out. A "not for profit" business in the UK is a slight misnomer, as JackofOz pointed out these organisations still have to demonstrate a surplus in their transactions, and they have to demonstrate solvency. What the term actually should be is "non-profit distributing" as they are not able to distribute their profits to third parties such as shareholders. Profits should be retained as reserves after paying staff, taxes and administration costs, and if the organisation is a charity in England and Wales, the Charity Commission has guidelines on the amount of reserves which should be maintained. --TammyMoet (talk) 09:40, 23 December 2008 (UTC)[reply]

Choosing your USAF aircraft

Hello. Is it possible that: when you enlist in the Air Force, that you can choose (or at least request) to fly an airplane of your choice?--DocDeel516 discuss 19:43, 22 December 2008 (UTC)[reply]

Recruiters are big on allowing people to request things, and will even promise things orally, but, if you read what you actually sign, they can do whatever they want with you, and probably will. StuRat (talk) 21:26, 22 December 2008 (UTC)[reply]
It should also be VERY clearly stated that only a very tiny percentage of people who join the USAF will ever get to fly a plane. To put some numbers on this (data taken from our USAF article: The USAF has 328,000 personnel on active duty (plus a bunch of reserves and such like) - and it has 5,800 aircraft - of which 1,700 are actually air national guard, reserve units, etc. So - bottom line is that there are a third of a million guys out there who all joined the USAF with the dream of flying a plane (why else?) and about 4,000 planes for them to fly. Of those - many are big boring transport planes - and of the rest, aircraft are being replaced with pilotless drones and transports flown by civilians as fast as they can make the transition. So your odds of being able to fly a plane AT ALL is only about one in 75 - and the odds are getting worse - not better. Getting to fly the precise kind of plane (fighter, helicopter, ground attack, transport) has got to be lower than that - and your chances of getting to fly a particular kind of plane is very low indeed. Please don't take the word of a recruiter. If they won't give it to you in writing - assume it's not true. SteveBaker (talk) 21:58, 22 December 2008 (UTC)[reply]
How about going through flight school first, THAN joining up? Even better, go to college (if you haven’t already) and flight school first. As a fully qualified pilot already you would have a much greater chance of flying than all the thousands of kids than join right out of high school and end up polishing bird poop off the windscreens. Contrary to popular belief, the US armed forces are very interested in hiring well educated slightly older candidates, and because such recruits are in short supply they often end up in higher ranking, better paid positions. --S.dedalus (talk) 23:09, 22 December 2008 (UTC)[reply]

I may be wrong, but I believe all pilots in the USAF, perhaps with the exception of some helicopter pilot, must be officers and hence, hold at least a bachelor's degree. Acceptable (talk) 23:42, 22 December 2008 (UTC)[reply]

SteveBaker's calculations depend on each plane only having one pilot so no co-pilots or rotating crews (I don't know if that is completely true). But also I would expect pilots to stay in the service longer than some more "lowly" occupations so that the number of grounded serviceman to pilots to be much higher. Any government statistics out there? Rmhermen (talk) 00:11, 24 December 2008 (UTC)[reply]
Some aircraft types would rotate pilots - and transports planes often have two pilots - but mostly not. If you're a fighter pilot they even paint your name on the plane. SteveBaker (talk) 02:33, 24 December 2008 (UTC)[reply]

Attaining US Residency as a student

I am going to the USA for university and am currently a Canadian citizen. First, do I need some sort of student Visa? Can I qualify for US permanent residency as a student? If so, how many years do i have to study in the US for? Hustle (talk) 22:16, 22 December 2008 (UTC)[reply]

I am also a Canadian and went to a US school. You need an I-20, which as I understand it is not an actual visa but for all practical purposes the equivalent of one. I believe I got all the information I needed from the school, or else they directed me to the American consulate. It wasn't a particularly big deal, especially compared to the visas required for some of my classmates (from India, Iran etc). The I-20 requires the school registrar to confirm you're enrolled and that you can support your studies financially, necessary in part because the I-20 does not entitle you to work off campus. You have to get the I-20 re-signed each year by the registrar people, if you try to go through and it has expired they can hold you up at the airport while they check to see if you're really enrolled (I made this mistake at the beginning of my second year there, luckily I got to the airport early. Another lesson learned: keep your SEVIS receipt). I believe you have a 3 or 6 month grace period after your I-20 expires before you are officially persona non-grata and quite deportable. I also think it's likely that studying in the states makes it easier to attain permanent residency, but I didn't apply for that so I wouldn't know fo rsure. I suspect having an employer lined up after your graduation is the critical part of that process. TastyCakes (talk) 22:44, 22 December 2008 (UTC)[reply]
As to the second part of the question: if a green card is your goal, look for a qualifying job; 'student' isn't it. —Tamfang (talk) 19:45, 28 December 2008 (UTC)[reply]

Some Variable terrestrial measurements

Could someone provide me with some variable terrestrial measurements, ex: atmospheric pressure, distance from Sun, temperature, wind speed, humidity, ceiling, etc... The stranger the better. Thanks! Acceptable (talk) 23:56, 22 December 2008 (UTC)[reply]

Albedo, surface magnetic field strength, Environmental radioactivity, Biomass distribution, habitat quality, rheological behavior, crust thickness organic content of soil just for starters. 76.97.245.5 (talk) 01:14, 23 December 2008 (UTC)[reply]
Magnetic declination is a favorite of mine: the variation of the compass needle from true north. It varies by location as well as over time at the same location. Old surveys were done by magnetic compass, and ewhen the declination varies several degrees in several decades, a new magnetic compass survey may put your house on the neighbour's farm. Edison (talk) 02:33, 23 December 2008 (UTC)[reply]
Global warming of course. -hydnjo talk 04:03, 23 December 2008 (UTC)[reply]
Duration of a day (compare UT1 and UTC). Daylight hours (by season). -- SGBailey (talk) 09:23, 23 December 2008 (UTC)[reply]
Even the strength of Earth's gravity changes depending on where you are on the planet. Raven4x4x (talk) 13:24, 23 December 2008 (UTC)[reply]
Yeah - forget diets - just weigh yourself at the top or Mount Everest and you'll be 0.3% lighter! SteveBaker (talk) 16:27, 23 December 2008 (UTC)[reply]
Yes, but taking all your clothes off first to get a true reading could be painful. StuRat (talk)
Also see Earth for some astronomical data on our planet. StuRat (talk) 16:56, 23 December 2008 (UTC)[reply]

December 23

snorkel monster

Once I was listining to the radio, and they said someting about a one footed snorkel monster. What's all this about?Warriorscourge (talk) 04:38, 23 December 2008 (UTC)[reply]

Without some context such as what radio program you were listening to or where the station is or something like that, all we could do is give a rough guess. This is the first Ghit for "one footed snorkel monster". Dismas|(talk) 04:44, 23 December 2008 (UTC)[reply]
Sounds like a relative of the Purple People Eater. Edison (talk) 04:56, 23 December 2008 (UTC)[reply]
It could be a "one-foot" (length) "snorkel" (long tube) "monster"... kinda like "Willy the One-Eyed Wonder Worm"... aka, its a slang term for a penis... --Jayron32.talk.contribs 13:48, 23 December 2008 (UTC)[reply]

Okay, well, the radio thing was KXNT, and they said it was being attacked by a UFO. I now, sounds pretty made up.-Warriorscourge (talk) 00:09, 25 December 2008 (UTC)[reply]

Town layouts

Why are US towns & cities laid out in blocks? A simplistic answer of "it makes things easy/regular" is all very well, but what about nature / slopes / rivers? Who says regular is good anyway? No British town has regular blocks and even Milton Keynes where the roads are gridded have bending windy (as in wind a clock not blowing air) roads within the gridsquares. There is an obvious difference in viewpoint between the two countries. I was wondering what the rationale for the difference might be. -- SGBailey (talk) 09:33, 23 December 2008 (UTC)[reply]

A look at Grid plan suggest that this design/city-planning idea has existed for a very long time. I suspect that sometimes nature dictates how a city can be designed, but sometimes the city can be 'planned' in a simple grid format quite easily. There is a section on 'benefits' Grid_plan#Benefits_and_criticisms. It would probably be worthwhile also reading Street hierarchy. I suspect a large reason is because the expansion of British cities wasn't 'planned' or at least not designed in the same way as in the US - whose cities are (mostly) much younger than in the UK. Additionally there'll be 'cultural' economics at play. In the UK a grid-network might not be associated with wealth/high-value and so the money that can be made selling houses on a grid-network estate is perhaps less than the meandering/bendy setup we are used to here in Britain. 194.221.133.226 (talk) 10:40, 23 December 2008 (UTC)[reply]

While not all U.S. towns and cities laid out in rectangular blocks, two factors at work are:
My tongue-in-cheek explanation is that George Washington, a former surveyor who grew up in Virginia where it's apparently illegal for roads to meet at right angles, had a hand in this. You can see the effects clearly across the Midwest. Given the regular pattern for larger units like townships (six miles square)and sections (one mile square), it was easy to organize settlements along existing property boundaries. For example, Eight Mile Road, the northern boundary of Detroit, is also known as Base Line Road because it serves as a reference point for land surveys throughout Michigan's lower peninsula. But even Eight Mile varies from its "correct" path because of a stream.
I'd add that many suburbs developed in the past 30 years or so feature curving streets, cul-de-sacs, and other non-grid arrangements. As a developer in the northern Virginia suburbs of Washington DC pointed out, in residential neighborhoods strict grids don't offer all that pleasing a view to the eye; curving streets offer greater interest and help modify the visual impact of roadways. --- OtherDave (talk) 12:19, 23 December 2008 (UTC)[reply]
In Victorian times in the UK, 'grid iron' layouts were popular - hence the rows of 'Coronation Street' style Victorian terraced housing in many towns and cities. Glasgow city centre is also built on a grid plan. 62.25.96.244 (talk) 12:33, 23 December 2008 (UTC)[reply]
The hey-day of the "grid-plan" in the U.S. was probably 1750-1900. Cities which came into their own in during that time period (Chicago, New York City, Philadelphia) display quite regular grid patterns. Cities which are older (notably Boston) than that have some rather, um, "un-gridy" streets, as anyone who has tried to navigate Boston can attest. However, parts of Boston that WERE built in that time period (like Back Bay), DO show a grid layout. Grids work best for horse-and-buggy economy; with the advent of the car in the 20th cenrutry, Grids fell out of favor for the street hierarchy system and for zoning, which keeps residential, commercial, and industrial parts of a city seperate. This is all friendlier to cars by better controlling traffic flow onto a smaller number of "major" roads, and keeping high-speed traffic out of residential areas. Cities which saw their development in the 20th century, like Atlanta, Houston, and my current city, Raleigh, North Carolina ascribe to the urban sprawl method of development, and not the old "grid plan". --Jayron32.talk.contribs 13:45, 23 December 2008 (UTC)[reply]
Except that New York City is six years older than Boston... Rmhermen (talk) 14:40, 23 December 2008 (UTC)[reply]
Absolute age of the city means little. The New Amsterdam of the Dutch has little remnant at all in the modern city of New York. New Amsterdam of the Dutch time period was laid out according to the Castello Plan, and aside from about half a dozen streets south of Wall Street (which at the time was the northern limit of New Amsterdam). So you could say that, for a little tiny plot of Manhattan, around the battery, there is no grid plan. However, the vast majority of the city was laid out much later, and most of the expansion of New York happened much later. Most of modern New York was laid out in a series of plans known as the Commissioners' plans. Everything north of 14th street was plotted out at that time, and most of the streets south of 14th street had been laid out in . Also see File:Grid 1811.jpg for a map of the New York city plan as it existed in the 1807 Commissioners' plan, an early draft of the final version that was passed in 1811, and which (except for Central Park) is still in place today. This article: History of New York City transportation is a helpful read as well.
The city of Boston never had a unified grid street plan, and it was mostly laid out in the 1630s and 1640s. According to our article on Boston transportation, most of the streets were laid out to follow the shorelines of the Shawmut Peninsula upon which Boston was first built. The local geography which was mostly hills, swamps, and brackish estuaries, which left little room for a real grid pattern. The "gridded" parts of Boston date to around the same time as the "gridded" parts of Manhattan; its just that by that time, there was far greater part of the Boston area settled and covered with streets already than there was in Manhattan. --Jayron32.talk.contribs 17:31, 23 December 2008 (UTC)[reply]

I would think the most obvious factor is that British city centers date to the Middle Ages, before they had the kind of surveying equipment and urban planning they had in the 19th century. I read once that New York considered straying from the rectangular grid system in the 19th century but stuck with it because it was easier to sell real estate in squares rather than oblong shapes. Note that nowadays, straight lines are very rare in North American residential developments, since curvy streets apparently give people the sense of exclusivity and higher property values. -- Mwalcoff (talk) 23:56, 23 December 2008 (UTC)[reply]

Yep - according to Bernard Cornwell (an author of fiction, but an impeccably-researched one and in this instance writing a factual article) the layout and even many of the property boundaries in Winchester date from King Alfred's time - well over a thousand years ago. PeteVerdon (talk) 01:48, 24 December 2008 (UTC)[reply]
Yes, but why is it felt to be a good idea? Grid plan suggests it reduces boundary arguuments and minimises road lengths. Modern British developments appear to be deliberatly curvy and non-griddy. -- SGBailey (talk) 00:06, 24 December 2008 (UTC)[reply]

The Manhattan grid has three advantages.

  1. As each block is the same size, the buying, selling, and trading of real estate was greatly simplified in the era of the city's expansion northward.
  2. As the plan was uncomplicated and straightforward, one could plan for future construction years before the work actually began. You could build with confidence that the rest of the city would expand to meet you. For example, the Dakota apartment building on Central Park West was built in lonely splendor a mile from where most construction was occuring, and thus, at less expense. In a few years, the plots around it filled in.
  3. Thw city was, and is, host to many immigrants for whom English is not their native language. By laying out the streets in a regular grid, and numbering them, the city became instantly accessible to many who would, otherwise, have been completely lost.

B00P (talk) 04:04, 24 December 2008 (UTC)[reply]

Modern British developments are definitely, as you say, non-gridded in almost all cases. A deviant of the road hierarchy system is in common use - Motorways for intercity traffic, dual carriageway A roads linking the former to the city centre (usually a ring road), single carriageway A roads radiating through the various districts, unclassified through roads serving individual estates and then cul de sacs off those roads that contain houses or industrial units. If you're building a town/city from scrach then a grid plan may be a good idea (ie Milton Keynes) but in existing urban areas which are not gridded then any developments have to fit in with whatever is around them.

Unlike US cities, UK towns/cities and even villages tend to have one of several different designs -

- Radial - All the main roads go out from a central point at whatever angle heads towards the road's destination (a lot of that is thanks to the Romans)

- Linear - The settlement has developed alongside a river, canal, railway or road and so tends to be longer than it is wide. 62.249.220.179 (talk) 00:23, 24 December 2008 (UTC)[reply]

Actually, there are lots of American cities, expecially in mountain areas, which follow that linear pattern. C.f. Wilkes-Barre, Pennsylvania and Charleston, West Virginia; where the cities are indeed quite long and narrow, as you note, following a river course. And the radial pattern you note, well that's essentially how urban sprawl works, see Atlanta, Georgia. --Jayron32.talk.contribs 00:54, 24 December 2008 (UTC)[reply]
I live in Boston (and love its non-griddiness), but as Jayron suggests, Boston is not the only U.S. city that is free of the oppressive order and monotonous sightlines of the grid. Citywide grids are really the exception rather than the rule east of the Mississippi and south of the Great Lakes drainage (except for Florida). Even in New York City, it is really only Midtown and Upper Manhattan that have a unified grid. Other parts of the city, such as most of Brooklyn, Queens, and the Bronx, have a more organic street pattern. Maybe it is just that the parts of the United States most likely to be visited by foreign tourists tend to have citywide grids. Marco polo (talk) 01:11, 24 December 2008 (UTC)[reply]
Brooklyn is more griddy than not, though there are several grids, not one throughout. (I used to live in Yonkers. I don't think there was a straight street in the entire city.) --Nricardo (talk) 01:54, 24 December 2008 (UTC)[reply]
Use of the Theodolite for surveying probably facilitated grid designs. Property lines defined by natural elements like rivers, hills etc. tend to be rather unreliable. While today's advanced surveying methods make same size lots with curvy property lines possible, in former days straight lines on a map, transferred to the actual plot, would have been easiest to get each buyer the exact acerage they paid for. The disputed border between Georgia and Tennessee shows that such transfer was not always successful. (Rumor has it that the surveyors didn't fancy meeting the local native American population and surveyed a bit farther south.) As for Atlanta: The roads were designed along former cow-paths; and the cows were drunk! ;-) 76.97.245.5 (talk) 04:11, 24 December 2008 (UTC)[reply]
Yes, I think that the reason for grid streets and parcels in the US circa 18-19th century had less to do with it being easier to sell the parcels and more to do with keeping surveying costs low. In those days there were vast amounts of land in America in the hands of people wishing to subdivide and sell, and not nearly enough skilled surveyers to fill the demand. Thus surveying work was costly. Grids are easier and quicker for even novice surveyors to plat out. There's more to the history of US street grids than just that, but the cost and time savings were likely a major factor. Pfly (talk) 10:19, 25 December 2008 (UTC)[reply]

As a response to the people talking about its origins, the Roman empire perfected the grid layout (Greece having fiddled with it previously) and built grid cities throughout the empire to demonstrate their opulence/intellect/awesomeness. Many of the cities were perfect squares, but the larger ones tending to distort with the topography. 98.122.161.31 (talk) 19:44, 27 December 2008 (UTC)[reply]

Why is cracking the knuckles/neck associated with toughness?

I do this all the time and I'm not particularly tough or manly! So what's the reason? 99.245.92.47 (talk) 13:41, 23 December 2008 (UTC)[reply]

Because tough muscular people find it a convenient way of drawing attention to their physique? By the way I have a colleague who is super skinny and he cracks his neck all the time. ReluctantPhilosopher (talk) 14:41, 23 December 2008 (UTC)[reply]
Because it says: "I'm so tough that I don't expect to live long enough for the tendon damage I'm causing to myself to become a problem." SteveBaker (talk) 16:24, 23 December 2008 (UTC)[reply]
I think the biggest reason of all is that cracking nuckles is intimately tied in to the image of a man getting in to a fight. You know, imagine a big, tough guy, getting ready to fight someone. He jumps around and hits some air to warm up his muscles, he cracks his knuckles and puts up his dukes. That sequence has been in like a gazillion movies, thus forever associating that action with masculinity. Belisarius (talk) 23:00, 23 December 2008 (UTC)[reply]

You know, I do that all the time before fighting in Taekwondo, so the last guy was right.-Warriorscourge (talk) 04:57, 27 December 2008 (UTC)[reply]

Or preparing to play concert piano? Julia Rossi (talk) 03:04, 24 December 2008 (UTC)[reply]
Watch it, bub. I find playing concert piano to be very manly. Perhaps that's because I do play the piano... Or the fact that the name for a piano player is pianist... flaminglawyerc 04:49, 24 December 2008 (UTC)[reply]
No-one doubts the masculinity involved in fingering away at a 88-key bench, but why a caricature that pianists first crack their knuckles? ; ) Julia Rossi (talk) 05:09, 24 December 2008 (UTC)[reply]
Perhaps it's because someone who doesn't crack their knuckles or neck whould assume that it's painful, and therefore assume that anyone voluntarily inflicting pain upon themselves is a hardass.--AtTheAbyss (talk) 05:20, 24 December 2008 (UTC)[reply]
I've been a pianophile for as long as I can remember, Julia, and I can honestly say I've never heard of that stereotype. But pianists often warm their fingers up before a concert. Most often they do this by playing some exercises on a piano away from the stage. Glenn Gould used to soak his hands for some minutes in the hottest water he could tolerate. It worked for him. But then he died of a heart attack at age 50, so you never know .... -- JackofOz (talk) 21:56, 29 December 2008 (UTC)[reply]

My man takes too long to orgasm.

This question has been removed. Per the reference desk guidelines, the reference desk is not an appropriate place to request medical, legal or other professional advice, including any kind of medical diagnosis, prognosis, or treatment recommendations. For such advice, please see a qualified professional. If you don't believe this is such a request, please explain what you meant to ask, either here or on the Reference Desk's talk page.
This question has been removed. Per the reference desk guidelines, the reference desk is not an appropriate place to request medical, legal or other professional advice, including any kind of medical diagnosis or prognosis, or treatment recommendations. For such advice, please see a qualified professional. If you don't believe this is such a request, please explain what you meant to ask, either here or on the Reference Desk's talk page. --~~~~

You should encourage your partner to speak to his physician or other expert about any concerns he might have; you might also ask if you can come along to participate in at least part of the discussion. TenOfAllTrades(talk) 17:11, 23 December 2008 (UTC)[reply]

Sorry I was did not mean to ask for doctor's advice, just some ideas and links to things to read. Thanks. —Preceding unsigned comment added by 194.80.240.66 (talk) 17:30, 23 December 2008 (UTC)[reply]
You should perhaps, then, start with Human sexuality and follow some links from there. The only advice I can give is rather general, and to note that "different" is not always "wrong", and just because one person has a different method of participating in sexual activity does not always mean that such methods are "wrong". However, as noted, if you have a concern, please talk to your partner, and then consider contacting a professional who specializes in these issues. If there is one "wrong" thing, it is going to the internet to seek advice from random strangers!!! --Jayron32.talk.contribs 17:37, 23 December 2008 (UTC)[reply]

I guess that "This question has been removed." doesn't apply to answers ;) -hydnjo talk 02:45, 24 December 2008 (UTC)[reply]

Kippah keptah on the headah

How the heck do Jews keep them from flying off their noggins? Clarityfiend (talk) 21:27, 23 December 2008 (UTC)[reply]

The heads up in Comb says they're held on with combs and google says combs or clips. Julia Rossi (talk) 03:01, 24 December 2008 (UTC)[reply]
But what if you're bald? BrainyBabe (talk) 14:22, 24 December 2008 (UTC)[reply]
Spirit gum works. Staples, too. --jpgordon∇∆∇∆ 16:52, 24 December 2008 (UTC)[reply]
You know what's really sad, is that I bet the standard belief in the Middle Ages was that the kippah/yarmukle existed to cover up those devil's horns. And there are bound to be some who believe it today. We mock ignorance at our peril. BrainyBabe (talk) 09:52, 25 December 2008 (UTC)[reply]
That's a good question actually...there were laws decreeing that Jews must dress differently from Christians but I don't remember ever reading about yarmulkes. They are often depicted wearing a judenhut though, maybe that is the same idea. Adam Bishop (talk) 14:11, 25 December 2008 (UTC)[reply]


  • Ahem* The short answer is: a bobby pin, or two bobby pins.

The longer answer is: Conservative Jews don't wear them full-time (usually) and hence don't have to worry about them flying off during a basketball game. Synagogue, and other religious rituals which require a kippah aren't usually particularly hectic or head-shaking.

To be perfectly frank, go to a religious jewish wedding, watch the men dance for about 10 minutes, and you'll see kippot soaring in all directions.151.203.23.82 (talk) 00:02, 26 December 2008 (UTC)[reply]

If it's the right size, it wont fly off in a light breeze. If the breeze is heavier, you'll still be OK if you slightly tilt your head into the breeze. In a windstorm, you either hold it down with a finger, or wear a hat. Also, Scunci makes clips that work very well.
You may also find some with a button on top that conceals a thumb tack. Furthermore, my wife tells me that if I lose any more hair she will get me a velcro implant. Phil_burnstein (talk) 23:26, 31 December 2008 (UTC)[reply]

Printing custom postage stamps

Is there a company that will print custom-designed, gummed postage stamps? I don't want internet generated US postage, I don't even want "real postage", I just want to design my own postage stamps and have them printed. Whenever I do online searches I get rubber stamps, real postage to print and use, etc, That's not what I want. I know that the USPS prints it's own postage and doubt they would print my custom stamps. Any ideas? Thanks! —Preceding unsigned comment added by 64.234.6.82 (talk) 21:58, 23 December 2008 (UTC)[reply]

When I was in Australia I found a company that made up some proper postage stamps that had a photo of me on them. It was a gimmick for tourists, like 'send a postcard home with your own holiday snap on it'. But I'm afraid that's all the info I can give.91.111.99.97 (talk) 22:37, 23 December 2008 (UTC)[reply]
I'm betting that now that you can get actual postgae with a picture of your stupid cat on it, you're flirting with a charge of counterfeiting to make something that looks like a stamp. I googled my ass off just now and got no farther than you did. --Milkbreath (talk) 22:40, 23 December 2008 (UTC)[reply]
These[8] people claim to. Oh, I see, you want ones that aren't actually for posting letters, just for putting them on hand-delivered envelopes, etc. Steewi (talk) 00:24, 24 December 2008 (UTC)[reply]
[9]This is what 91.111 was talking about. You get the normal stamp, but on an adjoining stamp (with a perforation between them) you get your custom stamp (usually a photo of a person, but it doesn't seem to be restricted to that). Steewi (talk) 00:27, 24 December 2008 (UTC)[reply]
Couldn't you do it yourself with a color printer and some special paper with either lickable glue or pull off tape on one side ? I bet hobby stores sell this. Cutting them into stamp sized portions would require a paper cutter or just some scissors, unless you want fancy edges. You'd need a special device to cut those. StuRat (talk) 01:31, 24 December 2008 (UTC)[reply]
I have a guillotine attachment that'll put fake perforation tearing down the edge of a sheet of paper. [10] - that would let you print the stamps out yourself. There is still the matter of adhesive. SteveBaker (talk) 02:28, 24 December 2008 (UTC)[reply]
As for adhesive, see this. Oda Mari (talk) 15:33, 24 December 2008 (UTC)[reply]
The term you need to Google for is Cinderellas. 94.196.76.55 (talk) 19:42, 27 December 2008 (UTC)[reply]

Scandalous politicians

I was reading Todd Alcott's analysis of The Dark Knight and he compares what would happen to the people of Gotham's morale of finding out Harvey Dent is Two-Face to people's reactions to a "gay-bashing senator elicit[ing] sex in airport mens' rooms, or your law-and-order governor gets caught soliciting prostitutes". Who are these politicians? Alientraveller (talk) 22:51, 23 December 2008 (UTC)[reply]

Larry Craig and Eliot Spitzer Belisarius (talk) 22:55, 23 December 2008 (UTC)[reply]
"Belisarius" isn't part of "Eliot Spitzer," just in case there's any confusion... · AndonicO Engage. 04:38, 24 December 2008 (UTC)[reply]

December 24

Moxie's

Moxie's has 3 "front" positions: Server, Expeditor and which other one?96.53.149.117 (talk) 01:39, 24 December 2008 (UTC)[reply]

Money collector? -hydnjo talk 02:40, 24 December 2008 (UTC)[reply]

Conan's last show

According to nbc.com, you're allowed a maximum of two tickets per request. Do you have to ask for those two tickets in your email or is it a default amount when they select your request? --Crackthewhip775 (talk) 06:24, 24 December 2008 (UTC)[reply]

Why take the chance ? I'd ask explicitly. StuRat (talk) 08:32, 24 December 2008 (UTC)[reply]

julius caesar

bring out the differences between portia and caliphurnia —Preceding unsigned comment added by 117.197.50.206 (talk) 15:40, 24 December 2008 (UTC)[reply]

Portia ? Caliphurnia ? The similarity is that neither did their homework. The difference is that neither ever lived in Caliphornia.
Portia has about 20 lines, Calpurnia maybe twice as many. This [11] and this [12] site gives you nothing but their lines / clues.--Cookatoo.ergo.ZooM (talk) 16:15, 24 December 2008 (UTC)[reply]
Although when you do do your homework about Julius, you should divide it into sections. You're sure to get a C. --- OtherDave (talk) 16:36, 24 December 2008 (UTC)[reply]
You might also get a better grade if you spelled the names of the characters correctly. Little Red Riding Hoodtalk 20:58, 24 December 2008 (UTC)[reply]
Sections? You should quarter it into three halves! Also, Portia spent more time in drag. --Maltelauridsbrigge (talk) 18:08, 26 December 2008 (UTC)[reply]

Theoretical Physicist

Hello. Since I want to become a Theoretical Physics doctorate, I better ask this question now: Where might a theoretical physicist be able to get a job, besides the universities? Merry Christmas to everyone at Wikipedia!--DocDeel516 discuss 20:05, 24 December 2008 (UTC)[reply]

At a science magazine (Throw in a couple of writing or journalism classes.) United States Department of Defense if you are in the US. Any government agency dealing with statistics and prognosis. (Quantum theory is being applied to economics sometimes.[13]) Theoretical physics concerns many areas of physics many of those have applications "in the real world" so it would depend on your specific background what companies or agencies would have use for your knowledge. If all else fails you can always become a contractor, write a book or become a motivational speaker. A lot depends on your personality, ability to network and how well you can sell yourself. o<:-)76.97.245.5 (talk) 23:27, 24 December 2008 (UTC)[reply]
Mostly what a degree proves is that you are a smart person who can learn - and a post-graduate degree shows that you can think independently and produce original work. This opens the doors to many jobs that are totally unrelated to Physics. I heard a few years ago that stock trading companies were seeking physicists and software engineers because they were better at the kinds of thinking needed than people with more traditional degrees in economics or business. But if your heart is set on actually working as a theoretical physicist as well as studying for the qualification - then you're almost certainly going to wind up in academia. Almost everyone else wants some kind of result that either makes money or kills people...and that's more Applied Physics than Theoretical Physics. SteveBaker (talk) 01:58, 25 December 2008 (UTC)[reply]
There are likely to be jobs in the energy industry, broadly defined, including solar and wind power and "smart" electric grids. Edison (talk) 05:13, 25 December 2008 (UTC)[reply]

It used to be the case that a sure-fire job for theoretical physics could be found on Wall Street (they looovve number crunchers with unusual backgrounds) though I doubt that is the case at the moment. --98.217.8.46 (talk) 21:15, 25 December 2008 (UTC)[reply]

Defensive Driving

Hello. I often see papers that urge you to "Drive defensively". What does this mean? (I am asking because I don't have a driver's license!)--DocDeel516 discuss 20:08, 24 December 2008 (UTC)[reply]

As in most cases, we have an article. It's not a great article, but the gist is "anticipating dangerous situations, despite adverse conditions or the mistakes of others." --LarryMac | Talk 20:14, 24 December 2008 (UTC)[reply]
I would add to that definition that it implies that one must not only be defensive about it, but also remember to actually do some driving, instead of going so slowly you become a hazard yourself.
It also implies that, to be prepared, you should be focused on the situation; this is why my state, plus others, have passed laws regulating what passengers teens are allowed to have in their cars, to avoid the problem of a bunch of teens having so much fun that the driver is distracted by his or her friends.209.244.30.221 (talk) 21:16, 24 December 2008 (UTC)[reply]
It would also usually be understood to include driving in a way that doesn't scare the bejeezus out of other drivers. Just because you know you can fit into that gap between the truck and the SUV doesn't mean that everyone else does and has the same trust in your driving abilities. Causing other parties to have to break hard because they anticipate running into you or speed up to prevent you from breathing up their exhaust pipe is usually considered opposite to defensive driving. 76.97.245.5 (talk) 23:06, 24 December 2008 (UTC)[reply]
At heart it means assuming that at any instant, any other driver may do something completely wrong. So instead of assuming that (for example) someone will definitely turn left because they have their left turn signal on - you have to consider the consequences should they not do so - and make sure that you have time to stop - or a space to safely move into. As the name implies, it's a matter of being defensive. In the UK (at least) you can actually take 'Defensive Driving' courses - which (assuming you pass) will get you a reduction in the cost of car insurance (I think I got 15% off...but that was a while ago). SteveBaker (talk) 01:45, 25 December 2008 (UTC)[reply]
And then there are the people who take defensive driving to mean "defending their patch of the road". And they will defend that patch of road fiercely... ;) --Jayron32.talk.contribs 01:56, 25 December 2008 (UTC)[reply]
You can find hundreds of sites related to defensive driving, which makes me suspect that in the U.S., some states now require a course that calls itself by that name. What SteveBaker said above is true (I remember my driver's ed teacher saying, "You never have the right of way; you can only yield the right of way," an excellent philosophy for not getting broadsided. There are also some advanced-defensive-driving schools that have things like skidpads (to let you safely practice panicky stops on a wet road). As for other drivers, it's good to keep in mind that 50% of them are below average. --- OtherDave (talk) 02:38, 25 December 2008 (UTC)[reply]
In NZ, if you take an approvaed defensive driving course, you reduce the amount of time you have to spend on a restricted license before you can take the full license test. [14] Nil Einne (talk) 12:30, 26 December 2008 (UTC)[reply]
As everyone knows "the best defense is a good offense". With that in mind, I practice offensive driving and run everyone else off the road (before they get a chance to cause me to get in an accident). :-) StuRat (talk) 04:48, 25 December 2008 (UTC)[reply]
Per Stu's suggestion, see the W.C. Fields film "If I Had A Million" starting at 1:30 in the clip. He (edited to clarify: Fields' character Rollo, not our StuRat) hires cars and drivers to follow him and run off the road anyone whose driving annoys him. Quote: "Perhaps you'll move over next time, you road hog!" Edison (talk) 05:09, 25 December 2008 (UTC)[reply]

Basically, when told to drive defensively, you should interpret the underline message as "drive to cause accidents, but do not get yourself involved in those accidents". —Preceding unsigned comment added by 68.72.221.139 (talk) 04:13, 1 January 2009 (UTC)[reply]

December 25

indians born in the US

can anyone tell me where i can get a list of famous indians who were born in the US? —Preceding unsigned comment added by 122.50.128.178 (talk) 07:04, 25 December 2008 (UTC)[reply]

See List of Indian Americans

Please search before asking next time. Rachmaninov Khan (talk) 09:38, 25 December 2008 (UTC)[reply]

You might make clear whether you mean Americans whose families came from India (as I think you do), or Native Americans, who often refer to themselves as Indians. --- OtherDave (talk) 15:05, 25 December 2008 (UTC)[reply]

Random letters

I randomly typed letters into the google search box (they turned out to be skdjfkds) and it came up with search results[15] instead of the expected reject notice. Do these letters actually mean something? Julia Rossi (talk) 10:49, 25 December 2008 (UTC)[reply]

If you look at result ~3, it apears to be a sort of lore ipsum. Or 1000 monkeys at 1000 typewriters? BrainyBabe (talk) 11:31, 25 December 2008 (UTC)[reply]
and what are the chances? One monkey on a keyboard, hehe. Julia Rossi (talk) 11:34, 25 December 2008 (UTC)[reply]
This is ye olde magick word (orally transmitted from the Dreamtime to the tribal elders) which is known to turn unsuspecting humans into supersonic wombats. Due to the ensuing atmospheric friction, of course, the wombats turn into magnificent balls of celestial fire.
I dimly remember reading about a case where three wise men from the Orient mistook the illuminating critter for a comet and - oh well, I seem to have forgotten the rest. --Cookatoo.ergo.ZooM (talk) 12:32, 25 December 2008 (UTC)[reply]
If you look at the location of those letters on the keyboard, it's probably not as random as you think. Those are closely-clustered letters around the areas your hands traditionally rest on a keyboard, and probably more likely than most to be randomly mashed when typing 'random letters'. There are a lot of people typing gibberish on the internet. ~ mazca t|c 13:30, 25 December 2008 (UTC)[reply]
Indeed, these "asdfjkl;" combinations are not random. If you google a more systematic "random" combination of the keys on which your fingers rest, you get even more hits: 115,000 for asdfjkl (left to right), 13,200 for alskdjf (alternating hands, inward bound). Even when including your pinkies and expanding these patterns to eight letters (and using the German keyboard layout, because it has an "ö" instead of a semicolon, which I don't know how to google), you still get 7,250 hits for asdfjklö and 279 for aösldkfj.
I used to occasionally play trivia quizzes online (in an IRC format not connected to WP), and one of the participants always let off her steam when someone else got the answer first by typing something like "asjdfksl" or "jsdkfjsffjl" or even "jasdlfkjalskdjaslkdfja;lskjalsj;dfalsd;flaljskdlkjf" (these are my random attempts), it came to be one of her pet-signatures, and we even greeted her with "asjdfkj!" when she entered the quiz room.
It would be interesting to see a statistical evaluation of a "type some random letters as fast as you can" test, but I found nothing. ---Sluzzelin talk 14:25, 25 December 2008 (UTC)[reply]
Ah, well I still found nothing worth linking here, but I did learn that the "asdfjkl;" keys lie on the home row and are called home keys, and that asdf is "often used as a metasyntactic variable". ---Sluzzelin talk 14:54, 25 December 2008 (UTC)[reply]
There are well over a billion web sites and over 30 billion individual web pages on the Internet - and Google also indexes things like usenet postings, forum systems, wiki's and such - so the total is likely to be quite a bit higher than that. Even for a short, completely random sequence, the odds are pretty good of finding it somewhere. As others have said - the less random your sequence, the higher the odds of it happening to be out there...and something you just type quickly and without special thought is MUCH more likely than a truly random sequence. So I don't think you should read anything special into that. SteveBaker (talk) 15:06, 25 December 2008 (UTC)[reply]
Thanks all for your interesting answers and helpful efforts. I was completely focused on the google revelation and only after your replies did I see skdjfkds doesn't involve any other row, just alternating hands, making the chances close to core. : ) Julia Rossi (talk) 03:24, 26 December 2008 (UTC)[reply]
See infinite monkey theorem for more info. I'm sure that's somehow related... flaminglawyerc 16:31, 27 December 2008 (UTC)[reply]

Toilet seat

Why is it that women complain about men not putting down the toilet seat? Is it that it is just good manners to put it down yourself so that the next person doesn't have to, is it because the woman might not look before sitting down, or is it something else that I am missing? Thanks Crack in the road (talk) 15:17, 25 December 2008 (UTC)[reply]

Not surprisingly, there is some information under toilet seat including a link to an economic efficiency study.
(In my world you don't need to put down the seat, because it's already down. You shouldn't be using a private toilet from the standing position. If you believe you're a straight-shooter, I recommend considering how the height of drop influences the ballistics of a splash and you can try the standing position while wearing shorts and registering wether your bare legs feel anything prickly during the process. Now think about the floor and walls ...) ---Sluzzelin talk 15:43, 25 December 2008 (UTC)[reply]
I have never really understood the complaint. The idea seems to be that it's somehow wrong for someone to have to put the toilet seat down, but a non-issue that someone else has to lift it up. And in any case, to hear people tell it, it's as if this was a Herculean effort that takes a great toll on the poor individual forced to change the position of the toilet seat. (I did check out the the article, and the idea that having the seat down somehow makes the bathroom more aesthetically pleasing, for example, strikes me as pretty ludicrous. Hygienic reasons also strike me as irrelevant -- I mean, if people were constantly getting sick because of this, that would be different, but I don't believe this to be the case, at all. You should be cleaning the toilet and the area around it on a regular basis anyway. I mean, if you miss, you clean up after yourself, right? And if you don't, you're a dick, but that's got nothing to do with the position of the toilet seat.)
I'd say the whole thing has a lot more to do with psychology -- the need to establish control over your environment, things like that -- than anything else. -- Captain Disdain (talk) 16:49, 25 December 2008 (UTC)[reply]
To bring a (possibly biased) woman's perspective to the issue: Women need the seat down 100% of the time. Men, as far as I know, need it up when urinating and down when defecating. Therefore, it spends more time down than up and that's what its default position should be. That, and most women tend to be more squeamish about touching bacteria-riddled objects with their hands than most men, and would prefer not to be the ones moving it back to what should be its default position anyway. (Of course, this "logic" ignores the fact that things like telephones and doorknobs are likely to be just as germy as toilet seats... On that note, please bear in mind that not all of the above is necessarily my personal opinion, just some general observations.) Cherry Red Toenails (talk) 00:27, 26 December 2008 (UTC)[reply]
Isn't 'toilets are bacteria-riddled' a myth? AFAIK, toilet seats are actually quite clean. The door knob has far more bacteria than the toilet seat. 67.184.14.87 (talk) 14:50, 26 December 2008 (UTC)[reply]
I don't care if it's up or down; I wouldn't touch it either way. SN0WKITT3N 00:49, 26 December 2008 (UTC)[reply]
But you DO touch it! With your A*** (unless you are percher)--GreenSpigot (talk) 02:32, 26 December 2008 (UTC)[reply]
One possible explanation might be that in my experience women are generally more houseproud than men and a lowered toilet seat could look neater when your mother comes to visit :-) However, when I find the seat down I have no problem raising the seat, so why can't women simply lower the seat without making such an issue about it? Astronaut (talk) 01:19, 26 December 2008 (UTC)[reply]
My solution would be (in a house with 1 woman, 1 man): If you are a man, leave it down. If you are a woman, leave it up. ie leave it as you would expect to find it if you were the other person. --GreenSpigot (talk) 02:30, 26 December 2008 (UTC)[reply]
Very tactful. Maybe it's because Emperors kept their toilet seat down. =P Julia Rossi (talk) 03:27, 26 December 2008 (UTC)[reply]
I may have an unusual perspective on this. I am male, and do pee standing up. However, I leave the seat down (but not the cover, obviously). Why do I do this ? Two reasons. First, a seat occasionally falls down mid-whiz, with very messy results. Second, I have back problems, so prefer to avoid any unnecessary bending. I try to avoid peeing on the seat, but, if I do, I clean it up after. Before you complain about this spreading germs, note that urine, unlike feces, tends to be quite antiseptic. StuRat (talk) 04:31, 26 December 2008 (UTC)[reply]
With respect StuRat I think you might mean sterile not antiseptic ;-) Richard Avery (talk) 14:30, 26 December 2008 (UTC)[reply]
I did mean antiseptic, as the acidity of urine can kill bacteria. However, I should have said "quite sterile and slightly antiseptic". StuRat (talk) 15:13, 26 December 2008 (UTC)[reply]

I've heard that some people have been injured by accidentally sitting on a toilet with the seat up when it's dark or they're in a rush to use it or something. --124.254.77.148 (talk) 06:34, 26 December 2008 (UTC)[reply]

Back to basics: if a toilet has a lid, it should be lowered. That of necessity involves lowering the seat too. BrainyBabe (talk) 13:48, 26 December 2008 (UTC)[reply]
I've never understood this, either. I'm a man—a he-man, if I do say so myself—and I put the lid back down when I'm done. I've had a running disagreement with my to all appearances equally testicular brother-in-law about this. He seems to take any female interference with the male toilet ritual as a challenge to his postion within society. Me, I think the toilet is a machine, and a real man leaves a machine in good order, be it a Camaro or a commode. I've had a lot of plumbing experience, having installed, repaired, and unstopped many, many toilets, so maybe that's the difference. You put the lid down. It keeps the dog out. If something goes flying in the bathroom, it can't fall in there. You don't have to look at any skid marks or rings. You can sit on it or put your foot on it. The top is often decorative and should be displayed to maintain the integrity of the decor (No, no, no—he-man, remember?). You put the lid down. Just one thing, ladies—no slip covers, please; I have trouble aiming while holding the seat up with my knee. --Milkbreath (talk) 16:14, 26 December 2008 (UTC)[reply]
I put the lid down ever since reading in the Straight Dope that flushing creates an aerosol. —Tamfang (talk) 23:06, 28 December 2008 (UTC)[reply]
But without the seat up, you'd have to remember to leave water out for the dog, and he'll soak the whole floor with it. I even had a cat who could drink from the toilet, quite impressive. StuRat (talk) 17:45, 26 December 2008 (UTC)[reply]

See also the dreaded Toilet-related injury: a small boy could be traumatized by an ill balanced lid, or one with a padded cover, which falls at an inopportune moment. Edison (talk) 22:28, 26 December 2008 (UTC)[reply]

"The toilet: friend or foe?" lmfao!! —Preceding unsigned comment added by 82.43.88.87 (talk) 22:31, 26 December 2008 (UTC)[reply]

From ABC News:

Fact or Myth? You can get infections or illnesses from sitting directly on a public toilet seat.

Answer: Myth

"Just sort of sitting on the seat and having that contact with the skin on your butt isn't going to be a way of transmitting an infection," said Elizabeth Scott, co-director and founder of the Simmons Center for Hygiene and Health in Home and Community Settings at Simmons College in Boston.

"I think that one's associated with the fact that we all find public toilets very disgusting," she said, adding that you were more likely to get sick from touching the toilet seat or the flush handle with your hand.

Dr. J. Owen Hendley, professor of pediatric infectious disease at the University of Virginia Children's Hospital, said that this myth has been a persistent one.

Of getting an infection, he said, "I guess you could, but I've never known of a documented case where that actually happened."

But that has not stopped the myth. Hendley noted that the concern might have originated with a fear that syphilis could spread through toilet seats. He said that that fear is likely behind the design of many public toilet seats in which the seat itself is open in the front, preventing contact between the person and the seat in that area.

But the knowledge that sitting directly on the seat doesn't spread the germs doesn't seem likely to make it more appealing.

[16] 216.239.234.196 (talk) 15:29, 29 December 2008 (UTC)[reply]

Hosting an Annual Party

Hi, can anyone help me with few tips , as i have been chosen randomly to host as an announcer, for the annual official party to be held shortly this month ,it is mailly for rewarding the employees for the year. i am nervous and do not want to let go it as well as i have never done this before, can anyone help me ovwercome this nervousness and suggest few tips,your answers will be treasured. —Preceding unsigned comment added by 203.122.36.6 (talk) 16:11, 25 December 2008 (UTC)[reply]

  1. Write out what you intend to say (almost word-for-word) and then
  2. recite your remarks in front of a friend or a family member several times and then
  3. when it's showtime, pretend that you're in "practice" mode and then,
  4. break a leg! hydnjo talk 16:39, 25 December 2008 (UTC)[reply]
If you are also going to be handing out the awards make sure that you have a list of who gets one and if there are physical awards that they are correctly labled. If you make an error just laugh it off and carry on. These are your co-workers and are probably going to be supportive and aren't going to heckle you. CambridgeBayWeather Have a gorilla 12:29, 26 December 2008 (UTC)[reply]
Practice recording yourself, at least audio or video if possible. BrainyBabe (talk) 13:49, 26 December 2008 (UTC)[reply]

Be sure you have a good start and a good finish. Write out, practice, perfect and learn the first 100 words, or so. The same with the conclusion. You are inexperienced so NO jokes. learn the beginning and end by heart. Break the middle down into sections. Understand what needs to be said in each. Put beginning, sections and end on separate post cards. Punch cards in top left hand corner and string them together (in case you fumble). If you have a practiced start you will get away smoothly, and clear most of your nerves. The end is important because you have the safety of knowing you can cut to a solid end from anywhere in your presentation. You won't just fade away. Nerves are normal. be grateful for them - they pump up your performance. Good luck.90.0.7.27 (talk) 15:20, 26 December 2008 (UTC)DT[reply]

Also, if you feel the need to expend some energy once up there, wiggling your toes a little always helped me, though your mileage may vary on that.Somebody or his brother (talk) 20:49, 26 December 2008 (UTC)[reply]

Lucid Dreams

I've been talking with my friends on the internets (well, not friends as such, more acquaintances) about lucid dreams. Many of us have extreme desires for a certain celebrity and it was suggested by someone about three weeks ago that we try lucid dreaming for this. I've looked at a few online guides, done the reality check and dream recall stuff but I still haven't had one. I know lucid dreaming isn't something you can do overnight, but what advice is there for this? Any good links or books to read? SN0WKITT3N 22:42, 25 December 2008 (UTC)[reply]

Oh crap I just realized what an awful unintentional pun there is in that :/ SN0WKITT3N 23:39, 25 December 2008 (UTC)[reply]
Overnight Pun, what pun? hydnjo talk 00:05, 26 December 2008 (UTC)[reply]
I know this is 'original research', but the best results I've had with encouraging lucid dreams, have been when I've had LOTS of time on my hands and have slept in day after day. Knowing that I don't have to get out of bed for anything drags out my sleep pattern and results in me being asleep through a morning that I would normally be awake during. This in turn seems to mean that my mind is active in a different way whilst I sleep. Add to this some unusual dietry intake (such as more cheese or alcohol than you're used to) and perhaps a different environment (such as a hotel room or a friends spare room) and you should up your chances considerably. This current seasonal break could be the perfect opportunity. Although I'm not sure how you would go about incorporating your 'certain celebrity'. Try influincing your dreams by spending plenty of time reading about him/her, watching them on video, etc. before you go to sleep. This often affects my dreams. Good luck. —Preceding unsigned comment added by 91.111.67.44 (talk) 22:40, 26 December 2008 (UTC)[reply]
The process of dreaming seems designed to eliminate all traces. This has survival value in the process of evolution, so if caveman Og dreams that caveman Gog tried to kill him, he does not get into a fight as soon as he wakes up, thinking the attack was real. You may be having vivid, realistic dreams for 5 hours out of an 8 hour sleep period, but remember nothing. Dreams seem evanescent, like unrehearsed nonsense syllables seen briefly once, or unattended conversations overheard, and they go away in seconds unless somehow noted. So if I am awakened by an alarm clock at random times of the night, I might be more aware of dreams I had, than if I slept uninterrupted for 8 hours. This is just an observation and not a suggestion. Edison (talk) 02:04, 27 December 2008 (UTC)[reply]

December 26

A Captain the Head of Guinea?

How did a Captain, a mere junior military officer, become the interim head of state of Guinea during the 2008 Guinean coup d'état? Why not a General? Acceptable (talk) 00:25, 26 December 2008 (UTC)[reply]

It's possible he's just a figurehead - the public face of the coup while the generals remain as the "power behind the throne". Exxolon (talk) 00:44, 26 December 2008 (UTC)[reply]
I don't think it's the rank per se, but rather means, motive, and opportunity. Although the Wikipedia article on Samuel Doe doesn't mention his rank, he was a master sergeant when he helped overthrow Liberian president William R. Tolbert, Jr., in 1980. Doe remained in power till 1990. --- OtherDave (talk) 03:33, 26 December 2008 (UTC)[reply]
He's not the only one, Muammar al-Gaddafi was a captain and only got the promotion after the coup. Jerry Rawlings was a Flight Lieutenant and Gamal Abdel Nasser wasn't a general either but his group made one president. It may be that there non-general officers have less to lose in a coup. Though it would also depend on their being able to pursade their fellow officers and involve the major part of the countries armed forces. CambridgeBayWeather Have a gorilla 12:21, 26 December 2008 (UTC)[reply]
The other obvious factor is that the people in government often have some say in the appointment of generals and in general who controls the armed forces. They may therefore choose people loyal to them and unlikely to stage coups whereas those who may stage coups are kept at relatively low ranks for precisely that reason Nil Einne (talk) 12:26, 26 December 2008 (UTC)[reply]
My understanding was that the coup was basically staged by a cadre of 'junior military officers' - which would explain why one is the leader of the resulting junta. 161.181.153.10 (talk) 21:45, 26 December 2008 (UTC)[reply]
As a rule of thumb, promotion to low and mid officer ranks is based on ability; promotion to high officer ranks is based on politics. --Carnildo (talk) 00:47, 1 January 2009 (UTC)[reply]

Subtitles

Why do subtitles sometimes show words that are already on the screen? For example, when the words "Joanne Buck: Director" are on the screen, why are they also in subtitles? 60.230.124.64 (talk) 02:47, 26 December 2008 (UTC)[reply]

An educated guess: because those subtitles can also be used to provide the source text for translation into other languages, so that the translation may be performed without the translator actually watching the TV show or movie in question. So in order to translate text that appears on the screen that may be vital to understanding the plot it must also be included in the source text. Then nobody bothers to go back and remove the unnecessary text strings from the original transcript. (There may well be other reasons as well, but my personal experience with localization work has included many instances of comparable situations.) -- Captain Disdain (talk) 03:43, 26 December 2008 (UTC)[reply]
Also, at the time of creating the subtitles, it is commonly not known exactly where on screen they will be. That means there is a chance that they will obscure the original "Joanne Buck: Director", and thus it is safer to include it as well. /85.194.44.18 (talk) 10:40, 26 December 2008 (UTC)[reply]
There may also be weird contractual/union rules about credits. I know that in the computer games industry, the rules for presentation of credits are long, complicated and (on occasion) bizarre. SteveBaker (talk) 01:24, 27 December 2008 (UTC)[reply]
I don't think the original poster was referring to credits, but rather a situation where there's a sign that says "Joanne Buck: Director" somewhere on screen. (You're absolutely right about the games industry, though; I just finally got around to completing Crysis a day or two ago, and the list of credits at the end is utterly ridiculous. As it happens, I also decided to wallow in nostalgia tonight and played through Half-Life: Blue Shift for the first time in, what, about eight years, and I was struck by the contrast: the in-game credits on that one included the names of maybe fifteen people and five companies. None of them were even credited with any kinds of titles. I very much doubt you'd see that in any modern game.) -- Captain Disdain (talk) 02:25, 27 December 2008 (UTC)[reply]
Crysis is a rather special case because it's not just a game - but also a 'game engine' that other games (some from other companies) will use as the core of their operations in the future. Their development team is especially large. They have more engineers working on graphics software alone than comprised the entire game team at my previous job - and more (by far) than the company I'm working on now. Crysis is an impressive achievement - but it cost them dearly to make it - and unless they can make a ton of money from selling the engine - they'll lose money no matter how many copies of the game they ultimately sell.
But as for the credits - I think it's plausible that someone at some time negotiated a rule that accreditation has to be in subtitles as well as 'on screen' for some obscure reason - probably relating to subtitles used in the context of providing translations - and that rule being applied mindlessly in the case of subtitles for the deaf. I don't know for sure though. SteveBaker (talk) 15:39, 27 December 2008 (UTC)[reply]
Yeah, I know -- I wasn't so much referring to the number of people who worked on Crysis; rather, I meant the endless litany of people whose actual connection with the project would've been peripheral at best. I mean, they list the marketing and PR people for literally thirty countries, among other people. But that's EA for you.
...anyway, I'm not sure if I'm being unclear or if I'm just misreading you, but the point I'm making is that I don't think the "Joanne Buck: Director" text mentioned by the original poster refers to the director of the movie, but rather something -- perhaps a sign on someone's desk or something -- that appears in the movie itself. So I don't think accreditation has much to do with it. -- Captain Disdain (talk) 23:33, 27 December 2008 (UTC)[reply]

Video Card RAM

Moved to Wikipedia:Reference desk/Computing#Video Card RAM Nil Einne (talk) 10:21, 26 December 2008 (UTC)[reply]

foreign born in the US

can anyone tell me in which US city, there is the most conentration of i) indian americans ii) chinese americans iii) japanese americans iv) african americans —Preceding unsigned comment added by 122.50.137.154 (talk) 11:05, 26 December 2008 (UTC)[reply]

Wouldn't a European (white) American be foreign born too? Nil Einne (talk) 12:23, 26 December 2008 (UTC)[reply]
I'd say they want to know about people born in India, China, Japan, and Africa who are now living in the US. And yes, people born in Europe and now living in the US are also foreign born, but that's not what they were asking about. StuRat (talk) 15:08, 26 December 2008 (UTC)[reply]
Are you asking about people who were born in Africa and live in the U.S. or "African Americans" whose African ancestor were brought to the U.S. as slaves before such importation became illegal in 1808? And do you want the cities with the largest number or the largest percentage of such people? Some small communities have a small population but 100% composed of one such group. Edison (talk) 16:51, 26 December 2008 (UTC)[reply]

i)New York City seems to have the largest number of immigrants from India per [17] . Edison (talk) 19:13, 26 December 2008 (UTC)[reply]

i want the largest percentage of the above mentioned groups please tell me. —Preceding unsigned comment added by 122.50.132.53 (talk) 17:45, 26 December 2008 (UTC)[reply]

In general, see Lists of U.S. cities with large ethnic populations or do your own search at the U.S. Census database. 1) We have no article List of U.S. cities with large Indian American populations, but see Millbourne, Pennsylvania which has about 63% Indian American population. 2) See List of U.S. cities with large Chinese American populations and see List of U.S. communities with Asian American majority populations .3)We have no article List of U.S. cities with large Japanese American populations 4) See List of U.S. cities with large African American populations says Detroit is 83.73% African-American. There are some all smaller communities with much larger African-American majorities, as shown at List of U.S. communities with African American majority populations. For instance McMullen, Alabama (population 66) is 100% African American, as is Birdsong, Arkansas (population 40). It is unlikely that any of the inhabitants were born in Africa in the two small communities, but some might have been. Edison (talk) 22:05, 26 December 2008 (UTC)[reply]

Missing storage

When one buys, for example, a 4 GB USB drive and plugs it into the computer, there is not actually 4 GB of memory available. Instead, it is always several hundred MB's smaller. The larger the marketed storage capacity, the larger the difference between this marketed value and the actual value. Why does this happen? Is there a name for this phenomenon? Should there not be laws regarding this practice? Acceptable (talk) 18:21, 26 December 2008 (UTC)[reply]

There is some disagreement as to how big a GB is. Binary prefix seems to be the most relevant Wikipedia article. Algebraist 18:29, 26 December 2008 (UTC)[reply]
(edit conflict)It's a combination of a couple of factors. One of them is the difference between definitions - see Binary prefix. As far as your computer is concerned, a kilobyte is 1024 bytes, and a megabyte is 1024 kilobytes. But in some contexts (including, conveniently, hard-drive marketing) you use powers of ten instead, so there are 1000 bytes in a kilobyte and 1000 kilobytes in a megabyte. Hence, a binary megabyte ends up being a fairly significant fraction larger than a decimal megabyte. As they're smaller, it entirely makes business sense to sell hard drives in 1000-multiple bytes instead.
The other factor is simply the directory file - when you format it, a hard drive needs to use a certain fraction of itself to store information on where everything else on it is stored. This can be 5% or so of the total capacity - so the larger the hard drive, the larger in absolute terms this file will be. Hence, even once you've figured out the binary/decimal difference in a hard drive's size, you may still find yourself losing several gigabytes in every hundred to the directory file. ~ mazca t|c 18:32, 26 December 2008 (UTC)[reply]
Then there's also the possibility of bad sectors, although this is more likely on older devices. StuRat (talk) 19:59, 26 December 2008 (UTC)[reply]
And in FAT/FAT32/VFAT formatted flash drives, the file allocation table itself requires some space to be put into, Tohd8BohaithuGh1 (t·c·r) 01:29, 29 December 2008 (UTC)[reply]

wiki codes about Sociology,Islamit, international Relations,Journalism

Respected sir i need the best wiki codes of these subjects for Educational purposes. Kindly inform me the exact location of wiki codes Thanx. —Preceding unsigned comment added by Awan 2008 (talkcontribs) 18:29, 26 December 2008 (UTC)[reply]

What do you mean by 'wiki codes'? Algebraist 18:30, 26 December 2008 (UTC)[reply]
Sociology, Islamit, international Relations, Journalism ? —Preceding unsigned comment added by 82.43.88.87 (talk) 18:34, 26 December 2008 (UTC)[reply]

Question: least talked about state

What is the least popular talked about United State? 60.230.124.64 (talk) 22:43, 26 December 2008 (UTC)[reply]

You could try Google. "north dakota" (in quotes) gets 70,900,000 hits, which is the lowest I can find, but I didn't try all of them. Also try Google Trends[18]. That could be taken as the least talked about online, but what do you mean "least talked about"? On TV? Usenet? In person? Radio? Newspapers? In the USA? In the world? --Maltelauridsbrigge (talk) 23:11, 26 December 2008 (UTC)[reply]
We're wise to your game, buster: If we talk about it enough here, it won't be the least-talked-about. Nice try. Who's the least interesting person on earth? Doesn't the fact that he is the least interesting make him interesting? --Milkbreath (talk) 23:16, 26 December 2008 (UTC)[reply]
Excellent, you just scored one for the inclusionists! The least notable are now notable! Off to rewrite WP:NOTE. ---Sluzzelin talk 00:38, 27 December 2008 (UTC)[reply]
@ Milkbreath due to bumped post (naughty sluzz): According to this, you've just given him a nice start,  ;) Julia Rossi (talk) 00:04, 27 December 2008 (UTC)[reply]

Well, I just did search on all the state names in Google and got the following counts. Google counts sometimes display weird variations and I think the very low count for Louisiana must be some sort of anomaly, not a meaningful result. And of course the high counts for New York and Washington are because many of the hits on those two will really be for the cities of the same names; similarly Kansas and Oklahoma for the same reason, and Georgia because of the country, and maybe some others. But just for fun, here are the numbers:

    1. "New York"       1,190,000,000   (see note above)
    2.  Washington        703,000,000   (see note above)
    3.  California        618,000,000
    4.  Texas             479,000,000
    5.  Florida           431,000,000
    6.  Georgia           369,000,000   (see note above)
    7.  Virginia          279,000,000
    8.  Ohio              265,000,000
    9.  Michigan          255,000,000
   10.  Illinois          241,000,000
   11.  Colorado          219,000,000
   12.  Arizona           213,000,000
   13.  Hawaii            207,000,000
   14.  Pennsylvania      206,000,000
   15.  Indiana           199,000,000
   16.  Oregon            195,000,000
   17. "New Jersey"       191,000,000
   18.  Massachusetts     187,000,000
   19.  Kansas            185,000,000   (see note above)
   20.  Wisconsin         183,000,000
   21.  Minnesota         179,000,000
   22.  Alaska            176,000,000
   23.  Maryland          172,000,000
   24.  Missouri          162,000,000
   25.  Tennessee         161,000,000
   26. "North Carolina"   160,000,000
   27.  Alabama           158,000,000
   28.  Maine             156,000,000
   29.  Montana           154,000,000
   30.  Oklahoma          153,000,000   (see note above)
   31.  Iowa              153,000,000
   32.  Kentucky          148,000,000
   33.  Utah              145,000,000
   34.  Nevada            139,000,000
   35.  Connecticut       139,000,000
   36.  Mississippi       135,000,000
   37.  Arkansas          129,000,000
   38.  Idaho             124,000,000
   39.  Nebraska          115,000,000
   40. "South Carolina"   114,000,000
   41.  Delaware          111,000,000
   42.  Vermont           108,000,000
   43. "New Mexico"       106,000,000
   44. "New Hampshire"    102,000,000
   45.  Wyoming            96,800,000
   46. "West Virginia"     88,500,000
   47. "Rhode Island"      85,100,000
   48. "South Dakota"      79,400,000
   49. "North Dakota"      76,100,000
   50.  Louisiana          15,800,000   (see note above)

--Anonymous, 01:17 UTC, December 27, 2008.

You must be right about the anomaly, Louisiana gives me 133 million hits at the moment for me. Either that or you spelt it wrong when searching. North Dakota gives me about 80 mill and South 83 mill Nil Einne (talk) 08:41, 27 December 2008 (UTC)[reply]
Virginia will include West Virginia ... etc. Kittybrewster 21:09, 27 December 2008 (UTC)[reply]

Sorry 'bout this but this thread prompts me to wonder: who is the most recognizable RD'er (Lc and myself excluded)? -hydnjo talk 23:03, 27 December 2008 (UTC)[reply]

Me? —Preceding unsigned comment added by 82.43.88.87 (talk) 00:02, 28 December 2008 (UTC)[reply]
Yes - you've received 100% of the responses! hydnjo talk 00:07, 28 December 2008 (UTC)[reply]

Some of the best questions are born in caves, and to continue the fun, here's another table. To exclude the ambiguity of Virginia, New York, or Washington, I googled "State of Virginia" and so forth. The table below shows California on top, while New York and Washington moved down three ranks. The ranking in parentheses is from Anonymous' list above with the exception of Louisiana which gets rank 36th, bumping all lower-ranked States down by one rank. Delaware and Connecticut move up more than twenty ranks, Minnesota and Missouri move up more than ten ranks. As predicted, Virginia drops quite a bit, over twenty ranks, but so do Massachusetts and Pennsylvania. Anyway, the bottom three ranks are identical on both lists: Rhode Island, South Dakota, and the overall winner of least talked about about States is North Dakota. Maltelauridsbrigge's intuition had it right from the beginning!

Rank (Rank for name alone) "State of ..." Hits on google
1 (3) California 6,520,000
2 (4) Texas 4,630,000
3 (5) Florida 4,260,000
4 (1) New York 2,990,000
5 (2) Washington 2'550'000
6 (10) Illinois 2,370,000
7 (9) Michigan 2,170,000
8 (8) Ohio 2,070,000
9 (21) Minnesota 1,810,000
10 (6) Georgia 1,590,000
11 (24) Missouri 1,530,000
12 (17) New Jersey 1,410,000
13 (22) Alaska 1,370,000
14 (35) Connecticut 1,340,000
15 (12) Arizona 1,270,000
15 (20) Wisconsin 1,270,000
17 (16) Oregon 1,220,000
18 (11) Colorado 1,160,000
19 (28) Maine 1,130,000
19 (30) Oklahoma 1,130,000
21 (42) Delaware 1,080,000
22 (26) North Carolina 1,050,000
23 (23) Georgia 1,030,000
24 (13) Hawaii 1,020,000
25 (15) Indiana 1,010,000
26 (34) Nevada 956,000
27 (25) Tennessee 947,000
28 (7) Virginia 939,000
29 (31) Iowa 910,000
30 (46) Wyoming 894,000
31 (36) Louisiana 856,000
32 (27) Alabama 835,000
33 (40) Nebraska 828,000
34 (33) Utah 822,000
35 (29) Montana 774,000
36 (47) West Virginia 735,000
37 (14) Pennsylvania 733,000
38 (37) Mississippi 718,000
39 (38) Kansas 657,000
40 (45) New Hampshire 624,000
41 (38) Arkansas 619,000
42 (43) Vermont 614,000
43 (44) New Mexico 582,000
44 (41) South Carolina 571,000
45 (18) Massachusetts 564,000
46 (39) Idaho 558,000
47 (32) Kentucky 550,000
48 (48) Rhode Island 461,000
49 (49) South Dakota 352,000
50 (50) North Dakota 242,000

---Sluzzelin talk 00:36, 28 December 2008 (UTC)[reply]

Sluzzelin, did you write a program or did you do it the "old-fashioned" way? hydnjo talk 00:42, 28 December 2008 (UTC)[reply]
I do everything the old fashioned way. I haven't written a program in over fifteen years! I also forgot to mention, that I'm of course aware that googling "state of ..." is not necessarily a better approximation of "least talked about". It's just another angle, and it seems to confirm North Dakota's notability for being non-notable. ---Sluzzelin talk 00:46, 28 December 2008 (UTC)[reply]
Talk about dedication, now that's impressive OR indeed - Geesh! hydnjo talk 01:17, 28 December 2008 (UTC)[reply]
What can I say? I tested a couple of states and found the results sufficiently interesting to turn it into refdesk-spam. I had fun, did it while listening to the radio, and it didn't take as long as it seems (35 minutes). ---Sluzzelin talk 01:47, 28 December 2008 (UTC)[reply]
I'll have some of what you're listening to, 35 minutes is quite a bit. "Some of the best questions are born in caves" is one of the best quotes I've seen lately - thanks for your insight and personal dedication to accuracy :) hydnjo talk 02:08, 28 December 2008 (UTC)[reply]
Alireza Eftekhari on IranianRadio.com - Traditional (iTunes). Great for humming numbers into your short-time memory while jumping from window to another. Sacrilegious. :-) ---Sluzzelin talk 02:36, 28 December 2008 (UTC)[reply]
And so it seems that after all of this conversation, North Dakota takes top honors - congratulations North Dakodians, you've gotten top honors in this quirky survey. Hey, someone had to win :) hydnjo talk 02:30, 28 December 2008 (UTC)[reply]
To be fair, the significant drop by Massachusetts and Pennsylvania that Sluzzelin mentions above is probably due to the fact that they're not "states", they're officially commonwealths. At the moment, I get 2,680,000 GHits for "Commonwealth of Massachusetts". jeffjon (talk) 14:15, 29 December 2008 (UTC)[reply]

December 27

Career officer

What is a career officer? — Sebastian 01:28, 27 December 2008 (UTC)[reply]

Someone who helps and advises on what career would suit you and what exams you need to do it.--GreenSpigot (talk) 02:05, 27 December 2008 (UTC)[reply]
Or, in another context, it typically refers to a military officer who purposefully pursues a military career (as opposed to an officer who only serves for a while and then does something else). -- Captain Disdain (talk) 02:12, 27 December 2008 (UTC)[reply]

What happens if you keep talking to your mind?

Talk to your mind? How rediculous you say. But, you somewhat (actually very little) guide your thoughts. And you can "address" your mental chemistry by taking something to alter it. You, right now are the "driver". Sort of, anyways. Talking "to" your mind, more or less.....Or at least the observer of consciousness. I've heard the expression "be at one with your subconscious, and be whole!" but then a new subconscious would form and take over. Just like when you "wake up" in a dream and realize your dreaming. Because something was running the part of your mind that you took over. Now a new "driver". Always a man behind a curtain. If you get a few stages into your mind, what happens? If you keep receeding further and further, would you eventually reach the seat of all consciousness? And if you do access that nervous system alpha box that runs existence itself, what then?!--Dr. Carefree (talk) 01:34, 27 December 2008 (UTC)[reply]

I've done what you describe for decades. What happens? You have a career and a rich life. Perhaps you become a Wikipedia editor or administrator. Edison (talk) 01:56, 27 December 2008 (UTC)[reply]
See Neuro-linguistic programming and Hypnosis for starters —Preceding unsigned comment added by GreenSpigot (talkcontribs) 02:03, 27 December 2008 (UTC)[reply]
Whatever dreams I might have I don't remember having had any since I was quite young and I don't think in words much so speaking to myself is out. However I don't seem to have missed out on any of the career and rich life and editor on Wikipedia business so I don't think there is a link in that direction. Dmcq (talk) 15:44, 27 December 2008 (UTC)[reply]

Do we have an article, or section of an article, about...

...the ethics of sexual activity with people who are legally adults, but with mental retardation giving them the mental age of a minor? I think it would be quite an interesting topic for an article if one doesn't exist already. I've tried looking but all I get is age of consent stuff which isn't exactly relevant.--J. F. Mam J. Jason Dee (talk) 02:00, 27 December 2008 (UTC)[reply]

I am familiar with JJASONDJFMAM, the months starting with June, but not with this editor. (Was there ever a DJ named J. Jason who was on both FM and AM? Did he live under a bridge?) Edison (talk) 02:09, 27 December 2008 (UTC)[reply]
Are you going to answer my question, or am I going to have to wait for someone else to do it? Are you going to continue to violate WP:BITE, WP:AGF and WP:NPA?--J. F. Mam J. Jason Dee (talk) 03:02, 27 December 2008 (UTC)[reply]
(DJ... Jason... AM/FM... Wha? Either I'm not getting something, or someone just said something really stupid.) After a thorough review of Template:Mental and behavioral disorders and mental retardation, I can't find anything on it. flaminglawyerc 03:26, 27 December 2008 (UTC)[reply]
I suppose this may have some information but I can't find anything about it in Wikipedia.--J. F. Mam J. Jason Dee (talk) 03:36, 27 December 2008 (UTC)[reply]
This ties into Informed consent#Sex as well. Exxolon (talk) 03:45, 27 December 2008 (UTC)[reply]
My post noted that someone who wants an article about having sex with people who are mentally retarded might just possibly be trolling. The articles you have edited started 3 days ago with Rings of Uranus and Anal Cunt , and your edits did not appear to be those of a new editor, given familiarity with redirects on your second edit, and an AFD nomination on your fifth edit. Edison (talk) 04:10, 27 December 2008 (UTC)[reply]
Can we lose the 'troll' epithet please? Anyone can rationally see that it in itself is a potentially legit encyclopedic topic, or at least a subsection of a legitimate topic somewhere. Just search for articles about it online.--J. F. Mam J. Jason Dee (talk) 04:54, 27 December 2008 (UTC)[reply]
You are welcome to do the editing in question, consistent with WP:RS, WP:NPOV, WP:V and other relevant guidelines or policies. The articles Human sexuality or Sexual abuse might be good places to cover having sex with persons who are mentally retarded, so that sex with them is like sex with a child, per your original question. Consider adding "Sexual abuse of retarded persons" to Sexual abuse. Edison (talk) 05:24, 27 December 2008 (UTC)[reply]
You already read age of consent you say? Maybe try reading it more carefully? In several places it says "In many jurisdictions, age of consent is interpreted to mean mental or functional age.[2] As a result, victims can be of any chronological age if their mental age is below the age of consent.[3]" Nil Einne (talk) 07:49, 27 December 2008 (UTC)[reply]
Incidentally, this was the topic of the movie The Other Sister. StuRat (talk) 08:25, 27 December 2008 (UTC)[reply]
Having a mental age of a minor does not mean that adults with mental retardation are like children in an adult's body. "Mental age" is a kind of measure for intelligence-as-tested-by-intelligence-tests. Adults with mental retardation have sexual feelings and can have sex just like anybody else. However, because of their low intelligence, they are more at risk both to be sexually abused, and to sexually abuse others. Lova Falk (talk) 10:46, 27 December 2008 (UTC)[reply]
And as for the original question, about the ethics of sexual activity with people with mental retardation: as long as they want it and enjoy it, as long as having sex is not used as a way to manipulate them, in short, as long as they are treated as you would treat anybody else, there is nothing unethical about it. Lova Falk (talk) 10:50, 27 December 2008 (UTC)[reply]
Dear Lova, This is exactly the sort of controversial personal opinion that this "question" was designed to elicit in hopes of sparking fruitless acrimonious debate. Soapbox stances have no place on the Refdesk, anyway. Even I am tempted to rebut. (The Other Sister is a great movie by the way.) --Milkbreath (talk) 15:47, 27 December 2008 (UTC)[reply]
Regardless of the original questioner’s motives in asking the question, this does seem like a perfectly reasonable subject for an article. --S.dedalus (talk) 09:20, 28 December 2008 (UTC)[reply]
I think the best thing to say is per our own article if one parties mental age is under the age of consent and the other party is an adult in all regards, and there is no closeness of age exemption, then it is illegal in a number of jurisdictionms. Whether you consider it unethical it or not is of course moot since the RD isn't the place for such discussions. Nil Einne (talk) 11:07, 28 December 2008 (UTC)[reply]

How Many US Businesses currently have a website

I have been looking for data on the numbers of US businesses that have a website, could you refer me to a good source of information? —Preceding unsigned comment added by 8.7.69.32 (talk) 03:04, 27 December 2008 (UTC)[reply]

<moved from WP:RD/C> flaminglawyerc 03:19, 27 December 2008 (UTC)[reply]
Extremely difficult number to ascertain, I'd imagine.--J. F. Mam J. Jason Dee (talk) 03:38, 27 December 2008 (UTC)[reply]
If you could get some other statistics about US businesses (number of businesses, no. of employees, turnover, profits, etc.) from the US equivalent of Companies house (would that be the SEC?), you could probably come up with some estimates like this:
  • Once a business grows beyond a certain size, it is almost compulsory these days to have a website, so suppose a business with more than 100 employees is 100% certain to have a website, and one with 50 employees is 50% certain to to have a website.
  • or, most if not all NYSE listed companies will have a website.
  • or, profitable companies can afford to employ a web consultant to design their website.
Astronaut (talk) 12:23, 27 December 2008 (UTC)[reply]

Day

What is the least talked about day of the year? 60.230.124.64 (talk) 12:15, 27 December 2008 (UTC)[reply]

My birthday to judge from the absence of cards and presents.90.9.82.48 (talk) 15:06, 27 December 2008 (UTC)DT[reply]

(lawl) (we see through your game, buddy. What's the least talked about day of the year? Why don't we talk about it so that it won't be the least talked about anymore? Wait a sec... You're the same person that asked about the states. <growl> flaminglawyerc 16:19, 27 December 2008 (UTC)[reply]
Well, can I have an answer? 60.230.124.64 (talk) 21:20, 29 December 2008 (UTC)[reply]
We have a page for each day of the year, and there's a list of notable people who were born or died on that day. It should be fairly easy (if a little tedious) to determine which day (apart from 29 February, which is often talked about out of proportion to its occurrence) has the least number of notable people associated with it. That might be a start. -- JackofOz (talk) 21:38, 29 December 2008 (UTC)[reply]
And now I can't help wondering what that day is, and why. Same for the day with the most people. Is there a quick way to count these, or is it as tedious a process as I imagine? -- JackofOz (talk) 21:35, 30 December 2008 (UTC)[reply]

How much snow is ONE INCH of rain?

Attention, ladies and gentlemen- If it snows, say, two inches does that mean it wouldve been a tiny bit of rain? By the same token does every inch of rain equal about 6 inches of snow?I'm Scared (talk) 13:11, 27 December 2008 (UTC)[reply]

According to our article new snow has a density of between 5% and 15% of water... Once the snow is on the ground, it will settle under its own weight (largely due to differential evaporation) until its density is approximately 30% of water. Increases in density above this initial compression occur primarily by melting and refreezing, caused by temperatures above freezing or by direct solar radiation. By late spring, snow densities typically reach a maximum of 50% of water. -hydnjo talk 15:14, 27 December 2008 (UTC)[reply]
And when compressed to ice, as in my driveway, it's around 90% the density of water. StuRat (talk) 18:22, 27 December 2008 (UTC)[reply]
Well then, that would be ice, not snow :) hydnjo talk 20:57, 27 December 2008 (UTC)[reply]
Yes, exactly - so one inch of rain would be between two inches of tightly compressed snow - and twenty inches of loose, fluffy snow - and two inches of snow would be between one inch and one tenth of an inch of rain. SteveBaker (talk) 15:55, 27 December 2008 (UTC)[reply]
The rule of thumb I've heard from the TV weathermen is that 10 inches of snowfall equals 1 inch of rain. (Always accompanied by a disclaimer saying it's highly dependent on the type of snow.) -- 128.104.112.113 (talk) 20:28, 28 December 2008 (UTC)[reply]
Very highly dependant. Around here, we've picked up almost five feet of snow in the past month, but the weather forecasters keep saying "It's only equivalent to three inches of rain, please don't kill yourself trying to shovel off your roof". --Carnildo (talk) 00:55, 1 January 2009 (UTC)[reply]
Wet snow is denser than powder snow. ~AH1(TCU) 18:29, 1 January 2009 (UTC)[reply]

Important events in 2008

can someone create a page on inmportant events of the year 2008, with the year coming to an end. It will be really helpful. Thanks in advanceKumar rvce (talk) 13:54, 27 December 2008 (UTC)[reply]

Have you tried putting 2008 in the Wikipedia search box? Happy 2009. Richard Avery (talk) 14:59, 27 December 2008 (UTC)[reply]

Cadillac Allante Headlamps

Headlamps work fine on high beam. I have no low beam. Checked bulbs and power to socket —Preceding unsigned comment added by 70.160.156.39 (talk) 13:59, 27 December 2008 (UTC)[reply]

Have you checked the fuse? Astronaut (talk) 14:41, 27 December 2008 (UTC)[reply]
You're saying that there is power to the socket into which the bulbs are plugged AND that the bulbs light when power is applied to them - but NEITHER of them work when plugged into the car?!?! That's impossible.
There are several ways that cars get high beams - and I don't know which way yours work - however, I suspect you have dual-filament bulbs - where each bulb has both a high-beam and a low-beam filament. That being the case, you might not be measuring the right thing - perhaps the high-beam filament is fine - but the low beam filament is blown. That's not such a surprising thing since most people use low-beams much more than high-beams. It would seem at first sight to be surprising that both headlamps would be blown - but since both bulbs most likely came off the same production line on the same day - were stored for the same amount of time under the same conditions - then subjected to the exact same on/off cycles - it'd not be such a surprise that they'd fail at close to the same time.
SteveBaker (talk) 15:50, 27 December 2008 (UTC)[reply]
Weeeeelll...not impossible. A high-impedance voltmeter would show a beautiful 12 volts across the socket with the bulb removed even though there was a high resistance in the line limiting current. You have to check the voltage with a good bulb in place. Look for corrosion at the contacts, although if both lamps are out that's a long shot. There are other even more unlikely situations that would cause this, also. I tried to Google for your schematic diagram to no avail, so I can't really help much except to say that they've been getting pretty tricky with xenon and LEDs and pulses in automotive lighting lately. If it was me, I'd get the Haynes Manual. --Milkbreath (talk) 17:37, 27 December 2008 (UTC)[reply]
The Allante ceased production in 1993 - it's at least 15 years old. I doubt it has anything other than filament bulbs - certainly they won't be LED's. SteveBaker (talk) 06:43, 28 December 2008 (UTC)[reply]
You also might have the wrong bulbs in the car, which might cause that behavior. We need to know some history on this car. Did you just buy it (used, of course) ? Did you just have the headlights replaced ? Did both low beams stop working at different times or at the same time ? StuRat (talk) 18:17, 27 December 2008 (UTC)[reply]
One suggestion: Until you get this fixed, fog lights, if present and operational, may be more acceptable to oncoming drivers than high beams. StuRat (talk) 18:19, 27 December 2008 (UTC)[reply]
Altruism aside, I wouldn't drive at night with only fog lights :( hydnjo talk 21:28, 27 December 2008 (UTC)[reply]
In a place with no street lights, I'd tend to agree. However, if there are street lights, then fog lights are sufficient to warn other vehicles of your location, without blinding them, and the lighting of the street, along with that from the street lights, should be sufficient so that you can see your way. StuRat (talk) 22:24, 27 December 2008 (UTC)[reply]

It's probably the switch. That also would explain the weird behavior that there is sometimes power and sometimes there isn't. God Bless America (talk) 20:00, 28 December 2008 (UTC)[reply]

I had this exact same problem a number of years ago and it was the switch. I just drove around with high beams on all the time for a couple days until I could get it fixed. Dismas|(talk) 04:29, 29 December 2008 (UTC)[reply]
And did you leave a trail of blinded oncoming drivers in the ditches ? There was probably a cop who was going to pull you over for this, but he was blinded and ended up in a ditch, too. :-) StuRat (talk) 20:01, 29 December 2008 (UTC)[reply]

Toyota Camry low frequency noise

While underway in my 2006 Camry, there is a pronounced low frequency rumble emanating from either the left front or left rear side. A sharp right turn of the steering wheel makes this more pronounced.

Dealer suggested I rotate the tires, which were cupped. Instead I replaced all four, with new. No change. Any suggestions? Thanks for any and all help! --71.98.16.211 (talk) 19:44, 27 December 2008 (UTC)[reply]

Could the tires be rubbing against the wheel well ? This is sometimes a problem with oversized tires, especially if new. StuRat (talk) 02:08, 28 December 2008 (UTC)[reply]
Additionaly might be worth your while to check your tyre balance (especially if there is a pronounced vibration at a specific speed), your wheel alignment (tracking), your disc brakes and brake caliper adjustments, your shock absorbers and your constant velocity joints (especially the rubber 'boot' that protects them - deflated boots can allow dirt to enter causing rapid wear. Exxolon (talk) 02:42, 28 December 2008 (UTC)[reply]
Doubtful, but possible; is one of the windows partly open when this happens? And does the problem go away when the window is open or closed? Sometimes the wind blowing across the open window will cause the same effect as blowing across an open bottle, except that the frequency is very much lower. Bunthorne (talk) 04:01, 29 December 2008 (UTC)[reply]

Ages of Greek Mythological figures

Hi. I'm writing a story that takes place at some point after the Oedipus trilogy. Which of these characters would still be alive at this point?

  • Creon of Thebes
  • Adrastus of Argos
  • Laertes of Ithaca
  • Theseus of Athens
  • Heracles
  • Mecistus of Argos
  • Phoroneus of Argive

And are there any other mythological figures known for strength in a specific skill? thanks User:Ye Olde Luke) 21:48, 27 December 2008 (UTC)[reply]

Presumably you have already read Creon, Adrastus, Laertes, Theseus, Heracles, Mecisteus and Phoroneus? I'm hardly an expert on the subject matter, but it seems to me that it's not at all unusual for many mythological characters like this to never die -- which is not to say that they are necessarily immortal, just that their death is never discussed or described. To answer your second question... yeah. Lots. You'll be sure to find many, many more characters known for their prowess in their areas of expertise in Greek mythology alone. -- Captain Disdain (talk) 23:59, 27 December 2008 (UTC)[reply]
The second question first: well, yes, that's what polytheism is all about, the gods specialize in specific skills. Zeus, for instance, is really bitchin' at throwing lightning-bolts and Hephaistos is one hell of a blacksmith.
As for mythological charactars who are not gods, then yes, lots of them have specific skills too. Obviously, there's a metric butt-load of great warriors (Achilles, Hector, Theseus, Heracles, etc.), but there are lots of others. Daidalos was a great inventor. Tiresias was a sooth-sayer. Medea was a witch. Orpheus could play great music. Helen of Troy made men drool and betray their countries and stuff. Any special skill you're looking for?
As for the second question, well, lets see: Creon succeeded Oedipus, so he'd be alive after. Adrastus was part of the whole Seven against Thebes thing, which takes place after Oedipus Rex, so he'd be alive back them. Same thing with Theseus, he was also part of Seven Against Thebes. Laertes is harder, he's sort-of in a different time-line, isn't he? Playing six degrees of greek mythology, we can assume that he was: Laertes was an Argonaut, meaning that he lived at the same time as Jason, who lived at the same time as Medea, who lived at the same time as Theseus (after killing her children and those other folk, she fled to Athens and king Aegeus and continued her shenanigans there), and Theseus, as we've already established, was alive after Oedipus Rex. It's dodgy, pretty much every hero mentioned in greek mythology is claimed to have been an argonaut at some place or other. But you could certainly get away with it ("artistic license", and all that). Heracles was an argonaut too, so same thing with him. Mecisteus was part of Seven against Thebes, so he'd be alive after Oedipus Rex too (the Illiad also mention him being part of Oedipus' funeral). Phoroneus I'd never heard about, but according to the article, he seems to be some sort of primordial ur-man (great grand-son of Cronos and Gaia, in fact), so he shouldn't have been alive, I think. Well, that's it. Belisarius (talk) 19:57, 28 December 2008 (UTC)[reply]
Aww, shite, I misread your question! You're not asking of people alive after Oedipus Rex, you're asking for people alive after the Oedipus trilogy. Well, Antigone is the last of the the Oedipus triology, and it takes place after Seven against Thebes. So who survived it? Well, Creon did, so he's still in. I think we can safely assume Theseus would be alive as well. Heracles and Laertes is anybody's guess (so if you want to include them, go ahead). Adrastus was alive after Antigone, because he went back to war against Thebes ten years later. I have no idea about Mecisteus and as before, I'm guessing Phoroneus had been dead a long time. So, same answer as before :)
BTW, can I ask something? It seems to me like you put the names of a bunch of greek heroes on slips of paper and put them in a hat, and then dragged seven out. Is there anything special about these guys? Is there any significance to the number seven? Belisarius (talk) 20:10, 28 December 2008 (UTC)[reply]
The only significance is that each seemed to have a specific skill, not counting Heracles and Phoroneus, who serve a different role in the story. Laertes is mentioned in a hunt for the Bear, Theseus defeated Medusa (so swordfighting), Adrastus is a good horseman, Mecisteus is described by Homer as defeated everyone from Cadmus in boxing. After your comment, I've decided to remove Phonoreus and have both roles played by Heracles, and add Paris of Troy for archery. Is that possible? I ask only because Paris is from the Trojan War, so he might be a bit young, but Oddyseus's father was alive during the timeline of my story, and Oddyseus lived at the same time of Paris, so I figured Paris was probably born in time. What do you think? --Ye Olde Luke (talk) 05:19, 29 December 2008 (UTC)[reply]
Sounds like fun to me! I must say though, I'm not all that familiar with many of the stories featuring the characters you name, but why the hell not! And if you're going to allow Trojan heroes, why not substitute Hector for Adrastus for the horse-taming skill? Homer refers to him as "Hector, the Horse-tamer" (now that's an awesome nickname!), so that would be appropriate, I think (personally, I'd find any excuse to put Hector in a story, he's like the coolest guy in The Illiad). Paris makes a fine archer, can't really think of any Greek hero that's famous for that except him. As for being too young, I don't really think that's a huge issue. Greek mythology is so "fractured" into tiny different sub-groups with their own time-lines, quite independent of each other, and you can only make the most tenous of connections. Like, the Ithaca stories about Jason and Medea, the Athens stories with Theseus, the Corinth stories following Oedipus, the Homeric stories, etc. The connections are extremely vague ("Some dude said Laertes was an Argonaut..."), so you can absolutely take some artistic liberties with that. It's definitely possible that Paris was alive in the time after the Oedipus triology.
Also, I really think you should include Daidalos (or Daedalus, depending on how you spell it)! Every story needs a MacGyver type problem solver that can quickly invent a gadget or two! And he fits in the story-line, he was contemporaneous with Theseus. Patron saint of engineers, that fella!
BTW, I don't know if it was a typo or not, but Theseus did not kill Medusa, he killed the Minotaur (and a number of other strange beasts). That involved sword-fighting too, I guess. Although mostly it was all about seducing princesses to find out the secret of the labyrinth and then abandoning them on beaches :P. It was Perseus killed Medusa, and used her cut-off head to turn the giant Atlas (he who held up the sky) to stone, thus creating the Atlas mountains (I'm not sure that part of story is all that canonical, but someone told it to me when I was a child, and it's remained one of my favorites).
Good luck with your story! Belisarius (talk) 23:33, 29 December 2008 (UTC)[reply]
Thanks for your help! And I totally agree about Hector, rooted for him and Troy right up to the end :( --Ye Olde Luke (talk) 02:04, 30 December 2008 (UTC)[reply]

December 28

birds

birds fly south in a season, and make nests, eggs ect then fly north for the other 6 months so does this mean that most birds breed twice every year, and live thier lives in eternal summer? —Preceding unsigned comment added by 82.3.145.61 (talk) 02:48, 28 December 2008 (UTC)[reply]

No, because most of them don't cross the tropics; their winter and summer territories often overlap. —Tamfang (talk) 04:56, 28 December 2008 (UTC)[reply]

i dont understand please elaborate —Preceding unsigned comment added by 82.3.145.61 (talk) 15:45, 28 December 2008 (UTC)[reply]

Take the American Robin as an example of a typical migratory bird. This map is a picture of the robin's range. The robin spends the summers in the yellow and green areas (Canada and the US), nesting and laying eggs. When winter comes, the robins fly south, to the blue and green areas (Mexico and the southern US). They don't go all the way to the southern hemisphere; the southern most robins stop in Central America. Robins only breed in April to July, so during the winter they're just "hanging out", fattening themselves up on the food available in the relative warmth of Mexico. Note, however, the large green area of year-round occupancy, which extends into Minnesota and Montana - hardly "eternal summer" there. For more examples, take a look at Bird_migration#Long-distance_migration - most birds don't migrate very far. There are some, however, which do swap hemispheres. The Bar-tailed Godwit, for example, migrates from Alaska to New Zealand each year. Note, however, that they only breed in the Northern Hemisphere, and don't lay eggs when they're in the Southern Hemisphere. -- 128.104.112.113 (talk) 20:23, 28 December 2008 (UTC)[reply]
That map is out of date. Robins have been extending their winter range northward, to the point where they are now overwintering in eastern Massachusetts, something I don't recall seeing even ten years ago. My understanding is that the robins that winter here are not the same ones that live here in the summer. In the spring, the winter robins will fly north into Canada and be replaced by robins who spent the winter farther south. So, the Canadian robins spend their life in eternal chilly weather: southern New England in winter, subarctic Canada in summer. Marco polo (talk) 03:42, 29 December 2008 (UTC)[reply]
I presume the OP means a bird such as the Swallow, which spends April - end August in the UK, then migrates to South Africa which means that it spends its whole life in summer? As far as I can recall they don't actually breed in South Africa but in the UK. --TammyMoet (talk) 10:30, 29 December 2008 (UTC)[reply]
To echo 128. the birds that come to the Arctic do so for the express purpose of breeding rather than breeding in the south. I believe that this is due to the lower number of predators. CambridgeBayWeather Have a gorilla 14:12, 29 December 2008 (UTC)[reply]

shitty xmas

i got no phone calls, presents or invites this year, is this normal as you get older, that people expect you to already have plans and people to give gifts to or is it just me, i only gave one present this year, and the person stood me up on the day anyway. is it normal for people to get nothing, not even a call from his mom? age 26 —Preceding unsigned comment added by 82.3.145.61 (talk) 02:54, 28 December 2008 (UTC)[reply]

First, sorry for the bad Christmas, and I hope it will get better in due time.
Second, to answer your question, it kinda depends where you're from. In the Philippines, where all aspects of Christmas form a very strong (and sadly rigid) tradition, people don't expect gifts from you when you're young, but as you grow older, and especially as you become an adult, there will come a time where you will need to have plans and need to give gifts as you become less and less dependent on your parents. I can't really answer for other places (except maybe the U.S., but I'm not so sure about that), but hope this helps. --Sky Harbor (talk) 03:44, 28 December 2008 (UTC)[reply]
Might I suggest you take the lead ? Call your Mom. Invite people over to your place for dinner. Put up a tree and decorations. Give out gifts. It's too late for Xmas this year, but how about doing this for New Year's Day. That's exactly what I did. You will find that people reciprocate. On the other hand, if you never do this type of thing, they likely think you're not the type who cares about Xmas traditions, so leave you out of their plans. StuRat (talk) 06:02, 28 December 2008 (UTC)[reply]
That likewise depends on the circumstances. Just because you don't have your own Christmas party or give people presents does not necessarily mean that you don't care about Christmas in general. If there was (or if you're in) some extraordinary circumstance, then people may understand why you don't celebrate Christmas, and they may console by including you in their celebrations. However, if you feel you're being excluded, then take the lead and include yourself in the season by celebrating it yourself, and don't be afraid to invite others so they can join in the fun. --Sky Harbor (talk) 06:30, 28 December 2008 (UTC)[reply]
I think Christmas does get worse as one gets older. For me, bad things happen around Christmas - terminal illness of family members, car crashes, finding neighbours dead, you get the picture - and so each year it's a real effort to put decorations up and buy presents. But it's really important that the effort is made. I'll leave the "why?" for you to decide.
What you might want to consider next year is offering your services as a volunteer to one of the shelters for homeless people in your area. There's nothing like a bit of selfless giving to make you appreciate what you have a bit more. If I hadn't had the UK's cold this year I might have done so myself, family or no family. --TammyMoet (talk) 09:41, 28 December 2008 (UTC)[reply]
Why are you expecting a call from your mother? Why are you waiting for invites? It seems tome your problem is you're waiting for others when you should be acting yourself. Call your mother. Ask people what they're doing for christmas or even consider throwing something yourself. I think the one thing that is normal is that as you get older, you do have to take greater responsibility for your life but really this should have began before 26. Maybe your mother thinks your too busy for her and so do your friends because your waiting for them? Nil Einne (talk) 11:03, 28 December 2008 (UTC)[reply]
Maybe he's afraid to take the lead for fear of failing to impress his peers or family. There are many reasons as to why someone can fail to act. It's all a matter of perception. --Sky Harbor (talk) 14:08, 28 December 2008 (UTC)[reply]

this is th OP, i did make plans, but when it came to the day, the people i had plans with stood me up and went to see other friends instead. i did not get a single card. real nice, boosts my view of humanity. all i need now is a kick in the balls —Preceding unsigned comment added by 82.3.145.61 (talk) 15:47, 28 December 2008 (UTC)[reply]

Ouch! That must really hurt. Anyway, were they aware that you had plans? Did it conflict with their prior commitments? Maybe they forgot. Regardless, don't blame yourself. Yeah, as you grow older, Christmas begins to suck big-time, but make the best out of it. Think positive. Try talking to them about it or something. --Sky Harbor (talk) 16:04, 28 December 2008 (UTC)[reply]

we had plans since late Nov, but then on boxing day they phoned me ans said oh, yeah sorry, but "bob" called and i went there instead, oh thanks for the present, sorry we didnt get you anything. its not that i want presents, i just want to know that some one loves me and cares. well, you know what they say, suicide is an option, and then they'll regret leaving me out of all the fun, 1 i hide my depression well, 2 a few different groups of my friends got together and even though they knew i was alone watching tv, eating a meal for one, they did not invite me. or even wish me happy xmas. sothis might be waht they call a cry for help, but fuck em theyll, regret it when i am dead, if they notice.

Uhh...well, this will be hard to solve. It seems that you were stood up, and that seems quite complicated to solve. I would normally invoke "forgive and forget" in this case, since if you reminded them about your plans, they should feel a little bit of guilt. Please do not kil yourself to make a point. By the way, Wikipedia isn't a suicide hotline (or is it?) ;) --Sky Harbor (talk) 17:38, 28 December 2008 (UTC)[reply]
If you're depressed to the point of being suicidal, seek medical attention. (I originally wrote a longer response to this, but I deleted it without posting it, because, frankly, the chances of you taking it the wrong way are astronomical. Seriously, call a doctor. I can't guarantee that it'll fix everything, but if you really are that depressed, I can pretty much guarantee that it's not just gonna sort itself out.) -- Captain Disdain (talk) 03:03, 29 December 2008 (UTC)[reply]
It might merely be a statistical coincidence that they all canceled. Let's say any given guest (or family) has a 50% chance of canceling and you invited 3 families. If the chances of each canceling are an independent event, then that would give you a 0.53 or 0.125 chance that all 3 families will cancel. That's a 1 in 8 chance, so you could expect this to happen every 8 years, on average.
Now for some advice: after the last person/family canceled on you, try to get yourself invited to wherever they were going, or to one of the other events. Also, if you live near home, ask your Mom if she would mind one more guest. StuRat (talk) 11:35, 29 December 2008 (UTC)[reply]

The way I'd deal with this is to make heaps of plans, throw heaps of parties and don't invite any of the people who stood you up. This will allow you to cheer up and make new friends while making a point to the old ones about ditching you. The reason they stood you up probably wasn't personal. Some friends are shitty and only care about themselves and will drop everything with you when they think they've got a better offer. You don't need them, show them what it feels like to be ditched. --Candy-Panda (talk) 06:45, 30 December 2008 (UTC)[reply]

There certainly are valid reasons for missing an event you said you'd attend, like sickness. However, it sounds like your friends pretty much said "We were invited to another event we'd rather attend", which makes them disloyal friends who don't keep their promises. At least they're honest about it. Still, I agree that you should find some better friends. StuRat (talk) 16:13, 30 December 2008 (UTC)[reply]

did I see a fox?

Recently I saw an unfamiliar animal trot across the street here. It had short legs like a cat, a long thick black-tipped tail, and a face that (twenty yards away in dim light) looked to me like a coyote's. Are there foxes in these hills? —Tamfang (talk) 04:55, 28 December 2008 (UTC)[reply]

yeah, probably lives on the golf course —Preceding unsigned comment added by 82.3.145.61 (talk) 05:13, 28 December 2008 (UTC)[reply]
Quite possibly. They're more common than you might think, but normally keep hidden. StuRat (talk) 06:08, 28 December 2008 (UTC)[reply]
Almost certainly a Gray Fox. They control the bunny population. --jpgordon∇∆∇∆ 08:31, 28 December 2008 (UTC)[reply]
It must be working, because I haven't seen a Playboy Bunny in years. StuRat (talk) 14:28, 28 December 2008 (UTC)[reply]
But have you seen a foxy chick? —Tamfang (talk) 08:00, 29 December 2008 (UTC)[reply]
I saw one or two foxes while hiking when I lived in the Bay Area. So they definitely live there, especially in the open spaces. Gray fox looks right. Marco polo (talk) 03:39, 29 December 2008 (UTC)[reply]

Bigamy

OK, keep your hair on, this is NOT a request for legal advice, just a general question! How come the crime of bigamy is so universally enforced? It seems (to me anyway) that no matter where someone is married, no matter how far out of the way it is or how different the laws may be, bigamy still seems to apply if you become married again, even if it is a different country to the one where the original marriage took place. Is the state of marriage a "universal" concept of law? Is anyone aware of a country whose marriage laws would NOT apply in the case of bigamy? Thanks everyone!! 121.44.151.254 (talk) 10:28, 28 December 2008 (UTC) —Preceding unsigned comment added by 121.44.151.254 (talk) 10:27, 28 December 2008 (UTC)[reply]

You seem to be confusing two concepts here. If a country recognises another countries marriages and it has laws on bigamy then clearly those laws will come into play if someone tries to/does re-marry. International recognition of marriages and local bigamy laws are largely, AFAIK, seperate concepts. There may be some cases where bigamy comes into play even if the marriage is not locally recognised but I think this is rare. Nil Einne (talk) 11:00, 28 December 2008 (UTC)[reply]
Bigamy is not universally outlawed. For example, in certain Islamic countries (such as Saudi Arabia, the United Arab Emirates, and Malaysia) having multiple (official) wives may allowed, given certain restrictions. (I was hoping to find more details in the article Polygyny in Islam, such as the exact legal situation and the relevant limitations, but details on the official situation are rather slim.) If someone enters into a plural marriage where it is legal, and then moves to a location where it is not, our Polygamy#Legal situation article says "Polygamists may find it harder dealing with government agencies, such as obtaining legal immigrant status." I think that most countries have laws allowing them to deny entry to "people of poor moral character", and may view polygamists as such. If you are talking about marrying in country A, then moving (without your spouse) to country B and then trying to get married to a different person there, it's local laws that apply. Most countries recognize marriages performed in other countries (I'm not sure if there is a treaty which covers this). Thus if you get married elsewhere and move to the US (for example), the US usually recognizes the foreign marriage. Since in the eyes of the US you are already married, trying to get married to someone else falls afoul of the US bigamy laws. On the other hand, if you got married in the US and then moved to some place where polygamy is legal (e.g. Saudi Arabia/Malaysia), it is unlikely that Saudi Arabia/Malaysia would object to you marrying an additional spouse, provided that you met all the other conditions for polygamy in Saudi Arabia/Malaysia which may be hard for an immigrant to do. The fact that all types of polygamy are illegal in the US doesn't enter into it - it's the local laws which apply. Note, however, that if you then decided to go back to the US, the US may object to you being a polygamist on the grounds of US laws, even though your marriage was legal when and where you got it. -- 128.104.112.113 (talk) 20:06, 28 December 2008 (UTC)[reply]

Do I remember right that the bigamy law in Venezuela (or possibly Colombia) was repealed on the theory that bigamy is its own punishment? —Tamfang (talk) 22:48, 28 December 2008 (UTC)[reply]

Bigamy is the state of having one spouse too many. Monogamy is the same. BrainyBabe (talk) 12:17, 29 December 2008 (UTC)[reply]
Also, it is worth pointing out that Islam and certain other legal systems, under certain restrictions, allow polygyny. They do not, and very few places do, allow polyandry. You might get better info from one of the modern polyamory groups that exist. BrainyBabe (talk) 12:19, 29 December 2008 (UTC)[reply]

Borland C++

Hi,

Can i know how to get a print out of the executed file created by borland C++. What is the header file and the command? —Preceding unsigned comment added by 202.124.160.216 (talk) 14:05, 28 December 2008 (UTC)[reply]

  • Firstly, you're asking the question in the wrong section of the reference desk, the Computing desk would have been a better choice, and I suggest you add a note here that the question has been moved, and repost the question there.
  • Secondly, I suggest that you add more information when reposting. I've compiled many a C++ program, with Borland's compilers and others, but don't understand the question. Are you asking how to decompile a program that has been compiled with a Borland C++ compiler? Do you have access to the source code? What information do you have access to, and what information, in what format, is it that you are trying to obtain? --NorwegianBlue talk 21:48, 28 December 2008 (UTC)[reply]
I see that the question now has been asked on the computing desk, so further answers should go there. --NorwegianBlue talk 22:26, 29 December 2008 (UTC)[reply]

Unexplored coasts

What are several of the most unexplored coasts in the world that aren't under constantly cold conditions? Thankyou —Preceding unsigned comment added by 87.111.102.250 (talk) 14:22, 28 December 2008 (UTC)[reply]

Honestly, I seriously doubt that there are any. Coasts are where boats go and where cities are. If there were a non-cold coast that was unexplored, it would be ridiculously easy to explore it. Just go there with a dinghy or something. If you're counting rivers, I'm guessing some of the rivers connecting to the Amazon in South America. Ridiculous amounts of jungle. Belisarius (talk) 20:18, 28 December 2008 (UTC)[reply]
It depends what you mean by unexplored. Pretty much everything has been digitally mapped by satellite, but a lot of the world has not been visiting by anyone who has written a book about it. Can you count the 2004 Congo River journey by TIm Butcher as "exploration"? His book, Blood River (2007), reads like exploration. BrainyBabe (talk) 12:33, 29 December 2008 (UTC)[reply]
Many of the islands in the Arctic will have coasts that have never had boats sail around them. And while the Arctic is cold now, −39 °C (−38 °F) here, it can reach 20 °C (68 °F) and higher in the summer so it's not "constantly cold". CambridgeBayWeather Have a gorilla 13:58, 29 December 2008 (UTC)[reply]

camp fire

how do you "bank" a fire so that it will burn all night without being tended, yet have flames throughout to scare off wildlife. this theory can be seen in the earth's children series, but the technique is not discussed, please help —Preceding unsigned comment added by 82.3.145.61 (talk) 15:44, 28 December 2008 (UTC)[reply]

I'm not an expert, but I don't think this is possible. If I was trying to make a campfire stay alight overnight so that I could build it up the next morning without starting from scratch, I'd be trying to make sure there were *no* flames, just a little airflow to ensure it kept smouldering. A fire with large, animal-scaring flames is burning quickly, and will only keep going if fed with fuel. I hadn't heard of Earth's Children before, but the article on it starts "Earth's Children is a series of historical fiction novels". The bold emphasis is mine, and I think is the salient point here. PeteVerdon (talk) 22:05, 28 December 2008 (UTC)[reply]
As a purely practical matter, if you need a fire to keep away predators, then you should also either post a watch or be prepared to sleep very lightly, if at all. Feeding a fire is a good way for the posted watch to keep usefully busy and thus awake. We bank a wood stove overnight so that it provides embers in the morning and continuous, low heat all night. Flames in this case are the sign of improper banking as PeteVerdon has written above. ៛ Bielle (talk) 03:18, 29 December 2008 (UTC)[reply]
This method wouldn't protect the flames, but rather the heat (like Pete mentions above). I've seen the SurvivorMan Les Stroud use dried cow patties (cow poop) to capture the heat and keep smoldering all night. If you kept some dried grasses nearby (not TOO nearby) you could probably get the fire going in less than 20 seconds. It wouldn't be a constant guard against animals though, just a one time "scare them away!" tool NByz (talk) 10:16, 29 December 2008 (UTC)[reply]
Large logs will smolder for long periods of time, so use one of those then restart the flame by adding on grass or pine straw to rebuild the fire proper the next morning.--droptone (talk) 17:46, 29 December 2008 (UTC)[reply]
I agree with the previous answers that a fire with large flames will not last all night, untended. However, I can think fo two exceptions:
1) A truly massive fire. There the oxygen is used up in the perimeter flames and the interior can't burn up until later.
2) An automatic continuous fuel supply. This would work best with a liquid, such as oil, which is fed by gravity. It could pour out of a small nonflammable metal tube into a nonflammable bowl. StuRat (talk) 20:37, 29 December 2008 (UTC)[reply]
Try a lantern. Phil_burnstein (talk) 05:30, 1 January 2009 (UTC)[reply]

Hi, I'm looking for a list of films that have fallen out of copyright or high quality torrents for them. Everything I found on google leads to nowhere. Thank you. Keria (talk) 16:08, 28 December 2008 (UTC)[reply]

There is a ton of stuff at the Internet Archive. And, we even have an article on it. --Milkbreath (talk) 16:18, 28 December 2008 (UTC)[reply]

Guitar Hero World Tour drum kit

Does anyone know if you can learn to play drums (at least to some degree) with the Guitar Hero World Tour drum kit? When I say 'learn to play drums' I don't mean become the next Phil Collins or John Bohnam, but have some real drumming skills such as timing or something? BTW, I'm referring to the Guitar Hero drum kit and not the Rock Band drum kit because the Guitar Hero drum kit seems more realistic since it has cymbals. 67.184.14.87 (talk) 16:54, 28 December 2008 (UTC)[reply]

yes, this is basically a beginner electric drum kit and can teach you the basics. but dont expect much, it will take practice, skill, technique and a proper drum kit to be even moderately decent.

You can learn about staying on a rhythm using a digital tool like that for sure. This skill is universally useful for any instrument. And the drums are the one instrument in that game that make it almost possible for each "hit of the drum" to actually correspond 1:1 to a "note" in the song (at least compared to a 'basic' drumset - most professional drummers will have more than four things to strike). This means that - if you learn to play without watching the screen, just listening to the music, pretending each pad represents a specific drum or cymbal - you can improve your memory. I think most people learn to play those games by responding to visual cues, not memory cues. These are completely different processes.
As for things that it wouldn't help with:
1) Bandleading: The drums are expected to lead the rhythm of the band, with the bass, rhythm and lead instruments following (in that order). Playing along with a game like that will teach you to follow, not to lead.
2) Dynamics and Quality: Dynamics means controlling how "loud" you are, and by Quality, I mean controlling how good each strike sounds. This is especially difficult on the big cymbals and requires that you accustom yourself to hitting each type of drum/cymbal differently. The game won't help with this (and may actually give you the bad habit of hitting each note in the exact same place, with the same part of the drumstick, and with the same velocity)
3) Playing Quickly: The pads just don't have the response of an analog instrument. You'll never be able to practice quick runs on the same drum.
4) Improvising: This is where new music ideas come from. Later on, when you jam with friends, they'll expect you to come up with your own patterns and respond to their ideas. The game doesn't help with this.
5) The high hat: There is a special skill to getting used to how the high-hat operates, and getting good at including it in drum patterns involving the snare. I notice that the world tour drum kit only has one foot pedal.
Now for my personal opinion: These games exist to help people avoid the frustration and time commitment of learning a real instrument. They don't act as a very good substitute for practicing. The primary skill that they develop is around pressing a button when something on the screen happens. My friends that are good at - and enjoy - these games tend to be the same people who enjoyed the other arcade-style console games (like "fusion frenzy") that, for the most part, use basic skills like this. There isn't very much in music that uses this skill, except maybe sight-reading musical notation. Even then, you have to develop the instrument-specific skills first.NByz (talk) 10:11, 29 December 2008 (UTC)[reply]
The Rock Band drumset is actually a good deal more realistic than the GH one, despite having less pads. For instance, when playing the disco drum beat in Everlong, a real drummer would play the high-hat seven times and the snare once, and on any normal right-handed drum set that would entail going from the far left to the middle left. It's naturally more comfortable for ninety percent of the population and is the easiest way to play it. In World Tour, it goes from the middle left to the far left, which is backwards and awkward to play. You're expected to lead with your left hand and bring one stick down and to the far left somehow to hit the snare, and it just doesn't feel "right". Neversoft has never gotten note charting right, and Harmonix is especially good at retaining how a song was originally played (I can do the bass to Everlong in RB2 with my eyes closed despite having only seen it once or twice because it's actually the same pattern as real bass). As for teaching you how to play drums in general, yes, music games can do that to an extent... It certainly helps staying in time and understanding the basic concept of a drum beat. Heck, you can sometimes play along with songs and then transfer it over to real drums. It only really works if the drummer is using a pretty basic set, but it's possible.
As for the comment above mine, I disagree. One of the fundamental parts of music is understanding rhythm, and music games can really help with that. They're also good for developing dexterity and the physical prowess needed for the real thing. Obviously they don't teach you enough to be a real musician, but it definitely doesn't hurt. --69.146.230.243 (talk) 12:07, 29 December 2008 (UTC)[reply]
I'm the OP on a different computer. I already play piano and have tried guitar and I can say that playing guitar on Guitar Hero is nothing like the real thing. (When I tried learning real guitar, I found it was harder than I thought. You really have to press on the strings pretty hard, plus my fingers kept accidentally touch the other strings.) I only play Guitar Hero on the easy mode because with all that time and effort it would take to play on advanced mode, I might as well learn to play a real guitar. One comment I have is that I agree with "The primary skill that they develop is around pressing a button when something on the screen happens". When I play Guitar Hero, I have absolutely no idea what notes I'm playing or what chord the band is playing, and without that critical information, you really aren't learning anything. I'm simply reacting to the buttons on the screen. I'll look into Rock Band's drumset more closely. 216.239.234.196 (talk) 17:33, 29 December 2008 (UTC)[reply]
Would something like this [19] help in learning to play drums? 216.239.234.196 (talk) 20:59, 29 December 2008 (UTC)[reply]

Who's in the wrong? And how to solve the problem.

My sister got married to a Jewish guy. Great. No problem there. But, I am South African, so although he and his family know me very little, they know my sister well, and I would hope they would expect our values and morals to be similar, and they are, being siblings. But they assume I am racist just because of my background, or some other reason, and actively say things to me when she is not around, such as: I happened to get a hair cut in a neat, manageable military style, and one of them said to me I look like an SS soldier and that I should fuck off back to Germany. I have never been to Germany, nor am I a Nazi. On a seperate occasion I was reading Dora the explorer to my nephew, getting him to count the number of trees in the Brazilian rain forest picture, for instance, and when Dora got to Africa one of them asked me, as if i was the child being read to: "How many Niggers are there?" But they are very sneaky about this and always do it when my sister is not around or change the subject entirely as soon as she appears, they also stick together, so even when I do try to say something, they all deny it happened, or openly accuse me of being a Nazi, saying that they would never use the N word in ANY context because they are Jewish and know more about racism than I (I doubt this as I am South African) So, is it just me, should i change what i wear, and how i look, fake a different accent? Change my religion? I am not racist, and have other friends who are Jewish, and of many other diverse cultures. I don't mean to sound antisemitic, and don't want to be, but they are ruining my relationship with my sister, who due to only hearing my side and their lies, seems to think it is me who is in the wrong. Or am I actually in the wrong? Should I join White Pride World Wide? Or just sit back and take it? Cut ties with my sister and her beautiful children? Once again, I am NOT antisemitic but if this continues I may soon start to understand what Hitler was talking about. I have also tried sitting down and calmly telling them of my beliefs and mortals and that I could be a wonderful addition (if removed by marriage) to their family if given the chance to. But they will not even speak to me on any one subject without calling another member to help insult me. It is not my fault that I was born in a country with racist laws, in fact I helped in a small way to reform my counties ways. Why do they hate me? They treat my mother in the same way saying things like: "Let me hold the baby, you will just drop it, in fact, we don't want you holding OUR baby at all."

It isn't anything to do with their being Jewish, there is a uniform percentage of eejits everywhere. I'm afraid you'll just have to confront it. But a tape recorder and hide it recording everything. Do not try leading them into anything, avoid it if possible. Take the machine away afterwards and listen to it. If what is being said is unacceptable to you play it to your sister. This is not a nice thing to do but it will ensure it is all out in the open and has to be dealt with. Dmcq (talk) 19:07, 28 December 2008 (UTC)[reply]
Yeah, I agree, this has nothing to do with being from south africa or being jewish, this is all about them being assholes. The way I see it, you have three options: take it, or confront them. They're not going to stop of their own volition, so either you sit down and take it for the rest of your life, or you say to them "Hey, listen up! I'm not a racist, I've never been a racist, I've never looked down on anyone because of their ancestry. I will not sit here idly while you imply that I do. In fact, it's you people who are looking down on me because of my ancestry. Knock it off."
If it was me, I'd just cut them out of my life. These people are poisonous. Continue your relationship with your sister and her husband, but don't attend functions with those other people if you can avoid it. You don't need it. Belisarius (talk) 20:25, 28 December 2008 (UTC)[reply]
In your shoes I'd say quietly: "I've never considered myself Antisemitic, but if you keep up your efforts I just might. Is that what you want?" —Tamfang (talk) 22:56, 28 December 2008 (UTC)[reply]
The family's Jewishness is really irrelevant to the situation. It is a fairly common thing, in my experience over the years, for one marriage partner (and or his or her family) to try estrange the other marriage partner from his or her family. What happens is that one side of the marriage wants their family to have exclusive rights to the couple and their offspring. Sadly, there may not be much you can do about it. You should let your sister know what you have experienced, but she may be under pressure from her spouse to side with his family or to "choose between" her spouse and her family of origin. You may need to give up on finding acceptance from your sister's in-laws and resign yourself to less contact with your sister. I must say that I think that Tamfang's proposed comment would probably only make matters worse. Commenting on the family's Jewishness would only strengthen their case that you are a racist. Marco polo (talk) 03:33, 29 December 2008 (UTC)[reply]
Er, as I read it they're rubbing his nose in their Jewishness, so he wouldn't be introducing the subject. Though on second thought I like Belisarii angle better, perhaps with a twist: "Given how much ethnic persecution seems to be on your mind, I'd think you'd be a little less quick with the stereotypes." —Tamfang (talk) 08:06, 29 December 2008 (UTC)[reply]
Has your brother-in-law ever said anything or is it just his family. If he's not said anything then is it possible a quiet talk with him might help? What about the rest of your family have they ever said anything to them as well? CambridgeBayWeather Have a gorilla 13:50, 29 December 2008 (UTC)[reply]
And if it does come down to fighting it out, find out what their position is on Palestinians. Chances are they have some rather racist attitudes towards that group. If so, whenever they claim you are a racist for being from SA, just bring that up. Hopefully they will see this coming next time and not accuse you of being a racist. StuRat (talk) 20:27, 29 December 2008 (UTC)[reply]
Dispute resolution Conflict resolution. If I may interject some personal opinion, it is the natural tendency to want to "win" a conflict - this is often counter-productive, unless you intend to dominate the other people (a difficult and tenuous proposition at best). Instead, intentionally "lose" conflicts and gird yourself for loss - there will be some poetic justice in this, as it is the traditional Jewish way - to endure. After all, eventually, Yahweh redeems his chosen. Alternatively, you could engage them on Judaism - the mitzvah article may be fruitful ground for things to inquire about. 98.169.163.20 (talk) 07:25, 30 December 2008 (UTC)[reply]

WAR

What are the implications if India decides to invade pakistan and there is a war.does it ensures the world would than be divided into two, one with pak(islamic world maybe) and the rest against them.would this probably lead to a third world war? —Preceding unsigned comment added by 203.122.36.6 (talk) 17:29, 28 December 2008 (UTC)[reply]

Well, we're not really a crystal ball, but world war three will likely be the last world war, because it will be fought with nukes, and after that there won't be anyone left to fight with. Would an India/Pakistan conflict touch off a world war? Who knows. Either way, we should hope it never comes to that. -mattbuck (Talk) 17:49, 28 December 2008 (UTC)[reply]
I would say that the risk of that happening is vanishingly small. Quite right, a world war involving nukes (and more than one country holding them) would very possibly wreak havoc on humanity, possibly to the point of extinction. That's the bad part. But you are forgetting the good part! You know, MAD! Neither India or Pakistan are insane enough to even consider using nukes against the other, and they would extremely unwilling to even go into a full-scale conventional war because of them. And if they did, the entire world would do everything in their power to intervene and stop it before it goes to far. Every other super-power (the US, China, Russia, the EU) would do everything in their power to stop it from happening. The whole "humanity-is-coming-to-an-end-like-tomorrow"-thing tends to get people on the same page. Belisarius (talk) 19:24, 28 December 2008 (UTC)[reply]
That reminds me of a quote from Albert Einstein. He was writing to President Truman, expressing his concerns about warfare with nuclear weapons:
I do not know with what weapons World War III will be fought, but World War IV will be fought with sticks and stones. -- (From wikiquote).
It's very difficult to predict the outcome of a political process, and even more difficult to predict the outcome of a war. The volunteers here at the Reference Desk are no more qualified to answer your question than anyone else; though I imagine that someone can probably find some apocalyptic science fiction writing that would be on point. TenOfAllTrades(talk) 21:14, 28 December 2008 (UTC)[reply]
I can't see why many other countries would want to become involved. The United States and United Kingdom have tried to maintain good relations and even to form alliances with both India and Pakistan. Surely they and the European Union, whose main concern is trade relations, would work to defuse the situation and to get Pakistan and India to negotiate peace. Neither Russia nor China would have much to gain by supporting either party. Russia does not want more instability in the Islamic lands on its southern doorstep. China has sometimes supported Pakistan in the past as a counterbalance to its rival, India, but in the end would probably prefer for India to remain a viable threat to Pakistan to serve as a focus for Islamic militants in South Asia, since these might otherwise redirect their efforts to opposing Chinese rule in historically Muslim East Turkistan (Xinjiang). It is really hard to imagine most of the world lining up behind either India or Pakistan rather than urging both countries to end any conflict. Marco polo (talk) 03:23, 29 December 2008 (UTC)[reply]
When two nuclear powers hate each other, like the US and former Soviet Union, you rarely get a hot war between the two, but rather a cold war with an occasional proxy war in a smaller nation, such as the Bangladesh war for independence. That proxy war happened before they were nuclear powers, but I'd expect similar proxy wars in the future, perhaps in Sri Lanka. StuRat (talk) 20:16, 29 December 2008 (UTC)[reply]
"You rarely get a hot war"—I think you've got a pretty low sample size, no? I mean, the Cold War is sort of n=1, no? --98.217.8.46 (talk) 17:58, 30 December 2008 (UTC)[reply]
Well, Pakistan and India have both had nukes for a few years now with no hot war. Pakistan and Israel could use nukes on each other, but don't. Same for China and the Soviet Union, who were once enemies, and China and the US, similarly. Other pairs of traditional enemies also have nukes, but have not had a war between them since this became the case. StuRat (talk) 04:04, 31 December 2008 (UTC)[reply]
Both nations obviously realise that using a nuclear weapon unilaterally against the other would be a "game over" kind of event for them. The condemnation from the rest of the world - and the possible retaliation would be utterly crippling to them. Nuclear weapons are effective against large ground targets - but countries like the major European nations, the US and others could quite safely pound them into the dark ages with conventional bombing from the air from bases far beyond the range of the missiles those countries own. You can't take down an F18 with a half-kiloton bomb! That kind of retaliation for the unilateral deployment of a nuclear weapon would be rather likely - if only in the minds of possible perpetrators. The only possible way either nation could benefit from a nuclear weapon launch would be if they were totally overrun - their military in tatters with no chance of continuing - then it might be a viable 'last ditch' weapon. So the effect of these weapons is to deter full-scale conventional war. Nobody with half a brain is going to nuke a city over border skirmishes and risk having one of their own cities nuked in retaliation.
However, I could imagine (say) North Korea using a nuke - their government are quite crazy - they care absolutely nothing whatever for their people and might well stay in a nice comfy bunker and launch one for some kind of muscle-flexing kind of reason - but India and Pakistan are just not that kind of country. What's much more likely is a gradual escalation into larger and larger non-nuclear confrontation - from which both sides must ultimately back down without really achieving very much. SteveBaker (talk) 00:44, 1 January 2009 (UTC)[reply]

missing members from Basketball Hall of Fame

Hi, I just linked to the list of members from today's main page, and was astounded to find people like Michael Jordan missing. Am I missing the point here? Is there more than one hall of fame? Will MJ eventually make the list? Sandman30s (talk) 18:13, 28 December 2008 (UTC)[reply]

According to Basketball Hall of Fame, "to be considered for induction by a screening committee, a player must be fully retired from play for at least five years." By my calculations, that means Jordan should be eligible for induction next year. Deor (talk) 18:34, 28 December 2008 (UTC)[reply]

Modifying Facial Structure

Assuming no external injuries, is one's facial structure entirely determined by genetics? When one performs weight-bearing exercises, the muscle tugs on the bones and stimulates them to grow and strengthen. Can the same apply to the bones in the face? If one smiles a lot, will that alter one's facial structure as one ages compared to if one did not smile? Acceptable (talk) 21:06, 28 December 2008 (UTC)[reply]

One's facial structure is surely determined by extra-genetic factors. Some diseases, acromegaly being the best example that comes to mind, have a large impact on facial structure, yet are only partially, if at all, genetically determined. --NorwegianBlue talk 21:28, 28 December 2008 (UTC)[reply]
Yeah, I would say that your facial structure could easily change over the years according to many factors, just not a great deal. You sometimes see identical twins that have developed slightly different features over time.91.111.67.44 (talk) 00:16, 29 December 2008 (UTC)[reply]
You do? Aside from physical damage, why would the features of identical twins develop differently over time, I wonder? Do you have a source or even a link to some photos on this? Thanks ៛ Bielle (talk) 06:11, 29 December 2008 (UTC)[reply]
Climatic conditions, large weight gain/loss and smoking are three things that I can think of that could cause identical twins to develop differently over time. CambridgeBayWeather Have a gorilla 13:40, 29 December 2008 (UTC)[reply]
This link looks at examples and has images. I admit they're not huge but I can imagine more extreme examples.91.111.67.44 (talk) 23:31, 29 December 2008 (UTC)[reply]

Cellphone's tap check by myself

Is there any way (especially simplest) to check whether my cellphone is tapped so that the tapper(s) wouldn't guess? 80.69.57.150 (talk) 22:11, 28 December 2008 (UTC)[reply]

No. Seriously, this just isn't possible. It's all in software (whether you're talking about someone listening in en-route or having modified your phone) and someone with the necessary access and skills to make modifications in order to listen can certainly make other modifications to cover their tracks. As a normal customer with a normal phone you have no chance of discovering such modifications. PeteVerdon (talk) 23:04, 28 December 2008 (UTC)[reply]
Unlike a land line, which requires a physical device to be placed on your phone-lines, and thus could be detectable with the right equipment, someone tapping your cell-phone can do so by something akin to Phone cloning, whereby they simply passively grab and decode the signals intended for your phone out of the air. Knowing that someone was listing in on your cell phone conversations would be exactly like knowing, when listening to your favorite radio station, that someone else out in the world just tuned in that station. Its an impossibility. --Jayron32.talk.contribs 04:10, 29 December 2008 (UTC)[reply]
Hmm but I suppose you could encrypt the signal somehow which would mean no one could decipher the message without a key... I guess that's not what he's asking though. TastyCakes (talk) 04:41, 29 December 2008 (UTC)[reply]

December 29

brown spit

Birthday

What is Tenma from School Rumble's birthday? 119.95.242.147 (talk) 01:43, 29 December 2008 (UTC)[reply]

According to ja:スクールランブルの登場人物#2年C組女子 it's November 30. And her blood type is B. -- BenRG (talk) 04:19, 29 December 2008 (UTC)[reply]

Hitch Hiking

I was wondering, has anyone ever worked out what is more dangerous statistically, hitchhiking or giving a hitchiker a ride? TastyCakes (talk) 04:35, 29 December 2008 (UTC)[reply]

I doubt that. First of all, I very much doubt that there are reliable and comprehensive statistics on this, because incidents where either the hitchhiker or the driver assault the other person probably don't get reported a lot of the time, and when they do, I doubt they show up in any statistics are hitchhiking-related crimes -- it'd just be an assault (or whatever the crime itself turns out to be). It'd be very difficult, if not impossible for anyone to collect statistics on those without examining each case to see if a hitchhiker was involved in it, and then it might not even show up in the paperwork.
Also, hitchhiking is considered dangerous not only because someone might stop with bad intentions, but because you might get hit by a car when you're standing on the side of a highway -- but those incidents certainly wouldn't show up as a hitchhiking-related statistic.
Personally, I think the dangers of hitchhiking are probably greatly exaggerated, as they usually are with things like this -- which is not to say that people shouldn't be careful whose cars they get into or who they pick up, of course. -- Captain Disdain (talk) 04:48, 29 December 2008 (UTC)[reply]
Probably giving a hitch-hiker a ride; he/she could be a killer or even- a vanishing hitchhiker! Narutolovehinata5 tccsdnew 06:17, 29 December 2008 (UTC)[reply]
I don't know, but hitchhiking is definitely much rarer than it was. When I was a student 25 years ago, I hitchhiked a lot (to save the train fare for more drinking money :-)). Later, when I first got a job and a car, I often picked up hitchhikers, usually for the company. I never had a problem; most drivers were pleasant enough and when I was driving, I had someone to talk to on a long trip. The only "rules" I adopted were to not say exactly where I was headed until we had chatted for a while - it gave me a chance to get out or dump the hitchhiker if I felt unconfortable; and to never get a lift from or pick up more than one person. However, all the hitchhikers seem to have disappeared. I only recall having seen one person hitching in the past 15 years or so. I think too many people are afraid of them being some kind of psycho-killer. Astronaut (talk) 16:40, 30 December 2008 (UTC)[reply]
I still see the odd hitcher here in the UK, and in theory I'd usually be happy to pick them up. However, perhaps because they're non-drivers, they often seem to stand in a stupid place where I have no chance to safely stop, like the middle of a motorway junction. The greater speed of traffic these days also means that by the time I've registered their presence, established that I'm in a generous mood and prepared to give them a lift, and checked the traffic, I'm so far down the road that it's not really possible. PeteVerdon (talk) 00:24, 31 December 2008 (UTC)[reply]

names of continents

hi in world totally how many continent are have?.and what are their names?. —Preceding unsigned comment added by Karganapathy (talkcontribs) 08:23, 29 December 2008 (UTC)[reply]

6 continents: Europe, Asia, America, Africa, Australia (or Australasia or Oceania), Antarctica. Simonschaim (talk) 08:32, 29 December 2008 (UTC)[reply]

What I do like is how they all begin and end with the same letter (well the letter they start with). It seems to depend on when/where you were taught though. 194.221.133.226 (talk) 10:14, 29 December 2008 (UTC)[reply]
See Continent Dismas|(talk) 08:45, 29 December 2008 (UTC)[reply]
There are two areas of dispute:
1) Are North America and South America the same continent ? I say no to this, based on the isthmus (at the Panama Canal) between the two being only 50 miles wide and the two having distinct tectonic plates.
2) Are Europe and Asia the same continent ? I say yes, based on the isthmus (if you can even call something so huge an isthmus), being thousands of miles across and them sharing the same tectonic plate.
So, depending on how you answer those two Q's, you get from 5 to 7 continents. StuRat (talk) 11:22, 29 December 2008 (UTC)[reply]
There are more than two areas of dispute! Can Oceania be considered a continent? Not by continental geography. The Australian continent does not include the region of Oceania. Some of that is on Zealandia (which doesn't begin and end with the same letter, so perhaps we can discount it). And what of Madagascar? Greenland? Iceland? Gwinva (talk) 22:59, 29 December 2008 (UTC)[reply]
Oceania considered a continent ? On what basis ? The Pacific Plate is an oceanic plate, not a continental plate. Madagascar is part of the African Plate and Greenland is part of the North American Plate. Iceland is way too small to be considered a continent. If Zeelandia ever was a continent, it certainly isn't any more. StuRat (talk) 04:45, 30 December 2008 (UTC)[reply]
You're begging the question by assuming that tectonic plate boundaries are a key criteria for determining continents (it isn't once mentioned at Continent#Separation_of_continents). Is India is a separate continent, due to it being on a different plate from the rest of Asia? How about Central America, which is on the Caribbean Plate? Are Hokkaido and the Kamchatka Peninsula part of Asia or North America? (They're on the North American Plate.) Additionally, I'd contend the oceanic/continental distinction really only makes sense for crust material, not plates as a whole. The Atlantic is entirely on "continental plates" (North American, South American, Eurasian, and African), despite clearly being an ocean, and clearly lying over oceanic crustal rocks. In fact the only ocean that's over an "oceanic plate" is the Pacific. The Atlantic, Indian, Arctic, and Antarctic are all over plates with continents on them. At any rate, we have a nice table at Continent#Number_of_continents, with a very apt observation at the top of the Continent article "[Continents] are generally identified by convention rather than any strict criteria." -- 75.42.233.82 (talk) 05:35, 30 December 2008 (UTC)[reply]
I didn't use the tectonic plates as the only criteria, but also included large contiguous regions in the same continent, with exceptions for very narrow land bridges, as at Suez and Panama. Being identified by "convention rather than strict criteria" simply means "it's that way because somebody said it should be", which seems most unscientific to me. Much as Pluto was considered a planet "by convention", but was later reclassified, we can do the same for the continents. StuRat (talk) 06:03, 30 December 2008 (UTC)[reply]

Busking in Europe

I´m a puppeteer busker and am considering doing a tour through Europe but not sure which countries or cities allow busking or which ones would be most profitable... Any light on the matter would be appreciated... Thankyou 83.37.15.27 (talk) 11:32, 29 December 2008 (UTC)[reply]

Join all the others on the South of France in the summer. Lots of pavement artists (and one can sleep on the beach.) Most seem to start at one end, say Nice, and move along from town to town. Lucrative I am told.86.216.251.55 (talk) 15:13, 29 December 2008 (UTC)DT[reply]

Barcelona allows licensed performers on Las Ramblas. Competition is extremely fierce. One thing might be to get together with someone with an existing license or apply as a group with others. Unlicensed performers can be found around the magic fountain [20]. They are tolerated to a certain degree. In Germany the season is extremely short and authorities frequently clamp down on busking. You'd probably do better applying at local "Kneipe" pubs [21](taverns?!) many of which will let you perform if you are good enough to increase their profits. For puppeteering there frequently are "Theaterfestival" [22] events in summer. You'll have to apply with the local authorities or risk finding a slot on the fringe. Local "Kindertheater" [23] theaters for children might offer you a gig. Another possibility might be a "Kinderzirkus" [24] circus. You'd need some rudimentary German there since the focus would be on teaching kids to puppeteer and maybe make their own (sock-) puppets. A bit more challenging financially would be renting a room at local schools or a local "Jugendzentrum" youth center [25]. You'd have to get posters printed at a copy shop and there would be no guarantee that you'd make a profit after paying the rent. Some small village places, though might let you have a room for free. Be sure to collect references / letters of reference they can open doors for you. Germans are sticklers for paperwork. Lisa4edit (talk) 07:00, 30 December 2008 (UTC)[reply]
After a love-hate relationship with buskers, Transport for London has allowed official busking at designated pitches on the Tube for the past few years. You will probably have to apply well in advance and maybe attend an audition. More details here. Outside of the Tube, there's a well known outside performance space at Covent Garden, but again you have to deal with some officialdom. Elsewhere, I think it is less certain, with the police only too ready to move you on, especially if you (or your audience) are obstructing the flow of pedestrians or vehicles, or if someone (shop owner, cafe owner, etc.)) has made a complaint; and there's also the rather variable weather to contend with.
Outside London, street performers are rarer but not unheard of. Probably the best way would be to contact the local council (see Category:Local authorities of England - their websites are usually nameofcouncil.gov.uk) and ask about street performers in their area. Astronaut (talk) 16:18, 30 December 2008 (UTC)[reply]
Look out for festivals that have a street theatre component. I'm sure that street theatre performers are more welcome than buskers. Itsmejudith (talk) 18:29, 31 December 2008 (UTC)[reply]
It seems to me that you're going to need a particular kind of spot in order to do well with puppets. For example, in the London Underground (where limited busking is allowed), people can hear musicians from a long way off down the narrow, twisty pedestrian tunnels and stairways - so before they even see the performer, they'll have gained an appreciation for the performance - and will be ready to toss some change into the hat and keep walking toward their platforms while listening. Since tube stations are busy places with people who are mostly in a hurry, that's an OK business model. But for puppetry - I'm guessing you're going to need people to stop and watch for a while in order to get enough from your performance to make it likely that they'll contribute. So someplace like the London underground stations would probably be a total disaster for you. You need to be in places like parks, beaches and shopping malls - where people are in less of a hurry and can SEE you from a greater distance. SteveBaker (talk) 00:21, 1 January 2009 (UTC)[reply]

Dental Records

A dead body that has decomposed is often identified through the use of dental records. If authorities don't know who the person is, how do they know which records to search? Does the government keep a secret database of everyone's dental records? —Preceding unsigned comment added by 66.120.95.34 (talk) 15:04, 29 December 2008 (UTC)[reply]

I can't speak for all governments, but the United States doesn't. (I'm guessing that you're interested in the U.S. answer based on your IP address.) The NCIC (National Crime Information Center) has maintained a database of dental records of missing persons for the last 25 years or so. More recently, the FBI established the NDIR (National Dental Image/Information Repository) to support collection and analysis of more detailed image information. Individal states may have their own databases as well.
The authorities don't keep – or don't admit to keeping; if there were a secret database, how would we know? :D – records for everyone. Instead, federal law requires that police collect dental records for any person reported missing for more than 60 days. Records may also be collected for wanted individuals under some circumstances. TenOfAllTrades(talk) 15:28, 29 December 2008 (UTC)[reply]
Such a database wouldn't be secret - more likely For Official Use Only (or a cousin) or the apparently new Controlled Unclassified Information. If it existed. 98.169.163.20 (talk) 07:16, 30 December 2008 (UTC)[reply]
The way we would know is that they would need to get those dental records from all the dentists in the nation, and there's no way the government could secretly obtain all of those. Such a massive conspiracy would be bound to fail. StuRat (talk) 19:56, 29 December 2008 (UTC)[reply]
It should be noted that often police have an idea of who the body is and the only way to conclusively identify it is via dental records. The dental records serve as official confirmation rather than a shot in the dark. Tomdobb (talk) 17:34, 29 December 2008 (UTC)[reply]
Keep in mind this is different than, say, fingerprinting, which is comparatively easy to check through computerized means. --98.217.8.46 (talk) 20:43, 29 December 2008 (UTC)[reply]
I see no inherent reason why computerized comparison of dental records would be impossible. Assign a number to each of the 32 teeth and record which of them have fillings or are missing for each person, for example. This should allow you to narrow the search considerably. The final step of comparing X-rays to the teeth in a skull may be a bit more difficult to computerize, but could be done manually until we get an automated system working. StuRat (talk) 04:33, 30 December 2008 (UTC)[reply]
The inherent reason against would be the cost of upkeep of such a system. While fingerprints stay moderately stable for most people, teeth get new fillings, fall out, get pulled, capped, replaced with implants at a fairly regular rate. (Which keeps lots of dentists in business throughout the US and elsewhere.) If you'd like them to report their activities to a central database that would considerably increase the already whooping costs of dental treatment. Keeping data on missing people makes a lot more sense, since what gets recorded is the state of their teeth at the time they went missing. Given that an overwhelming percentage of those who get murdered suffer that fate close to the date of their disappearance, the dental records on file will be relevant for most cases. Lisa4edit (talk) 04:59, 30 December 2008 (UTC)[reply]
Even out-of-date records could be useful, though. For example, if the skull has a perfect tooth 7, you can eliminate all people on record with any dental work done to that tooth. StuRat (talk) 05:54, 30 December 2008 (UTC)[reply]

Global debts.

Is it possible that the Us global debt (private+public)/GDP is 835% and public debt (considering the public debt of FNM anfd FRE)/GDP is about 130%? Thanks. —Preceding unsigned comment added by Campi Lorenzo (talkcontribs) 17:59, 29 December 2008 (UTC)[reply]

I suppose anything's possible; what figures are you using? In the economy section of the article on the U.S., the GDP is $14.3 trillion. The United States public debt is around $10.6 trillion; toss in another $5 trillion for Fannie Mae and Freddie Mac, and call it $16 trillion for public debt. If by "private debt" you mean what's owed by individuals and organizations outside of the government, private debt would need to be about $103 trillion for the 835% figure to work: $16t public + $103t private = $119t; 119 is 832% of 14.3. I'm no economist but I'm doubtful that people and companies in the U.S. owe nearly one and a half times the world's gross domestic product. Keep in mind that if I owe $250,000 on a mortgage and I default, and Fannie Mae has to cover that, it's not the same as saying that for my house, Fannie and I owe $500,000 altogether -- it's the same $250,000. --- OtherDave (talk) 20:27, 29 December 2008 (UTC)[reply]
Also, a large part of the public and private debt in America is domestic, that is it is owed to other Americans. So from the holistic angle you appear to be driving at, I do not believe America as a whole is in anywhere near that much debt. TastyCakes (talk) 22:46, 29 December 2008 (UTC)[reply]

Dear OtherDave the numbers taht you showed were rights,but they consider the debt only once.Usa debt considering FNM and FRE is about 130%.The whole debt (private+public) is considered once and is about 835%.I know it's very much,but looking at many articles in financial newspapers of all over the world just today they are right.Thanks anyway for your answer. —Preceding unsigned comment added by Campi Lorenzo (talkcontribs) 16:48, 30 December 2008 (UTC)[reply]

Artist John Stanford

I bought a Civil War picture. I found the title to the picture it is " Blue Soldier on Horses" it is signed by John Stanford. I have tried every link to find info on the picture or the Artist. Is there any way you can help me or lead me to the place I need to look? I just wanted info on the Artist and if there was any history to the picture. Wanting to know what the flag stood for as well. Thank you in advanced. Seabird —Preceding unsigned comment added by 68.210.114.145 (talk) 21:27, 29 December 2008 (UTC)[reply]

Could this be him? Little Red Riding Hoodtalk 23:10, 30 December 2008 (UTC)[reply]

THE NEW YORK STOCK EXCHANGE

i want to know what the captions mean on the "big board": "VOLU", "UVOL", "TRIN", etc.. where can i get an explanation of all the captions? i have written all the news networks and the NYSE several times, without an answer.. are they really that busy? —Preceding unsigned comment added by Remoat (talkcontribs) 21:47, 29 December 2008 (UTC)[reply]

These are ticker symbols, abbreviations for company names and sometimes stock types. You may read, for instance, a news article mentioning "Electronic Arts (NYSE:ERTS)"; this means Electronic Arts' ticker symbol on the NYSE is ERTS, and ERTS on the display boards refers to EA. Your best bet is to look them up one at a time here. NeonMerlin 22:39, 29 December 2008 (UTC)[reply]
A quick google search will define these terms for you. VOLU is volume, UVOL is "up volume" (the volume of shares traded by issues that are currently (or were closed while) above their opening price), TRIN is the "short-term trading index" ([up issues / down issues] / [up volume / down volume]. If there are more that you are interested in, try googling the term and "NYSE". NByz (talk) 05:10, 30 December 2008 (UTC)[reply]
The original poster is referring to a particular display board in the New York Stock Exchange often seen on channels like CNN. The anchor will say "Let's take a look at the big board", then the entire board will be shown, and then a highlighted area will zoom in on the INDU and INDP numbers. (See Google Image Search Dow big board.) I found a list with many of the relevant symbols at Google Book Search Technical Trading Online, p 122. It's difficult for me to search for because I know nothing about stocks. And "Big Board" also means the entire New York Stock Exchange not just that one display board, so I'm having a hard time finding better pictures of it. --Bavi H (talk) 05:32, 30 December 2008 (UTC)[reply]

THANKS FOR THE REFERENCE TO THE NYSE TRADING CAPTIONS.. next question: is there a way to print this info? i don't see anything on this; highlighting doesn't work.. i never saw anything on the 'net i couldn't print until now.. what's the problem?? —Preceding unsigned comment added by Remoat (talkcontribs) 05:51, 31 December 2008 (UTC)[reply]

Rate of news

Have there been any studies to track how the rate at which distinct news events above a given notability threshold are reported in the media, or are actually generated by primary sources -- to track not just changes in reporting but changes in the amount of news actually happening? Is there a long-term trend? Can we expect more news next year than this year? Even more news the year after that? If there is a growth in the rate of bad news, will the rate of good news keep pace? NeonMerlin 22:36, 29 December 2008 (UTC)[reply]

Hmm I'm not sure but on a sort of related note, I remember seeing a map (I think it might have been animated through time) showing where news stories were happening, based on the tag lines in the associated press, which was pretty neat. I tried to find it again and couldn't, but I found this instead which is sort of interesting along the same lines: [26] TastyCakes (talk) 22:53, 29 December 2008 (UTC)[reply]
It's interesting how uneven reporting is. Some major events may not get reported at all, while some triviality makes all the major network news shows. Or an ongoing event, like the Iraq War or Afghanistan War, can just drop right off the face of the Earth as far as news reporting goes. StuRat (talk) 04:28, 30 December 2008 (UTC)[reply]

News Reliability

Not sure if this is the right place for this question, but here goes. On CNN and the BBC websites, each has top stories on politics, the economy, etc. Why do some of them have an author listed and some do not? For example, <http://news.bbc.co.uk/2/hi/middle_east/7803711.stm> has no author listed. Why?

Being a high school senior, I am taught that a source that does not have an author listed is not particularly reliable. Yet, because it is from the BBC or CNN, all is well?!? Thank you, The Reader who Writes (talk) 22:49, 29 December 2008 (UTC)[reply]

Some publications, particularly news outlets, do not give individual authors. I believe in most cases on cnn.com and bbc.co.uk this is because a number of people work on the stories and they are presenting facts (often from cooperative news pools like the Associated Press) rather than opinion or original research, for which knowing who produced the information is much more important. There are some publications (notably The Economist), that do present more opinionated articles but still don't identify individual authors. Their stated reason for doing this is to present a "collective voice" which remains consistent with the magazine's guiding principles in tone, content and opinion. In all of these cases, the organization as a whole is effectively giving their stamp of approval, so basically yes, it is all well that the BBC and CNN don't give the individual authors for everything because they would be held responsible as an organization if the information proved inaccurate, which should be as or more convincing than the name of a possibly unknown reporter. At least in theory... TastyCakes (talk) 23:06, 29 December 2008 (UTC)[reply]
As a counter example, most blogs prominently display their author's name, but are not terribly reliable! The determining factors in reliability are editorial control, that is multiple eyes on the same work, all checking each other, and reputation. Given that BBC and CNN and the Associated Press are all well respected, and show high degrees of editorial integrity, you can probably trust them. Even more important, is to state clearly where you got your information, so others can judge the reliability for themselves (i.e. "According to a CNN report dated yada yada yada" or "The BBC reported on such and such a date that so and so occured".) --Jayron32.talk.contribs 02:06, 30 December 2008 (UTC)[reply]
Also, many stories are "off the wire", meaning they print them pretty much as they were reported by a news service, such as AP, UPI, or Reuters. In such cases you should see one of those services listed, but no author's byline. StuRat (talk) 04:20, 30 December 2008 (UTC)[reply]

One more point is that in newspapers it's often only longer stories, representing a substantial amount of work, that credit the author with a byline. In other words, the decision may be up to the editor's judgement about whether the individual story represents enough work for a credit to be appopriate. (This only applies to news stories as opposed to personal columns or opinion items.) --Anonymous, 03:56 UTC, December 31, 2008.

Orchids

In your experience, where in (A) United States and (B) continental Europe can one see the best collections of orchids? Thank you in advance. --Dr Dima (talk) 23:57, 29 December 2008 (UTC)[reply]

In the United States, Longwood Gardens near Philadelphia, Pennsylvania, has got to be in the running. By their count, they have "3,200 different types of orchids" and display from 200 to 500 plants at a time. It's breathtaking, and the orchids are only one small part of the place. --Milkbreath (talk) 00:33, 30 December 2008 (UTC)[reply]

December 30

Obama Doll Gag gift

Any tips on how i can get an Obama doll out of the corner of a Claw vending machine using the claw?--76.28.73.16 (talk) 03:37, 30 December 2008 (UTC)[reply]

One toddler seemed to figure out the only way to get anything decent out of those machines...she crawled inside through the prize slot. They had to call the paramedics to rescue her, if "rescue" is the right word, since she was having a ball inside playing with all the good toys. StuRat (talk) 04:11, 30 December 2008 (UTC)[reply]

I heard that claw machines are designed so that the claws are weak most games then tighten up once every so many games. The way to test this is watch people use the claw machine until someone wins something then count the number of games until someone else wins. Then just count the number of games until you can win. --Candy-Panda (talk) 05:27, 30 December 2008 (UTC)[reply]

I wouldn't assume that it's a fixed interval, it's probably random. StuRat (talk) 05:48, 30 December 2008 (UTC)[reply]
Claw machine toys are usually cheap trinkets. Google for it, I bet you can just buy it outright for less than it would take to try and get it with the claw. --98.217.8.46 (talk) 17:56, 30 December 2008 (UTC)[reply]
Many, MANY years ago - my father made neon signs. He made a really nice one for an arcade on the beachfront at Margate - but the owner couldn't afford to pay for it. In recompense, he agreed to let my father run the arcade for thee months in the summer season and to take all of the profits. Hence (via a very complicated series of events) I ended up (as a 10 year old kid) being in charge of restocking the claw machine during my summer vacation. First, let me tell you that the 'prizes' in those machines cost less than the money you put into the machine for one play. It's AMAZING how cheap those crappy dolls and things actually are. So if everyone won a prize every time, the owner would still make a profit. Secondly, the owners invariable 'adjust' the spring tension in the claw so that they are not physically strong enough to lift the larger and more expensive prizes - so rule #1 is to always aim for something LIGHT. Rule #2 is that arcade owners almost always put very desirable - but very unobtainable things into the machine to attract people into trying to get them instead of the cheaper things. Since an Obama doll probably costs more than the other junk in the machine - the owner will have carefully placed that doll in a position where the claw cannot possibly get to it. In "our" arcade on Margate seafront, my father placed his own gold Rolex wristwatch into the machine - having first made very sure that it was smaller than the minimum gap between the teeth of the claw! After three months, it was still in there - and we probably made more than it was worth (which was a lot!) from the suckerscustomers who tried to get it out. Some owners have been known to surruptitiously glue valuable prizes to the bottom of the machines to achieve the same result. SteveBaker (talk) 00:06, 1 January 2009 (UTC)[reply]
How about if they grabbed the Rolex and a big plush doll in the same grasp ?
Were there any laws that governed such gambling machines ? Let's take an extreme case where every single prize was glued down. Would that have been legal ? StuRat (talk) 08:40, 1 January 2009 (UTC)[reply]

Finding a Finance Sector Job in Vancouver, BC

I will be moving to Vancouver shortly, and am looking for a job in Finance. Naturally, it's not the best time for this, but luckily the big Canadian banks were conservatively capitalized coming into this recession and haven't announced any layoffs yet.

My plan is to gather a list of firms, rank them by my desire to work with them, gather HR or Management contact information and politely harass them in the same order as I have ranked them. This worked well for my last job in Victoria, BC, but Vancouver is much bigger, and I want to make sure that my list of firms is exhaustive.

I am looking interested in the following order:

1) Private Equity or VC 2) Commercial Banking or Business Financial Services 3) Asset Management (Mutual Funds, Pension Funds, anything market oriented) 4) Corporate Treasury

I am working on my CFA designation, so I need a job that satisfies the requirement that I be "...making investment decisions or adding value to the process."

I plan on using the chamber of commerce site, local Venture Capital firm listings, and just doing general searches on job listing sites etc. I want to approach firms before they post jobs, when possible. Can anyone recommend other places where I could find lists and/or HR contact info for financial firms around Vancouver? Any specializations or search terms I'm overlooking? General Job-Seeking advice? Thanks! NByz (talk) 05:57, 30 December 2008 (UTC)[reply]

What is this......thing!?

It's like the drainpipe grew legs......I HOPE YOU GUYS CAN IDENTIFY - AND STOP THIS CRAZY THING!...(!).....God, I can't figure out what this is.--Mark L. Dowry (talk) 06:08, 30 December 2008 (UTC)[reply]

A photo might be nice. -- JackofOz (talk) 06:11, 30 December 2008 (UTC)[reply]
See thing. - Lisa4edit (talk) 07:13, 30 December 2008 (UTC)[reply]
How many legs? --Dr Dima (talk) 08:12, 30 December 2008 (UTC)[reply]
Wild guess: House centipede? --98.217.8.46 (talk) 17:53, 30 December 2008 (UTC)[reply]
Thanks 98! I want some, they eat Cockroaches! What could be better? If they also eat moths I'll cancel our exterminator and get a couple of those guys instead. Does anyone know anything about their environmental impact in the US? Are they an animal form of kudzu? Lisa4edit (talk) 18:47, 30 December 2008 (UTC)[reply]
Another wild guuess a shape shifting alien that'll destroy us all Dmcq (talk) 18:40, 30 December 2008 (UTC)[reply]
I'd go with a badger flusher. CambridgeBayWeather Have a gorilla 23:17, 30 December 2008 (UTC)[reply]

Player-managers

There was a time not that many years ago when player-managers were fairly common in English football, at least in the lower reaches of the league - but I can't remember the last time I heard of one. Are there any player-managers currently in charge of any English league clubs, and have there ever been any in the history of the Premier League? Thanks in advance, Grutness...wha? 09:08, 30 December 2008 (UTC)[reply]

I'm not sure of current player-managers but Chris Coleman managed Fulham as a player manager (EDIT - maybe he didn't - anyhoo check Player-manager it has a list of notable ones), and if I recall correctly so did Gianluca Vialli when he was at Chelsea 194.221.133.226 (talk) 11:23, 30 December 2008 (UTC)[reply]

Glenn Hoddle at Chelsea and Bryan Robson at 'Boro? Heh, the way things are going at West Ham, Zola might end up having to pull his boots on again... --Kurt Shaped Box (talk) 11:38, 30 December 2008 (UTC)[reply]
Gordon Strachan famously used his role as Coventry's player-manager to get around the 'manager can only give instructions from the technical area' rule by going for long and elaborate warm-up routines down the touchline whilst shouting at his players (and the linesman). Nanonic (talk) 12:36, 30 December 2008 (UTC)[reply]
Peter Reid and Stuart Pearce (at Forest) were two others. Currently the only player-manager in the league is Barnet's Ian Hendon, but that's kind of cheating because he's a caretaker who was appointed on a temporary basis yesterday. I'd speculate that the reason for their decline is that Premier League rules stipulate that all managers must have a UEFA Pro Licence, a rule which was only introduced recently. Some countries do not permit player-managers at all, Italy is one of them, which is what made Vialli's time at Chelsea unusual. Oldelpaso (talk) 17:17, 30 December 2008 (UTC)[reply]
Parts of Paul Tisdale's article seem to imply he is still registered as a player with the number 17 shirt, and he manages Exeter City. He has not played a match since 2000, however. --Iae (talk) 18:52, 30 December 2008 (UTC)[reply]
IIRC, Alan Curbishley was still registered as a player at Charlton for several seasons after he last made an appearance for them... --Kurt Shaped Box (talk) 21:20, 30 December 2008 (UTC)[reply]
Roberto Martinez of Swansea City is still officially a player-manager as he has not yet officially "retired" from playing - however he is not listed on the squad listings for the team and therefore refered to simply as "manager" of the swans... He could however add his name to the squad list if he wanted and play... So that sort of fits i think... Gazhiley (talk) 10:53, 31 December 2008 (UTC)[reply]

About the pro-license. I am to understand that this is recommended but not required. From the looks of this (http://forums.cfcnet.co.uk/index.php?s=689ada1041157aed0e108f6fbfc318c4&showtopic=37553&st=0&p=639019&#entry639019) it seems that it is not until 2010 that managers coming into the Premier League are required to have the license. 194.221.133.226 (talk) 09:51, 31 December 2008 (UTC)[reply]

From [27] and [28], it seems that the rules already require it except for short term appointments, but seeing as Gareth Southgate, Paul Ince and Avram Grant all lacked it, the rules are rarely enforced. Oldelpaso (talk) 16:24, 31 December 2008 (UTC)[reply]

Canadians abroad

Is it true that Canadians outside Canada often wear something with the the Canadian flag on it, so that no one confuses them with Americans? --88.27.176.105 (talk) 12:32, 30 December 2008 (UTC)[reply]

Yes, or so the joke goes. The joke continues with a story of astute and sensitive American tourists proudly displaying their Canadian flag as well. The real reason is that it is easier to find, and be found by, other Canadian tourists, or simply to show a bit of patriotism, but everyone has an anecdote about this situation, which may or may not be true. Adam Bishop (talk) 12:59, 30 December 2008 (UTC)[reply]
Dave Foley had a great quote, which I will try to paraphrase here: "I'm Canadian. That's like being an American, but without a gun". I miss the Kids in the Hall. --Jayron32.talk.contribs 18:09, 30 December 2008 (UTC)[reply]
Heh - and I miss Due South for it's continual series of US vs Canada jokes and situations. Grutness...wha? 23:01, 30 December 2008 (UTC)[reply]
Those wearing the Maple Leaf are usually Americans with a touching faith in its ability to repel terrorists. (See I Am Canadian for the difference between the countries, seen from north of the border.) Canadians can be identified by the MEC patch they tend to wear, especially when travelling. BrainyBabe (talk) 19:27, 30 December 2008 (UTC)[reply]
In my experience, the number of Americans wearing maple leafs in foreign countries is vastly exaggerated. Most of the people you see are in fact Canadians, who are given to the (IMO) somewhat naive notion that Canadians are loved by foreigners as much as Americans are despised. I believe the true feelings of most non-north americans towards Canadians lies much closer to indifference. TastyCakes (talk) 21:52, 30 December 2008 (UTC)[reply]
FWIW, as someone living in a country that gets a lot of visitors from both countries, if you're unsure, assume the person is Canadian. Canadians will feel delighted that you've "spotted them correctly", and Americans for the most part don't mind being mistaaken for their northern neighbours 9those that do have usually made it abundantly clear they're from the States anyway). As far as wearing the maple-leaf is concerned, hitchhikers do, I've noticed, but hitch-hikers often temd to wear some sort of "country of origin" identifying feature anyway (the number of German flag patches I've seen over the years...) Grutness...wha? 23:01, 30 December 2008 (UTC)[reply]
I just finished watching the movie "In Bruges" in which Collin Farrell punches out two tourists that he believes are American in a restaurant. Upon being arrested, and learning that they were Canadian, he shows remorse. NByz (talk) 02:45, 31 December 2008 (UTC)[reply]

On the whole I believe that American tourists (note tourists) are not liked in Europe. Their money is welcome, but so many brash 'yanks' have soured the image for the others. BUT - one has to acknowledge that brash and unwelcome tourists from any country are money welcome only. British and Germans abroad have received much criticism. Canadians seem relativelt few and far between - perhaps any that misbehave are regarded as from the US anyway ?86.202.24.247 (talk) 14:40, 31 December 2008 (UTC)DT[reply]

Another factor is the similarity in speech patterns. A random talking Canadian without any country identifiers on their clothing can quite easily be assumed to be a random talking American, until they say words like "about", which to my ears is more like "a boat". The differences between Canadian and American speech are somewhat fewer than those between Australian and New Zealand speech, for example. There are a lot more Americans than Canadians, so the assumption would be reasonable and statistically supportable. -- JackofOz (talk) 00:27, 1 January 2009 (UTC)[reply]
Eh? CambridgeBayWeather Have a gorilla 08:19, 1 January 2009 (UTC)[reply]

Bush "White House Weekly Review" Emails

The Bush White House has sent out a weekly "White House Weekly Review" email to those who subscribe at least since June 2003. Most times it seems to come on Saturdays, which is how it is advertised here:

http://www.whitehouse.gov/email/

However, I've subscribed since 2003 and found that sometimes the email comes on Fridays or, less frequently, Sundays, and sometimes not at all (the last few weeks).

I've got most of these emails saved in my email account going back to 2003. However, there are significant gaps. And I can't remember if I deleted them some weeks or if they were not sent out from the White House those weeks. So I'm looking for a list of these emails - including the date sent and the subject header.

Now, there is a "Newsletter Archive" website that has these emails available going back to only December, 2006, here:

http://www.newsletterarchive.org/2006/from/White+House+Weekly+Review/

I need them going back to June, 2003.

I'm working on a memoir and I need these emails to fill out the story.

Any help is greatly appreciated.

Thank you...

-SM —Preceding unsigned comment added by 173.110.187.23 (talk) 12:37, 30 December 2008 (UTC)[reply]

Um contact the Whitehouse? Isn't all presidential communication supposed to be a matter of public record anyway Nil Einne (talk) 16:38, 1 January 2009 (UTC)[reply]

How to stop a cavity from getting infected?

This question has been removed. Per the reference desk guidelines, the reference desk is not an appropriate place to request medical, legal or other professional advice, including any kind of medical diagnosis, prognosis, or treatment recommendations. For such advice, please see a qualified professional. If you don't believe this is such a request, please explain what you meant to ask, either here or on the Reference Desk's talk page.
This question has been removed. Per the reference desk guidelines, the reference desk is not an appropriate place to request medical, legal or other professional advice, including any kind of medical diagnosis or prognosis, or treatment recommendations. For such advice, please see a qualified professional. If you don't believe this is such a request, please explain what you meant to ask, either here or on the Reference Desk's talk page. --~~~~
TenOfAllTrades(talk) 17:31, 30 December 2008 (UTC)[reply]
Go to a dentist. Don't mess around. Tempshill (talk) 16:13, 30 December 2008 (UTC)[reply]
Even if you don't have the time or ability to visit a dentist, you should at least talk to one. We can't offer you medical advice, and in any case you probably shouldn't trust the advice of random strangers from the internet. TenOfAllTrades(talk) 17:31, 30 December 2008 (UTC)[reply]

International / U.S. State borders

Where can I learn about the origins of border locations between nations and states? In the United States, I'm interested in finding out why in some places (mostly eastern states) the border follows natural landscape features such as rivers, while in others (mostly western states) the border is a straight line. I know the border between the U.S. and Canada is at the 49th parallel by treaty, but there are some oddities such as north-central Minnesota and a small plot of land in northwest Washington which are part of the U.S. but connected by land only to Canada. Also, West Virginia has a thin strip of land separated from Ohio to the west by a river and Pennsylvania to the east by a straight line. The way these boundaries are drawn sometimes don't make any sense. —Preceding unsigned comment added by 66.120.95.34 (talk) 16:13, 30 December 2008 (UTC)[reply]

As to the US/Canada border, the reason for the oddity at Lake of the Woods, where Minnesota meets Ontario and Manitoba, is as follows. At the end of the AMerican Revolution, the Treaty of Paris (1783) specified the water border as we know it through the Great Lakes and Lake of the Woods and the rivers and lakes connecting them. This ended at "the most northwestern point" of Lake of the Woods" and from there the border was supposed to go due west until it hit the Mississippi River. Unfortunately this was impossible, as the northwest angle of the lake is at 49°23'N while the Mississippi runs south starting from Lake Itasca at only 47°13'N. There are tributaries of the Mississippi, farther west, that do extend far enough north (the Frenchman River or Creek, a tributary of the Milk River, has a source in the Cypress Hills at around 49°45'N), but the treaty-makers didn't mean to count those. They meant the Mississippi proper, but the map they were using simply got the relative positions of the Mississippi and Lake of the Woods wrong.

Later, when this was realized, the 49°N border was established by the Treaty of 1818. This time the treaty-makers wanted to make sure they didn't have an impossible border, so they specified that from the northwest angle of Lake of the Woods the border would run "due North or South, as the Case may be" to reach 49°, and then due west. And this produced the present border. In other words, it forms a weird shape because it was drawn by people who didn't have maps they could trust (and through country that didn't have a lot of settlers already living in it, so their wishes didn't have to be considered). And when it turned out that the north-south border clipped off a peninsula on the west side of the lake, well, that was just too bad.

(Incidentally, it turned out that the 1818 border actually was still impossible, or at least ambiguous. The northwesternmost point of the lake turns out to be at the end of an inlet that runs almost due north from the main part of the lake, with the result that the border within this inlet, as defined in 1818, crosses itself. No land area was affected this time and a later treaty cleared things up.)

As for Point Roberts, it's a similar thing: the Oregon Treaty specified that the 49° border would be continued as far west as the middle of the channel between Vancouver Island and the mainland, nobody had detailed maps that showed that this line clipped off the tip of a peninsula, and there weren't enough people there who would care to worry about it. No doubt the third such anomaly, the Alburg Peninsula in Lake Champlain, arose in the same way when the Treaty of Paris (1783) specified latitude 45°N for that part of the border. (This one is less well known because the lake is narrow enough that the peninsula is easily reached from the US mainland by roads bridging across it.) --Anonymous, 05:07 UTC, December 31, 2008.

"How the States Got Their Shapes" is a pretty good book for this topic, but it is not without mistakes. For example it traces the use of the 42nd parallel to the Nootka Convention, and even explains the reason why the Nootka Convention chose the 42nd parallel--but in fact the Nootka Convention says nothing about the 42nd parallel and the reason given in the book is absurd. Still, for the most part, the book is good. Pfly (talk) 08:19, 31 December 2008 (UTC)[reply]
Also, see wedge (border). ~AH1(TCU) 18:44, 1 January 2009 (UTC)[reply]

For the rectilinear boundaries of western states, the longitudes were defined from Washington (in at least some cases); what was the exact reference point? —Tamfang (talk) 20:26, 1 January 2009 (UTC)[reply]

Correct Etiquette of some difficult foods

What is the correct etiquette for eating cherries? Does one put the entire cherry into one's mouth then spit out the seed in the middle? Where does one place it afterwards? Acceptable (talk) 18:13, 30 December 2008 (UTC)[reply]

When eating cherrys or olives or other pitted foods, I generally politely spit the pit either into my hand, by bring my hand directly to, and covering, my mouth, and then immediately transfering the pit to the edge of my plate or a napkin, or alternately, you could bring your napkin to your mouth, and deposit the pit that way. Eating pitted fruits means you will ALWAYS have a pit to spit out, the trick is to extricate the pit from your mouth without showing everyone what you are doing. --Jayron32.talk.contribs 18:24, 30 December 2008 (UTC)[reply]
For some reason I remember watching a morning news show like Good Morning, America or something and they had an etiquette expert on the show. A similar question was asked of the expert. She basically just said to nonchalantly pull the pit, bit of bone, whatever, out from between your lips and place it on the side of your plate. Trying to hide what you're doing would look silly and childish. Though don't make a production over it such as spitting it out which would be considered vulgar. Dismas|(talk) 20:19, 30 December 2008 (UTC)[reply]
There's a fine line between discretely removing the seed from your mouth, and pretending there's no seed to remove. Pretending is absurd, particularly if the cherries are on the table and everyone's seen you partake of the fruit (and, more than likely, others are doing the same thing). Just use your fingers, as Dismas said. Everyone knows it's a seed, and you don't have to take any special pains to hide it, because it's going to be visible once it gets to the plate anyway. Putting a napkin up to your mouth seems somewhat distasteful to me; it's normally only done when you've put something in your mouth that turns out to be off, or extremely hot, or for whatever reason you need to get rid of it immediately without appearing to spit or vomit. -- JackofOz (talk) 21:26, 30 December 2008 (UTC)[reply]
An old joke that my grandmother used to tell in order to discourage swallowing cherry pits:
Mother: Son! You're not swallowing those cherry pits are you?
Son: Of course not mama!
Mother: So what are you doing with them?
Son: (who is seated with a window behind him) I'm throwing them out the window mama!
Mother: The window is closed you stupido .... <fill in the rest>...! hydnjo talk 22:24, 30 December 2008 (UTC)[reply]

This says to do it while hiding the pit with your hand as does this. This says to put stuff on a seperate plate. However, my favourite is the 1940 Formal Dinner Etiquette. They suggest using a spoon first or if in the mouth already just remove it. Of course the most fun way involves two people, nudity and no kids around. CambridgeBayWeather Have a gorilla 23:13, 30 December 2008 (UTC)[reply]

Some etiquette book I read once (I wish I could remember which one it was) said that you should remove a seed, pip, pit, etc. the same way you put it into your mouth. In other words, if you used a fork to insert it, remove it with a fork, if you used your fingers to insert it, remove it with your fingers. Little Red Riding Hoodtalk 23:15, 30 December 2008 (UTC)[reply]

Uploading picture

I'd like to upload a picture of a chicken that I took myself. I don't care if anyone uses it and I don't care if I'm not given credit. I can't figure out how to upload it on Wikimedia Commons. Can someone help me?--Pufferfish4 (talk) 19:24, 30 December 2008 (UTC)[reply]

Have you registered on commons? If so, you go to the commons main page and click on "upload file" on the left hand bar. TastyCakes (talk) 20:14, 30 December 2008 (UTC)[reply]

I know how to do that. I can't figure out what lisence to use.--Pufferfish4 (talk) 21:55, 30 December 2008 (UTC)[reply]

At commons they give you several choice for your own work, any of which is fine. GFDL works fine for wikipedia using it, PD (i.e. agreeing to put it in the Public Domain) lets anyone use it. RJFJR (talk) 22:09, 30 December 2008 (UTC)[reply]

Converting software

What is a good video converting software where I can at least convert a fifty-second video from .wmv to .avi? A Google search returns no good free ones. 75.169.199.193 (talk) 20:01, 30 December 2008 (UTC)[reply]

Are you using Windows XP or Vista? If so, have a look at this page: [29]. - Akamad (talk) 21:06, 30 December 2008 (UTC)[reply]
Oh... I have never thought of that! Thank you (I have Vista and the link does work). 75.169.199.193 (talk) 22:53, 30 December 2008 (UTC)[reply]

Additional question

I have converted it and upon attempting to place it on my Nikon L18 camera, a message appears stating the file has one or more properties outside the device limits, then encouraging me to use a media program to convert the file... however, the file is already converted to the proper format. If I should place this question on the technology section just say so. Thanks for the help. 75.169.199.193 (talk) 23:15, 30 December 2008 (UTC)[reply]

Place it on a camera? As in on the built-in memory? AVI videos are uncompressed, it's not inconceivable that it's rather larger than the (23MB) space available. You may need to purchase an SD card. If this is what you're trying to do, then this probably isn't the problem. Angus Lepper(T, C, D) 01:15, 31 December 2008 (UTC)[reply]
Correct, I am trying to place it on the camera - however, I did compress the AVI video to 13 MB so space is not an issue - perhaps the compression renders it incompatible with the camera? 75.169.199.193 (talk) 01:34, 31 December 2008 (UTC)[reply]
Check to see that your converted file matches one of the supported modes, which (according the manual on the Nikon site) are -
Image size Frame Rate 
640x480    30 fps
320x240    30 fps
320x240    15 fps
--LarryMac | Talk 17:05, 31 December 2008 (UTC)[reply]
Lowered the frame rate (640x480 video) to 29, but to no avail. It simply looks like I won't be able to do this, but that's okay. Thanks for all your help. 75.169.208.225 (talk) 20:16, 1 January 2009 (UTC)[reply]

December 31

What is " A NI SA HA NI " ?Siouxlin (talk) 01:28, 31 December 2008 (UTC)

Saw on a T shirt: may be an organization or philosophy or group. I would like to know what it refers to.Siouxlin (talk) 01:28, 31 December 2008 (UTC)[reply]

Blue Clan. CambridgeBayWeather Have a gorilla 03:07, 31 December 2008 (UTC)[reply]
(after ec on Humanities desk, where the question was drawn away from under my feet, while I was answering): Possibly the Blue Clan in Cherokee/Ah-ni-yv-wi-ya society. See the article on Cherokee Clans where it's transliterated as "Ah-ni-sa-ho-ni". ---Sluzzelin talk 03:12, 31 December 2008 (UTC)[reply]

Song

In NCIS Season 3 Episode 21 (Bloodbath) when the team is entered Krime Kleaners building there is a song playing, I've been unable to identify it, can anyone tell me which one it is? Joneleth (talk) 03:29, 31 December 2008 (UTC)[reply]

It's got to be Rob Zombie, "American Witch". --Milkbreath (talk) 03:59, 31 December 2008 (UTC)[reply]

Yea thats it, thanks alot. Joneleth (talk) 04:32, 31 December 2008 (UTC)[reply]

Market prices/agriculture

Where can I find market prices(U.S., or anywhere, just ballpark values) for farm-type stuff like cattle, pigs, whatever. —Preceding unsigned comment added by 96.227.98.148 (talk) 04:12, 31 December 2008 (UTC)[reply]

The Chicago Board of Trade and Chicago Mercantile Exchange are the main places where futures contracts (an agreement to deliver this good at a future time) are traded. This should represent the expected basic price of a large-volume or industrial order, but not a consumer-facing price. Here is CME: [30] and here is CBOT [31]
You will probably have to look up the units of measurement used to convert it into whatever you're using it for. Most things are "bushels" etc. NByz (talk) 06:25, 31 December 2008 (UTC)[reply]
I was interested, so I looked this one up: Live Cattle is traded in units of 40,000 lbs of "55% choice, 45% select grade" (not sure what that means). Price quoted is cents per pound. NByz (talk) 06:33, 31 December 2008 (UTC)[reply]
You might find the USDA Agricultural Marketing Service a useful source of info. Astronaut (talk) 08:27, 31 December 2008 (UTC)[reply]

British Farthing

Is there a connection between the last year the Farthing was legal tender and the fact that it was 1/960th of a pound sterling? Or just coincidence? Coolotter88 (talk) 04:36, 31 December 2008 (UTC)[reply]

There are no coincidences, only hitsuzen ;) HHOS --Dr Dima (talk) 06:19, 31 December 2008 (UTC)[reply]
Maybe you mean "a coincidence" is not "just a coincidence". -- JackofOz (talk) 06:23, 31 December 2008 (UTC)[reply]
Which strangely reminds me of the Seinfeld about whether there are degrees of coincidences - "there are no big coincidences or small coincidences, there are just coincidences" is how it went, I think. 194.221.133.226 (talk) 09:46, 31 December 2008 (UTC)[reply]
I can't imagine it would be anything other than coincidence. Astronaut (talk) 08:10, 31 December 2008 (UTC)[reply]
To give the questioner a more serious answer, I think there is a connection. The farthing went out of circulation in 1960. In 1960, the pound sterling was worth about only one third as much as it was in 1940 due to inflation. People's earnings grew even faster than inflation during that period. Consequently, where a shopper might have cared in 1940 whether potatoes cost two pence per pound or two pence and a quarter per pound, so that it was worth the trouble to the shopper and the merchant to account for farthings in accounts and in tills, the shopper no longer cared much about such a trivial difference in 1960, and it was no longer worth the trouble. Put differently, the 1940 farthing was worth about 5p in 2008 terms. By 1960, it was worth barely more than 1p today. It would not be so far fetched today for the Bank of England to retire the penny as a cost saving measure, since the average shopper doesn't care much whether a pound of potatoes costs 55p or 56p. Marco polo (talk) 16:53, 31 December 2008 (UTC)[reply]
I realize that the coin was not worth a lot in 1960. I was talking about whether if the year 1960 had anything to do with 1/960th of a pound. Everyone seems to say that it's a coincidence. Thanks for the help though. Coolotter88 (talk) 19:15, 31 December 2008 (UTC)[reply]
I suppose it's not utterly out of the question that there could have been a subconcious influence on the decision-makers. We don't seem to have an article on arbitrary coherence, but here's an article on it. I don't have any suggestion as to how you might figure out whether this was actually a contributing factor. --Trovatore (talk) 21:48, 31 December 2008 (UTC)[reply]
It seems to me that if someone in government with enough influence to set the date of the farthing's demise had come up with this idea - they'd have made it very public at the time and we'd be able to discover that fact. There would be little point in picking that exact year - forcing either the early or late retirement of the coin through all of the necessary regulatory hurdles just to make that date work - then keeping entirely quiet about all of that cunning planning! So I'm going to say "Coin-cidence". SteveBaker (talk) 23:35, 31 December 2008 (UTC)[reply]

Sexy veggies

A few months ago there was an add campaign for greenpeace or a similar organization that showed sexy vegetables and greens photographed in an erotic manner. I can't find it anymor. Would anyone have a link to the adds? Thank you. Keria (talk) 14:42, 31 December 2008 (UTC)[reply]

Where did the ads appear? (What country, region, or market?) Were they print, television, or billboards? Were the ads in English? TenOfAllTrades(talk) 15:33, 31 December 2008 (UTC)[reply]
Unfortunately I didn't see them at the time, they were all over the ternet though, and I guess they must have been published in some magazines I really couldn't say where (surely in the US and UK, I would guess in continental Europe, I don't know where else). They were print and internet campaigns I don't know if they made videos for them. It's strange that I can't find it through search words as I remember it being all over add and visual production blogs. Cheers. Keria (talk) 15:45, 31 December 2008 (UTC)[reply]
Did you mean People for the Ethical Treatment of Animals#Lettuce Ladies? CambridgeBayWeather Have a gorilla 20:22, 31 December 2008 (UTC)[reply]
Should have looked further down, Lettuce Ladies and Broccoli Boys. Good thing I know the enemy. CambridgeBayWeather Have a gorilla 20:24, 31 December 2008 (UTC)[reply]
All right, I think I found what I was looking for. It is actually called Forest Love and I was really misleading as it seems to actually be a campaign to save trees by Greenpeace. I also heard there was a Norwegian campaign to promote vegetarianism that used the theme of sexy vegetables, so if anyone has any more reference on that theme they are very welcome. Keria (talk) 21:33, 31 December 2008 (UTC)[reply]
Have you tried a Google Image search on "sexy veggies"? I have, and I'll never eat cucumbers or corn on the cob again. Search tip: turn on SafeSearch. --Milkbreath (talk) 21:40, 31 December 2008 (UTC)[reply]
It was pretty bad, but it wasn't as bad as the time I didn't know what "yaranaika" was. bibliomaniac15 21:48, 31 December 2008 (UTC)[reply]
There was an ad campaign advocating use of condoms last time I was in Germany. There were posters featuring veggies wearing "rubbers" in places like subway stations. Found one in Google: [32] 76.97.245.5 (talk) 09:10, 1 January 2009 (UTC)[reply]

What is Topaz Gold please

No definition on WKPD —Preceding unsigned comment added by 91.110.149.97 (talk) 15:59, 31 December 2008 (UTC)[reply]

I'd never seen that abbreviation before and thought you were talking about a television or radio station... Anyway, could you provide some context? Topaz Gold could be used to refer to anything from a brand of organic beverages to gold rings with topaz gems in them to a color of decorative bead to a cellphone. Dismas|(talk) 16:13, 31 December 2008 (UTC)[reply]

MIT Undergrad

What is the best dorm? —Preceding unsigned comment added by 96.227.98.148 (talk) 22:05, 31 December 2008 (UTC)[reply]

Presuming there are MIT residents on here, you might get some answers, but you'll have to provide a little more context, as to what you're looking for? Low crime? (Yeah, even at MIT it could be a problem?) Activities? (Some dorms have things more geared for fellowship and community.) Offering single rooms versus rooms where you must share?
I'd recommend you contact MIT and search under "housing." My hunch is you'll get a good description of each dorm. Of course, the description won't always be perfect. At College of Wooster when I was there, one dorm was supposed to be a quiet community, with much more stringently enforced quiet times, but the times they could be loud it was loudest on camps; and even times when it was supposedly quet. :-) So, some contact with students who can answer would help, too.Somebody or his brother (talk) 23:46, 31 December 2008 (UTC)[reply]
This article might interest you [33] - 76.97.245.5 (talk) 09:19, 1 January 2009 (UTC)[reply]
Here is the official introduction to MIT housing. Here is a clickable list of dorms. And here are some student blog posts on life in specific dorms. Marco polo (talk) 19:24, 1 January 2009 (UTC)[reply]

January 1

What Vietnamese ethnic groups originate from the North Central Coast region?

I understand that the Montagnard/Degar people originated from Central Highlands (Vietnam) between Huế and Ho Chi Minh City. I am inclined to believe that Vietnamese people from Huế are similar to the ethnic groups found in the Central Highlands and South Vietnam regions but I don't want to make any rash assumptions. During the Vietnam War, for instance, Huế was considered South and was anti-Communist. Lhboga (talk) 01:00, 1 January 2009 (UTC)[reply]

Death Pool 2009

Well it's that time of year again. Once again I dominated my celebrity death pool with the help of Wikipedia. Thank you to anyone who helped me last year. If anyone knows of a famous person who is likely to cash it in in the year 2009 please reply below.

DISCLAIMER - the death pool I am in is for entertainment purposes only(no money.) I will not kill anyone and please don't kill anyone for me. The only requirment to being famous is that they have to have a page on wikipedia. Thanks again! --ChesterMarcol (talk) 01:26, 1 January 2009 (UTC)[reply]

I hope you're making good by donating half your winnings to the Wiki foundation (see top of page)! SteveBaker (talk) 01:32, 1 January 2009 (UTC)[reply]
If I got any money for winning I would.--ChesterMarcol (talk) 01:40, 1 January 2009 (UTC)[reply]
Angela Lansbury is getting up there in age. Though her article doesn't mention anything more than a knee replacement in the last few years. Dick Cheney and John McCain are thought by many (WP:OR) to be in view of death's door. Bob Barker is no spring chicken. And Ed McMahon has had a number of health issues in recent years. The stress of being in financial trouble could be weighing on his health. Dismas|(talk) 20:47, 1 January 2009 (UTC)[reply]
A couple of people I've written articles about must be about due: Hugues Cuénod is 106 and Roy Douglas is 101. But Cuénod married his partner only 2 years ago, when he was 104, so maybe he now has a new reason to keep on going. -- JackofOz (talk) 21:51, 1 January 2009 (UTC)[reply]

Question (2012)

Why do people believe there will be an acopalypse in 2012? 60.230.124.64 (talk) 01:59, 1 January 2009 (UTC)[reply]

Because they are extremely misguided. There's a bit more information at 2012#2012 metaphysical speculations. Algebraist 02:02, 1 January 2009 (UTC)[reply]
Millenarianism is very, very common. People are constantly harping about the end of the world (and have been for literally centuries and centuries and centuries), and it consistently fails to occur. In a way it's a nice idea—no need to plan for the future, to think about today's problems. But it's not a sound bet. --98.217.8.46 (talk) 03:14, 1 January 2009 (UTC)[reply]
Naturally, nobody can actually guarantee there won't be an apocalypse in 2012; but I agree, that's not the same thing as saying there will be one. -- JackofOz (talk) 03:30, 1 January 2009 (UTC)[reply]
Ask yourself how close did George Orwell get with his prediction in the novel 1984 or how close did Arthur C. Clarke get with his predictions in 2001: A Space Odyssey and they were comparatively modern and scientifically educated people trying to guess only a few decades ahead. 86.4.182.202 (talk) 15:21, 1 January 2009 (UTC)[reply]
That's not fair to either Orwell or Clarke. They wrote fiction as in, "I made this up myself". Neither ever claimed that their visions of the future was going to come exactly true. Especially 1984, you'd be nuts to think that that is the actual future. Orwell wrote it to make a point, it's a futuristic parable describing the dangers of totalitarianism, not a prediction of things to come. That's the difference between great writers like Orwell and Clarke and hucksters like those maniacs saying that the world will end in 2012. Lets not smear the former group with the taint of the latter. Belisarius (talk) 16:44, 1 January 2009 (UTC)[reply]
Although good science fiction, as opposed to fantasy, does try to predict a possible future. However, even someone like Clarke is not writing with any attempt to predict the future. Clarke was merely looking at the way things were going, using his imagination, and creating a fictional world from the way things appeared to him. It's just like Joel Rosenberg has done with his books. Some of the things he postulated as possible have come true, though even those he has written about haven't happened in exactly the same way. (His fictional account of how the U.S. might invade Iraq, for instance; obviously, a good writer with his finger ont he pulse of the world could have predicted something like that.)
Bring this full circle back to 2012. the end of the Mayan calendar in its present cycle, I think some New Age stuff, and other things are merely guesses. they don't really have their "finger on the pulse" of anything. Those who do have their finger on the pulse of things (some sci-fi writers with space travel, Rosenberg with the Middle East) may come close to the truth, but none of them is predicting the world to end in 2012.
However, it is much easier for the masses - the "lowest common dennominator," if you will - to follow those who just prdict big things that make noise. Noise makes news. It requires more effort to figure things out in a logical way, or even to listen to those who use knowledge of technology, Middle Eastern politics, etc.. The masses would often rather listen to those who excite them rathe rthan those who reason things out in a logical manner. Although, if it can be written about in an entertaining way (like Rosenberg), it can still be a bestseller. :-)Somebody or his brother (talk) 17:29, 1 January 2009 (UTC)[reply]

Poster keeps falling down

I have a holographic poster that I tried to stick on my door but it keeps falling off no matter how much blutack I put on it. What can I use to stick it up without wrecking my door? I can't be bothered getting it framed. --124.254.77.148 (talk) 06:32, 1 January 2009 (UTC)[reply]

Is it coming off because it's too heavy or because the Blu-Tack won't stick to it?
  • If it's because the poster is too smooth and the blutack won't stick to it - then you could permenantly stick some kind of backing to the back of your poster using a really strong glue (maybe stick a sheet of thin cardboard onto the back using PVA glue) - then stick that backing to the door using blutack. You'd actually only need to stick little pieces of backing to the poster in the corners where the blutack has to go.
  • If it's because the poster is too heavy then use many more blobs of blutack all over the back of the poster.
Incidentally - it's possible for the blutack to damage your door - it's kinda oily and that can affect some surface finishes. SteveBaker (talk) 07:26, 1 January 2009 (UTC)[reply]
Go to an office supplies store and get yourself something like this [34] Clip a couple on to the rim of your poster. You can then attach them with 3M "Command" strips or those blutack things to your door. If all else fails you can run a string through them and put a small nail or tack onto the top of your door (not the face) where qne can't see it. Then suspend your poster from that nail. 76.97.245.5 (talk) 10:03, 1 January 2009 (UTC)[reply]
Blu-tack can indeed damage walls and doors—I've often seen it leave behind nasty oily residues. I usually use Scotch tape myself—it doesn't damage the door or wall. One way to get around it damaging the poster is to put up little pieces of tape on the back of the poster that you then use double-sided tape on for the wall. If you do it that way you should be able to just peel off the poster later without any tape ripping it. --98.217.8.46 (talk) 12:57, 1 January 2009 (UTC)[reply]

What is this?

File:Dog feces.jpg

--Wmrwiki (talk) 14:06, 1 January 2009 (UTC)[reply]

troll food?86.4.182.202 (talk) 14:15, 1 January 2009 (UTC)[reply]
Picture size reduced. --Cookatoo.ergo.ZooM (talk) 14:40, 1 January 2009 (UTC)[reply]
Contemplation ---Sluzzelin talk 18:17, 1 January 2009 (UTC)[reply]
Defececation. ~AH1(TCU) 18:30, 1 January 2009 (UTC)[reply]

pharmaceutical oath?

I heard a short exchange of dialogue from 6teen while flipping through the channels last night, which inspires me to ask: Is there a law (or something like it) that prevents people working at drug stores (pharmacies) from telling everyone what people bought? i.e., if someone goes into a drug store and gets like 70 packs of condoms and a couple energy drinks (they're getting ready for an all-nighter), what stops them from chatting to their coworkers about it over a lunch break? On that show, they called it a "pharmaceutical oath," which sounded authentic, but Googling it gets no good results... flaminglawyerc 19:29, 1 January 2009 (UTC)[reply]

I doubt there is an oath. There are basic medical privacy laws, though, which would surely cover situations in which practitioners were exchanging patient medical information just for a laugh. --98.217.8.46 (talk) 19:53, 1 January 2009 (UTC)[reply]
I would be surprised if there's any rule that says they can't discuss what's happened at work. The privacy laws, I assume, would restrict them only from doing so in a way that allows individually identifying the person involved. I'm not a lawyer. --Trovatore (talk) 20:15, 1 January 2009 (UTC)[reply]

How tall is Danbo?

I'm making some papercrafts from a bunch of huge boxes I have, and wanted to make a life-size Danbo. Could someone find out (or hell, approximate) his height?

Thanks.

71.74.181.173 (talk) 22:09, 1 January 2009 (UTC)[reply]